Sei sulla pagina 1di 223

Theoretical Competition:

12 July 2011

Question 1

Page 1 of 3

1. A Three-body Problem and LISA

FIGURE 1 Coplanar orbits of three bodies.


1.1 Two gravitating masses M and m are moving in circular orbits of radii R and r ,
respectively, about their common centre of mass. Find the angular velocity !0 of the line
joining M and m in terms of R,r,M,m and the universal gravitational constant G .
[1.5 points]
1.2 A third body of infinitesimal mass is placed in a coplanar circular orbit about the same
centre of mass so that remains stationary relative to both M and m as shown in Figure 1.
Assume that the infinitesimal mass is not collinear with M and m . Find the values of the
following parameters in terms of R and r :
[3.5 points]
1.2.1 distance from to M .
1.2.2 distance from to m .
1.2.3 distance from to the centre of mass.
1.3 Consider the case M = m . If is now given a small radial perturbation (along O ), what
is the angular frequency of oscillation of about the unperturbed position in terms of 0 ?
Assume that the angular momentum of is conserved.
[3.2 points]

Theoretical Competition:

12 July 2011

Question 1

Page 2 of 3

The Laser Interferometry Space Antenna (LISA) is a group of three identical spacecrafts for
detecting low frequency gravitational waves. Each of the spacecrafts is placed at the corners of an
equilateral triangle as shown in Figure 2 and Figure 3. The sides (or arms) are about 5.0 million
kilometres long. The LISA constellation is in an earth-like orbit around the Sun trailing the Earth by
20o . Each of them moves on a slightly inclined individual orbit around the Sun. Effectively, the
three spacecrafts appear to roll about their common centre one revolution per year.
They are continuously transmitting and receiving laser signals between each other. Overall, they
detect the gravitational waves by measuring tiny changes in the arm lengths using interferometric
means. A collision of massive objects, such as blackholes, in nearby galaxies is an example of the
sources of gravitational waves.

Earth

FIGURE 2 Illustration of the LISA orbit. The three spacecraft roll about their centre of mass with a
period of 1 year. Initially, they trail the Earth by 20 . (Picture from D.A. Shaddock, An Overview
of the Laser Interferometer Space Antenna, Publications of the Astronomical Society of Australia,
2009, 26, pp.128-132.).

Theoretical Competition:

12 July 2011

Question 1

Page 3 of 3

C
Earth
A

FIGURE 3 Enlarged view of the three spacecrafts trailing the Earth. A, B and
C are the three spacecrafts at the corners of the equilateral triangle.

1.4 In the plane containing the three spacecrafts, what is the relative speed of one spacecraft with
respect to another?
[1.8 point]

Q1_THEORY_SOLUTION_1700_SENT_TO_LEADER.DOCX

Theoretical Competition:Solution
Question 1

Page 1 of 8

I. Solution
'
+2

+1
r1

r2

(1

R O

(2
m

1.1 Let O be their centre of mass. Hence

MR ! mr " 0

m%02 r "
M% R "
2
0

GMm

#R & r$

(2)

GMm

#R & r$

From Eq. (2), or using reduced mass, %02 "


Hence, !02 "

(1)

G # M & m$

#R & r$

G(M ! m)
GM
Gm
"
"
.
3
2
(R ! r )
r (R ! r )
R(R ! r )2

(3)

Q1_THEORY_SOLUTION_1700_SENT_TO_LEADER.DOCX

Theoretical Competition:Solution
Question 1

Page 2 of 8

1.2 Since ' is infinitesimal, it has no gravitational influences on the motion of neither M nor
m . For ' to remain stationary relative to both M and m we must have:

G # M & m$ '
GM '
Gm'
cos +1 & 2 cos +2 " '%02 * "
*
(4)
3
2
r1
r2
# R & r$
GM '
Gm'
sin +1 "
sin + 2
(5)
2
r1
r22
GM
Substituting 2 from Eq. (5) into Eq. (4), and using the identity
r1
sin +1 cos +2 & cos +1 sin +2 " sin(+1 & +2 ) , we get

sin(+1 & + 2 ) # M & m $


"
* sin +1
3
r22
#R & r$

(6)

The distances r2 and * , the angles +1 and + 2 are related by two SineRule equations

sin( 1

Substitute (7) into (6)

"

sin +1
R

(7)

sin( 1 sin #+1 & + 2 $


"
r2
R&r

# M & m$
1
R
"
r23 # R & r $4
m

(10)

m
R
"
,Eq. (10) gives
M &m R&r
(11)
r2 " R & r
Gm
By substituting 2 from Eq. (5) into Eq. (4), and repeat a similar procedure, we get
r2
(12)
r1 " R & r
Since

Alternatively,

r1
R
"
and
!
sin +1
sin #180 ! ) $

sin +1
R r
" , 2
r r1
sin + 2

"

r2
r
"
sin )
sin + 2

m r2
,
M r1
2

Q1_THEORY_SOLUTION_1700_SENT_TO_LEADER.DOCX

Theoretical Competition:Solution
Question 1

Page 3 of 8

Combining with Eq. (5) gives r1 " r2

Hence, it is an equilateral triangle with


( 1 " 60-

(13)

( 2 " 60The distance * is calculated from the Cosine Rule.

* 2 " r 2 & ( R & r )2 ! 2r ( R & r ) cos 60* " r 2 & rR & R 2

(14)

Alternative Solution to 1.2


Since ' is infinitesimal, it has no gravitational influences on the motion of neither M nor
m .For ' to remain stationary relative to both M and m we must have:
G # M & m$ '
GM '
Gm'
*
cos +1 & 2 cos + 2 " '% 2 * "
(4)
3
2
r1
r2
#R & r$

Note that

GM '
Gm'
sin +1 "
sin + 2
2
r1
r22
r1
R
"
!
sin +1
sin #180 ! ) $
r2
r
"
sin )
sin + 2
sin +1
R r
" , 2
r r1
sin + 2

(see figure)

"

m r2
,
M r1

r1 " r2
sin +1
m
"
sin + 2
M
(1 " ( 2

Equations (5) and (6):

(5)

The equation (4) then becomes:

M cos +1 & m cos + 2 "

# M & m$ r 2 *
3 1
#R & r$

(6)
(7)
(8)
(9)

(10)
3

Q1_THEORY_SOLUTION_1700_SENT_TO_LEADER.DOCX

Theoretical Competition:Solution
Question 1
Equations (8) and (10): sin #+1 & + 2 $ "
Note that from figure,

M & m r12 *
sin + 2
M # R & r $3

(11)

*
r
"
sin( 2
sin + 2

Equations (11) and (12): sin #+1 & + 2 $ "

Page 4 of 8

(12)

M & m r12 r
sin( 2
M # R & r $3

(13)

Also from figure,

#R & r$

" r22 ! 2r1r2 cos #+1 & +2 $ & r12 " 2r12 571 ! cos #+1 & +2 $68
(14)
sin( 2
Equations (13) and (14): sin #+1 & + 2 $ "
(15)
2 571 ! cos #+1 & + 2 $ 68
9

+1 & +2 " 180! !( 1 !( 2 " 180! ! 2( 2


1
cos( 2 " , ( 2 " 60! , ( 1 " 60!
2

(see figure)

Hence M and m from an equilateral triangle of sides # R & r $


Distance ' to M is R & r
Distance ' to m is R & r
2

Distance ' to O is * "

2
3 ;<
. R&r
/ :<
!
R
&
R
&
r
#
$
=
> "
0
1
2 @<
3 2
4 <?

R 2 & Rr & r 2

1.3 The energy of the mass " is given by

GM ' Gm' 1
d*
!
& 2 ' (( )2 & * 2% 2 )
..(15)
r1
r2
dt
Since the perturbation is in the radial direction, angular momentum is conserved
( r1 " r2 " # and m " M ),
E"!

2GM ' 1 . d * 2 *0 4%0 2 /


E"!
& 2 ' 0( ) &
1
2
*2 4
3 dt
..(16)
Since the energy is conserved,
dE
"0
dt
4

Q1_THEORY_SOLUTION_1700_SENT_TO_LEADER.DOCX

Theoretical Competition:Solution
Question 1

Page 5 of 8

*0 4%0 2 d *
dE 2GM ' d 2
d * d 2*
'
'
"
&
!
" 0 (17)
dt
dt dt 2
22 dt
* 3 dt
d2 d2 d * d * *
"
"
dt
d * dt
dt 2

.(18)

*0 4%0 2 d *
dE 2GM ' d *
d * d 2*
"
*
&'
!'
" 0 .(19)
dt
23
dt
dt dt 2
* 3 dt
R O R
d*
A 0 , we have
Since
dt
2GM
d 2 * *0 4%0 2
*
&
!
" 0 or
23
dt 2
*3
*0 4%0 2
d 2*
2GM
.
(20)
"
!
&
*
23
dt 2
*3
. B2 /
. B* /
The perturbation from 20 and * 0 gives 2 " 20 0 1 &
.
1 and * " * 0 0 1 &
20 4
* 0 14
3
3
Then
d 2* d 2
"
( *0 & B* ) " !
dt 2 dt 2

. B* /
*0 4%0 2
*0 01 &
1&
3
3
*0 4
3
B2 /
B* /
3.
3.
*0 01 &
20 0 1 &
1
1
20 4
*0 4
3
3
2GM

(21)

Using binomial expansion (1 & C )n D 1 & nC ,

d 2 B*
2GM . B* /. 3B2 /
3B* /
2.
" ! 3 *0 01 &
101 !
1 & *0%0 01 !
1.
2
dt
*0 43
*0 4
20
20 4
3
3
2
Using B* " B2 ,
*
d 2 B*
2GM . B* 3*0 B* /
3B* /
2.
" ! 3 *0 01 &
!
1 & *0%0 01 !
1.
2
2
dt
20
*0
20 4
*0 4
3
3
2GM
Since %02 "
,
230

.(22)

.(23)

. B* 3*0 B* /
. 3B* /
d 2 B*
2
" !%02 *0 01 &
!
1 & %0 *0 01 !
1
2
2
dt
*0
20 4
*0 4
3
3

.(24)

. 4B* 3*0 B* /
d 2 B*
" !%02 *0 0
!
1
2
202 4
dt
3 *0

.(25)
5

Q1_THEORY_SOLUTION_1700_SENT_TO_LEADER.DOCX

Theoretical Competition:Solution
Question 1
.
3*02 /
d 2 B*
2
"
!
%
B
*
4
!
0
1
0
dt 2
202 4
3

.(26)

From the figure, *0 " 20 cos30- or

*02

20

"

3
,
4

d B*
9/
7
.
" !%02 B* 0 4 ! 1 " ! %02 B* .
2
dt
44
4
3
2

Angular frequency of oscillationis


Alternative solution:

M " m gives R " r and %0 2 "

Page 6 of 8

.(27)

7
%0 .
2

G( M & M ) GM
. The unperturbed radial distance of ' is
"
( R & R )3
4R3

3R ,

so the perturbed radial distance can be represented by 3R & E where E FF 3R as shown in


the following figure.
2GM '
d2
(
3
R
&
E
)
"
'
( 3R & E ) ! '% 2 ( 3R & E ) .
Using Newtons 2 nd law, ! 2
2
2 3/2
dt
{R & ( 3R & E ) }
(1)
The conservation of angular momentum gives '%0 ( 3R)2 " '%( 3R & E )2 .
(2)
Manipulate (1) and (2) algebraically, applying E 2 D 0 and binomial approximation.

%0 2 3R
2GM
d 2E
(
3
R
&
E
)
"
!
dt 2 (1 & E / 3R)3
{R 2 & ( 3R & E )2 }3/2

%0 2 3R
2GM
d 2E
&
D
!
(
3
E
)
R
dt 2 (1 & E / 3R)3
{4 R 2 & 2 3E R}3/2

%0 2 3R
GM
(1 & E / 3R)
d 2E
! 3 3R
" 2 !
4R
(1 & 3E / 2 R)3/2 dt
(1 & E / 3R)3
. 3 3E / .
E / d 2E
3E /
.
!%0 2 3R 001 !
D 2 ! %0 2 3R 01 !
11 01 &
1
1
4R 4 3
3R 4 dt
3R 4
3
3
d2
7
E " ! .0 %0 2 /1 E
2
dt
34
4

1.4 Relative velocity

Q1_THEORY_SOLUTION_1700_SENT_TO_LEADER.DOCX

Theoretical Competition:Solution
Question 1

Page 7 of 8

Let v = speed of each spacecraft as it moves in circle around the centre O.


The relative velocities are denoted by the subscripts A, B and C.
For example, vBA is the velocity of B as observed by A.
The period of circular motion is 1 year T " 365 , 24 , 60 , 60 s.
2G
The angular frequency % "
T
L
" 575 m/s
The speed v " %
2cos30-

(28)

(29)

The speed is much less than the speed light ! Galilean transformation.
In Cartesian coordinates, the velocities of B and C (as observed by O) are

vCB

v
vCA

vBA

vBC

vAB

vAC

"
For B, vB " v cos 60-i ! v sin 60-j

Q1_THEORY_SOLUTION_1700_SENT_TO_LEADER.DOCX

Theoretical Competition:Solution
Question 1

Page 8 of 8

"
For C, vC " v cos 60-i & v sin 60-j

"
Hence vBC " !2v sin 60-j " ! 3vj
The speed of B as observed by C is 3v D 996 m/s

(30)

Notice that the relative velocities for each pair are anti-parallel.
Alternative solution for 1.4
One can obtain vBC by considering the rotation about the axis at one of the spacecrafts.

vBC " % L "

2G
(5 ,106 km) D 996 m/s
365 , 24 , 60 , 60 s

Theoretical Competition: 12 July 2011


Question 2

Page 1 of 2

2. An Electrified Soap Bubble


A spherical soap bubble with internal air density !i , temperature Ti and radius R0 is surrounded by
air with density !a , atmospheric pressure Pa and temperature Ta . The soap film has surface tension
! , density !s and thickness t . The mass and the surface tension of the soap do not change with the

temperature. Assume that R0 ! t .


The increase in energy, dE , that is needed to increase the surface area of a soap-air interface by dA ,
is given by dE = !dA where ! is the surface tension of the film.

2.1 Find the ratio

!iTi
!aTa

in terms of ! , Pa and R0 .

2.2 Find the numerical value of

!iTi
!aTa

[1.7 point]

! 1 using ! = 0.0250 Nm!1, R0 = 1.00 cm , and

Pa =1.013 !105 Nm"2 .

[0.4 point]

2.3 The bubble is initially formed with warmer air inside. Find the minimum numerical value
of Ti such that the bubble can float in still air. Use Ta = 300 K , !s = 1000 kgm!3 ,
!a = 1.30 kgm!3 , t = 100 nm and g = 9.80 ms!2 .

[2.0 points]

After the bubble is formed for a while, it will be in thermal equilibrium with the surrounding. This
bubble in still air will naturally fall towards the ground.
2.4 Find the minimum velocity u of an updraught (air flowing upwards) that will keep the
bubble from falling at thermal equilibrium. Give your answer in terms of !s , R0, g, t and
the airs coefficient of viscosity ! . You may assume that the velocity is small such that
Stokess law applies, and ignore the change in the radius when the temperature lowers to
the equilibrium. The drag force from Stokes Law is F = 6!"R0u .
[1.6points]
2.5 Calculate the numerical value for u using ! =1.8!10"5 kgm"1 s"1 .

[0.4 point]

The above calculations suggest that the terms involving the surface tension ! add very little to the
accuracy of the result. In all of the questions below, you can neglect the surface tension terms.

Theoretical Competition: 12 July 2011


Question 2

Page 2 of 2

2.6 If this spherical bubble is now electrified uniformly with a total charge q , find an equation
describing the new radius R1 in terms of R0, Pa , q and the permittivity of free space !0 .
[2.0points]
2.7 Assume that the total charge is not too large (i.e.

q2
<< Pa ) and the bubble only
!0R04

experiences a small increase in its radius, find !R where R1 = R0 + !R .


n

Given that (1 + x) ! 1 + nx where x ! 1 .

[0.7 point]

2.8 What must be the magnitude of this charge q in terms of t, !a , !s , !0,R0,Pa in order that the
bubble will float motionlessly in still air? Calculate also the numerical value of q . The
permittivity of free space !0 = 8.85!10"12 farad/m .

[1.2 point]

Q2_THEORY_SOLUTION_2300_SENT TO LEADER.DOCX

Theoretical Competition: Solution


Question 2
Page 1 of 7

2. SOLUTION
2.1. The bubble is surrounded by air.

Pi , Ti , 'i
R0

Pa , Ta , 'a

O
's , t

Cutting the sphere in half and using the projected area to balance the forces
give
Pi# R02 % Pa# R02 & 2 ! 2# R0$ "
Pi % Pa &

4$
R0

(1)

The pressure and density are related by the ideal gas law:
PV % nRT or P %

' RT
M

, where M = the molar mass of air.

(2)

Apply the ideal gas law to the air inside and outside the bubble, we get
M
R
M
' aTa % Pa ,
R

'iTi % Pi

(
'iTi
P
4$ )
% i % *1 &
+
'aTa
Pa
, R0 Pa -

(3)
1

Q2_THEORY_SOLUTION_2300_SENT TO LEADER.DOCX

Theoretical Competition: Solution


Question 2
Page 2 of 7

2.2. Using $ % 0.025 Nm.1 , R0 %1.0 cm and Pa %1.013 /105 Nm.2 , the numerical value
of the ratio is
'iTi
4$
% 1&
% 1 & 0.0001
' aTa
R0 Pa

(4)

(The effect of the surface tension is very small.)


2.3. Let W = total weight of the bubble, F = buoyant force due to air around the
bubble
W % ! mass of film+mass of air " g
4
0
1
% 2 4# R02 ' s t & # R03 'i 3 g
3
4
5
'T (
4
4$ )
% 4# R02 ' s tg & # R03 a a *1 &
+g
3
Ti , R0 Pa -

(5)

The buoyant force due to air around the bubble is


B%

4 3
# R0 'a g
3

(6)

If the bubble floats in still air,


B 6W

'T
4
4
# R03 'a g 6 4# R02 ' s tg & # R03 a a
3
3
Ti

(7)

(
4$ )
*1 &
+g
R
P
0 a ,

Rearranging to give
R0 ' aTa (
4$ )
1&
*
R0 ' a . 3 ' st , R0 Pa +6 307.1 K

Ti 6

(8)

The air inside must be about 7.17C warmer.


2

Q2_THEORY_SOLUTION_2300_SENT TO LEADER.DOCX

Theoretical Competition: Solution


Question 2
Page 3 of 7

2.4. Ignore the radius change ! Radius remains R0 % 1.0 cm


(The radius actually decreases by 0.8% when the temperature decreases
from 307.1 K to 300 K. The film itself also becomes slightly thicker.)
The drag force from Stokes Law is F % 6#8 R0u

(9)

If the bubble floats in the updraught,


F 6W . B

4
4
0
1
6#8 R0u 6 2 4# R02 ' st & # R03'i 3 g . # R03' a g
3
3
4
5

(10)

When the bubble is in thermal equilibrium Ti % Ta .


0
(
4
4$ ) 1
4
3
6#8 R0u 6 2 4# R02 ' st & # R03' a *1 &
3 g . # R0 ' a g
+
3
3
R
P
0 a -5
,
4

Rearranging to give
0 4$ 1
4 2
R0 ' a g 2
3
R0 Pa 5
4 R0 ' s tg 3
4
&
u6
68
68

(11)

2.5. The numerical value is u 6 0.36 m/s .


The 2nd term is about 3 orders of magnitude lower than the 1 st term.

From now on, ignore the surface tension terms.


2.6. When the bubble is electrified, the electrical repulsion will cause the bubble
to expand in size and thereby raise the buoyant force.
The force/area is (e-field on the surface charge/area)
There are two alternatives to calculate the electric field ON the surface of
3

Q2_THEORY_SOLUTION_2300_SENT TO LEADER.DOCX

Theoretical Competition: Solution


Question 2
Page 4 of 7

the soap film.


A. From Gausss Law
Consider a very thin pill box on the soap surface.

Pi; , Ta 'i;
O R1

Pa , Ta , 'a

E = electric field on the film surface that results from all other parts of the

soap film, excluding the surface inside the pill box itself.
Eq = total field just outside the pill box =

q
4#: 0 R

= E + electric field from surface charge 9


= E & E9

2
1

Using Gausss Law on the pill box, we have E9 %


as a result of symmetry.
Therefore, E % Eq . E9 %

9 9
9
1
q
.
%
%
: 0 2: 0 2: 0 2: 0 4# R12

9
:0

9
perpendicular to the film
2: 0

(12)

Q2_THEORY_SOLUTION_2300_SENT TO LEADER.DOCX

Theoretical Competition: Solution


Question 2
Page 5 of 7

B. From direct integration


R=<

<

oO

charge=q
0 q 1
=q % 2
2# R sin < .R=<
2 3
4 4# R 5

To find the magnitude of the electrical repulsion we must first find the electric
field intensity E at a point on (not outside) the surface itself.
!!!"

Field at A in the direction OA is

= EA %

EA %

1
4#: 0

!q

!q

4# R12 " 2# R12 sin <=<

<1
0
2 2 R1 sin 3
25
4

4# R12 " < %180

2: 0

>

< %0

< 0< 1

cos d 2 3 %
2 425

sin

!q

<
2

!q

4# R12 "
2: 0

4# R12 "
2: 0

< 0< 1
cos = 2 3
2 425

(13)

The repulsive force per unit area of the surface of bubble is


q 4# R12 "
!
0 q 1
E %
2
2 3
2: 0
4 4# R1 5

(14)

Let Pi; and 'i; be the new pressure and density when the bubble is electrified.
This electric repulsive force will augment the gaseous pressure Pi; .
Pi; is related to the original Pi through the gas law.
4
4
Pi; # R13 % Pi # R03
3
3
5

Q2_THEORY_SOLUTION_2300_SENT TO LEADER.DOCX

Theoretical Competition: Solution


Question 2
Page 6 of 7

0R 1
0R 1
Pi; % 2 0 3 Pi % 2 0 3 Pa
4 R1 5
4 R1 5

(15)

In the last equation, the surface tension term has been ignored.
From balancing the forces on the half-sphere projected area, we have (again
ignoring the surface tension term)

!q
P;&
i

0R 1
Pa 2 0 3 &
4 R1 5

4# R12 "

% Pa

2: 0

! q 4# R12 "

(16)

2: 0

% Pa

Rearranging to get
4

0 R1 1 0 R1 1
q2
.
.
% 0
2 3 2 3
2
4
4 R0 5 4 R0 5 32# : 0 R0 Pa

Note that (17) yields

(17)

R1
% 1 when q % 0 , as expected.
R0

2.7. Approximate solution for R1 when

q2
32# 2: 0 R04 Pa

?? 1

Write R1 % R0 & @R, @R ?? R0


4

R
@R 0 R 1
@R
Therefore, 1 %1 & , 2 1 3 A 1 & 4
R0
R0 4 R0 5
R0

Eq. (17) gives:


@R A

q2
96# 2: 0 R03 Pa

R1 A R0 &

0
1
q2
R
A
&
1
3
02
2
3
2
4
96# : 0 R0 Pa
4 96# : 0 R0 Pa 5
q2

(18)

(19)

(20)
6

Q2_THEORY_SOLUTION_2300_SENT TO LEADER.DOCX

Theoretical Competition: Solution


Question 2
Page 7 of 7

2.8. The bubble will float if


B 6W

(21)

4
4
# R13 'a g 6 4# R02 ' s tg & # R03 'i g
3
3
0

Initially, Ti % Ta B 'i % 'a for $ C 0 and R1 % R0 21 &


4

@R 1
3
R0 5

0 @R 1
4
4
2
3
# R03 2 1 &
3 ' a g 6 4# R0 ' s tg & # R0 ' a g
3
R
3
0 5
4
4
# ! 3@R " ' a g 6 4# R02 ' stg
3
4
3q 2
#
' a g 6 4# R02 ' s tg
2
3 96# : 0 R0 Pa
q2 6

(22)

96# 2 R03 ' s t: 0 Pa

'a

q A 256 / 10.9 C A 256 nC

Note that if the surface tension term is retained, we get


0
2
2
2
4
2 q 96# : 0 R0 Pa
R1 A 21 &
( 2 0 4$ 1 )
2
1& 2
*
3+
2
3 4 R0 Pa 5 ,
4

1
3
3
3 R0
3
3
5

Theoretical Competition:

12 July 2011

Question 3

Page 1 of 2

3. To Commemorate the Centenary of Rutherfords Atomic Nucleus:


the Scattering of an Ion by a Neutral Atom
Ion, m,Q

Ion, m,Q

v0

v
!
r

rmin
Atom, M

FIGURE 1
An ion of mass m , charge Q , is moving with an initial non-relativistic speed v 0 from a great
distance towards the vicinity of a neutral atom of mass M >> m and of electrical polarisability ! .
The impact parameter is b as shown in Figure 1.
!
The atom is instantaneously polarised by the electric field E of the in-coming (approaching) ion.
!
!
The resulting electric dipole moment of the atom is p = !E . Ignore any radiative losses in this
problem.

( )

!
!
3.1 Calculate the electric field intensity E p at a distance r from an ideal electric dipole p at the
!

origin O along the direction of p in Figure 2.

[1.2 points]

+q

!
p

p = 2aq, r ! a

!q
FIGURE 2

Theoretical Competition:

12 July 2011

Question 3

Page 2 of 2

!
3.2 Find the expression for the force f acting on the ion due to the polarised atom. Show that this
force is attractive regardless of the sign of the charge of the ion.
[3.0 points]
3.3 What is the electric potential energy of the ion-atom interaction in terms of !,Q and r ?
[0.9 points]
3.4 Find the expression for rmin , the distance of the closest approach, as shown in Figure 1.
[2.4 points]
3.5 If the impact parameter b is less than a critical value b0 , the ion will descend along a spiral to
the atom. In such a case, the ion will be neutralized, and the atom is, in turn, charged. This process
is known as the charge exchange interaction. What is the cross sectional area A = !b02 of this
charge exchange collision of the atom as seen by the ion?
[2.5 points]

Theoretical Competition:
Question 3

Solution
Page 1 of 3

QUESTION 3: SOLUTION
1. Using Coulombs Law, we write the electric field at a distance r is given by
q
q
Ep #
$
2
4!" 0 (r $ a)
4!" 0 (r % a) 2

&
'
(
)
1
1
q (
)
Ep #
$
4!" 0 r 2 ( & a '2 & a ' 2 )
( (1 $ ) (1 % ) )
** r + * r + +
Using binomial expansion for small a ,

2a '
& 2a
$1% )
(1 %
4!" 0 r *
r
r +
4qa
qa
=+
=+
3
4!" 0 r
!" 0 r 3

Ep #

.(1)

2p
4!" 0 r 3

..(2)

2. The electric field seen by the atom from the ion is


!
Q
.. (3)
Eion # $
r
4!" 0 r 2
The induced dipole moment is then simply
!
,Q
!
.. (4)
p # , Eion # $
r
4!" 0 r 2
From eq. (2)
!
2p
Ep #
r
4!" 0 r 3
!
The electric field intensity E p at the position of an ion at that instant is, using eq. (4),
!
Ep #

- 2, Q
/$
4!" 0 r 1 4!" 0 r 2
1

.
,Q
r 0 # $ 2 2 5 r
8! " 0 r
2

The force acting on the ion is

!
!
, Q2
f # QE p # $ 2 2 5 r
8! " 0 r

.. (5)

The $ sign implies that this force is attractive and Q 2 implies that the force is attractive regardless
of the sign of Q .

Theoretical Competition:
Question 3

Solution
Page 2 of 3

3 !
!
3. The potential energy of the ion-atom is given by U # 4 f .dr .(6)
r

! !
f .dr # $

,Q
(7)
32! 2" 02 r 4
r
! !
[Remark: Students might use the term !p " E which changes only the factor in front.]

Using this, U # 4

4. At the position rmin we have, according to the Principle of Conservation of Angular Momentum,

mvmax rmin # mv0b


b
vmax # v0
rmin
And according to the Principle of Conservation of Energy:
1 2
$, Q 2
1
mvmax %
# mv02
2 2 4
2
32! " 0 rmin 2
Eqs.(8) & (9):

.. (8)

.. (9)

2 1
2
2
4
& b ' , Q 2 mv0 & b '
(
) $
(
) #1
32! 2" 02b4 * rmin +
* rmin +

,Q
& rmin ' & rmin '
# 0
(
) $(
) %
2 2
2 4
* b + * b + 16! " 0 mv0 b
2

.. (10)

The roots of eq. (10) are:

rmin #

b , Q2
/1 5 1 $ 2 2 2 4
4! " 0 mv0 b
2 /1

.1
02
02

.. (11)

[Note that the equation (8) implies that rmin cannot be zero, unless b is itself zero.]
Since the expression has to be valid at Q # 0 , which gives
1
b
61 5 17 2
2
We have to choose + sign to make rmin # b

rmin #

Hence,

rmin #

.1
b ,Q 2
/1 % 1 $ 2 2 2 4 0 2
4! " 0 mv0 b 02
2 /1

.....(12)

Theoretical Competition:
Question 3

Solution
Page 3 of 3

5. A spiral trajectory occurs when (12) is imaginary (because there is no minimum distance of
approach).
rmin is real under the condition:

, Q2
18
4! 2" 02 mv02b 4
1

& , Q2 ' 4
b 8 b0 # ( 2 2 2 )
* 4! " 0 mv0 +

.. (13)

& , Q2 ' 4
For b 9 b0 # ( 2 2 2 ) the ion will collide with the atom.
* 4! " 0 mv0 +
Hence the atom, as seen by the ion, has a cross-sectional area A ,
1

& , Q2 ' 2
A # ! b02 # ! ( 2 2 2 )
* 4! " 0 mv0 +

.. (14)

Experimental Competition:

14 July 2011

Problem 1

Page 1 of 4

1. Electrical Blackbox: Capacitive Displacement Sensor


For a capacitor of capacitance C which is a component of a relaxation oscillator whose frequency
of oscillation is f , the relationship between f and C is as follows:
f =

!
C +C S

where ! is a constant and C S is the stray capacitance of our circuits. The frequency f can be
monitored using a digital frequency meter.
The electrical blackbox given in this experiment is a parallel plate capacitor. Each plate consists of
a number of small teeth of the same geometrical shape. The value of C can be varied by displacing
the upper plate relative to the lower plate, horizontally. Between the two plates there is a sheet of
dielectric material.
Equipment: a relaxation oscillator, a digital multimeter for measuring frequency of the relaxation
oscillator, a set of capacitors of known capacitances, an electrical blackbox and a
battery.
Caution: Check the voltage of the battery and ask for a new one if the voltage is less than 9 V.
Do not forget to switch on.

Battery

Relaxation oscillator

Electrical connectors to
the plates
Electrical blackbox:
Parallel plate capacitor

Sliding upper plate

Switch
Frequency output

Connectors to capacitor
FIGURE 1

Experimental Competition:

14 July 2011

Problem 1

Page 2 of 4

FIGURE 2 Capacitors

The position for frequency


measurements

FIGURE 3 Digital multimeter for measuring frequency

TABLE 1 Nominal Capacitance values


Code
33J
68
82J
151

Capacitance value
(pF)
34 1
68 1
84 1
150 1

Experimental Competition:

14 July 2011

Problem 1

Page 3 of 4

Part 1. Calibration
Perform the measurement of f using the given capacitors of known capacitances. Draw appropriate
graph to find the value of ! and C S . Error analysis is not required.

[3.0 points]

Part 2. Determination of geometrical shape of a parallel plate capacitor


[6.0 points]
Given the three possible geometrical shapes as Pattern I, Pattern II and Pattern III as follows:
lower plate

Top view

upper plate

slide in and out

Pattern I

lower plate

upper plate

Pattern II

Top view

slide in and out

Experimental Competition:

14 July 2011

Problem 1

Page 4 of 4

lower plate

upper plate

Top view

slide in and out

Pattern III

For each pattern, draw qualitatively an expected graph of C versus the positions of the upper plate
but label the x-axis. Then, perform the measurement of f versus the positions of the upper plate.
Plot graphs and, from these graphs, deduce the pattern of the parallel plate capacitor and its
dimensions (values of b and w ). The separation (d )between the upper and lower plates is 0.20 mm.
The dielectric sheet between the plates has a dielectric constant K = 1.5 . The permittivity of free
space !0 = 8.85!10"12 Fm"1 . Error analysis is not required.
Part 3. Resolution of digital calipers

[1.0 point]

As the relative position of the parallel plates is varied, the capacitance changes with a pattern. This
set-up may be used as digital calipers for measuring length. If the parallel plate capacitor in this
experiment is to be used as digital calipers, estimate from the experimental data in Part 2 its
resolution: the smallest distance that can be measured for the frequency value f ! 5 kHz . An error
estimate for the final answer is not required.

Q1_EXPERIMENT_SOLUTION_FINAL.DOCX

Experimental Competition:
Question 1

14 July 2011
Page 1 of 7

Part 1. Calibration
From the relationship between f and C given,

f =

!
C +C S

That is, theoretically, the graph of

C
1
1
= C+ S
f
!
!

1
on the Y-axis versus C on the X-axis should be linear of
f

CS
1
respectively.
and
!
!
The table below shows the measured values of C (plotted on the X-axis,) f and,

which the slope and the Y-intercept is

1
, which is plotted on the Y-axis.
f

(pF) (kHz)
33 13.94
68 8.30
82 6.99
151 4.17
233 2.79
219 2.98
184 3.48
150 4.20
115 5.24
101 5.89

1 f
(ms)
0.0717
0.1205
0.1431
0.2398
0.3584
0.3356
0.2874
0.2381
0.1908
0.1698

1/f vs. C graph


0.4000
0.3500
1/f (1/kHz)

additionally,

y = 0.0014x + 0.0251

0.3000
0.2500
0.2000
0.1500
0.1000
0.0500
0.0000
0

50

100

150

200

250

C (pF)

From this graph, the slope (1 !) and the Y-intercept (C S !) is equal to 0.0014 s/nF and
0.0251 ms respectively.
Hence,
and

1
1
=
= 714 nF/s
slope 0.0014 s / nF
Y - intercept
0.0251 ms
=
= 17.9 pF
CS =
0.0014 s / nF
slope
! =

as required.

Q1_EXPERIMENT_SOLUTION_FINAL.DOCX

Experimental Competition:
Question 1

14 July 2011
Page 2 of 7

Part II. Determination of geometrical shape of parallel-plates capacitor


PATTERN I: The expected graph of C versus the position

2w

3w

4w

5w

Distance

5w

Distance

5w

Distance

PATTERN II: The expected graph of C versus the position

2w

3w

4w

PATTERN III: The expected graph of C versus the position

2w

3w

4w

Q1_EXPERIMENT_SOLUTION_FINAL.DOCX

Experimental Competition:
Question 1

14 July 2011
Page 3 of 7

By measuring f and C versus x (the distance moved between the two plates,) the data and
the graphs are shown below.
x (mm)

f (kHz)

C (pF)

x (mm)

f (kHz)

C (pF)

0
1
2
3
4
5
6
7
8
9
10
11
12
13
14
15
16
17
18
19
20
21
22
23
24
25
26
27
28
29

7.41
8.09
8.64
9.30
9.30
8.21
7.02
6.40
5.98
5.91
6.38
6.96
7.61
8.40
8.20
7.13
6.37
5.96
5.38
5.33
5.72
6.34
6.85
7.53
7.23
6.33
5.56
5.36
4.73
4.53

77.9
69.8
64.2
58.3
58.3
68.5
83.3
93.1
100.9
102.4
93.5
84.1
75.4
66.5
68.6
81.7
93.6
101.3
114.3
115.5
106.4
94.2
85.8
76.4
80.3
94.3
110.0
114.8
132.5
139.2

30
31
32
33
34
35
36
37
38
39
40
41
42
43
44
45
46
47
48
49
50
51
52
53
54
55
56
57
58

4.94
5.52
6.19
6.48
6.64
5.72
5.08
4.39
4.06
3.97
4.32
4.86
5.33
6.05
5.98
5.14
4.47
3.93
3.74
3.64
3.93
4.30
4.91
5.46
5.49
4.64
4.07
3.62
3.36

126.1
110.9
96.9
91.7
89.1
106.4
122.1
144.2
157.4
161.4
146.8
128.5
115.5
99.6
100.9
120.5
141.3
163.3
172.5
177.7
163.3
147.6
127.0
112.3
111.6
135.4
157.0
178.8
194.1

Q1_EXPERIMENT_SOLUTION_FINAL.DOCX

f (kHz)

Experimental Competition:
Question 1

14 July 2011
Page 4 of 7

f vs. x graph

10.00
9.00
8.00
7.00
6.00
5.00
4.00
3.00
2.00
x (mm)

1.00
0.00
10

20

30

40

50

60

70

C vs. x graph

C (pF)

250.0

200.0

150.0

100.0

50.0
x (mm)
0.0
0

10

20

30

40

50

60

70

Q1_EXPERIMENT_SOLUTION_FINAL.DOCX

Experimental Competition:
Question 1

14 July 2011
Page 5 of 7

From periodicity of the graph, period = 1.0 cm


Simple possible configuration is:

0.5 cm

1.0 cm

node
1
2
3
4
5

Cmax
105.1
118.6
139.5
163.7
182.1

200

Cmax (pF)

The peaks of C values obtained from the C vs. x graph are provided in the table below.
These maximum C are plotted (on the Y-axis) vs. nodes (on the X-axis.)

Cmax vs. Node graph

180
160

y = 19.924x + 82.04

140
120
100
80
60
40
20

Node

0
0

This graph is linear of which the slope is the dropped off capacitance !C = 19.9
pF/section.
Given that the distance between the plates d = 0.20 mm, K = 1.5 ,
!C "

and

K !0A

,
d
A = 5!10"3 m ! (b mm) !10"3 m 2

Q1_EXPERIMENT_SOLUTION_FINAL.DOCX

Experimental Competition:
Question 1
Then, b (mm) !

("C )d

K !0 !10"3 !5!10"3
dielectric of which K = 1.5 .

# 60 mm

14 July 2011
Page 6 of 7

if medium between plates is the

Part III. Resolution of digital micrometer


From the given relationship between f and C, f =
!f !

"

!
,
C +C S

df
"!
!C =
!C
dC
(C +C S )2
f2
= !C
!
!
!C = 2 !f
f

And since C linearly depends on x , C = mx + !

"C = m"x .

Hence,

!
!f ,
mf 2
where !f is the smallest change of the frequency f which can be detected by the multimeter,
x 0 is the operated distance at f = 5 kHz, and m is the gradient of the C vs. x graph at
!x =

x = x0 .
From the f vs. x graph, at f = 5 kHz, the gradient is then measured on the C vs. x graph
around this range.

Q1_EXPERIMENT_SOLUTION_FINAL.DOCX

Experimental Competition:
Question 1

14 July 2011
Page 7 of 7

C vs. x graph 2
250.0

200.0

C (pF)

y = 17.455x - 504.54
150.0

100.0

50.0

0.0
0

10

20

30

40

50

60

70

x (mm)

From this graph, m = 17.5 pF / mm = 1.75!10"8 F / m .


Using this value of m, f = 5 kHz , ! = 714 nF/s , and !f = 0.01 kHz ,

714 !10"9
!x =
!(0.01!103 ) = 0.016 mm
"8
3 2
(1.75!10 )(5!10 )
NB. The C vs. x graph is used since C (but not f ) is linearly related to x .
Alternative method for finding the resolution
(not strictly correct)
Using the f vs. x graph and the data in the table around f = 5 kHz , it is found that when f
is changed by 1 kHz ( !f = 1 kHz ,) x is roughly changed by 1.5 mm ( !x ! 1.5 mm .)
Hence, when f is changed by !f = 0.01 kHz (the smallest detectable of the change,) the
distance moved is !x ! 0.015 mm .

Experimental Competition:
Problem 2

14 July 2011
Page 1 of 2

2. Mechanical Blackbox: a cylinder with a ball inside


A small massive particle (ball) of mass m is fixed at distance z below the top of a long hollow
cylinder of mass M . A series of holes are drilled perpendicularly to the central axis of the cylinder.
These holes are for pivoting so that the cylinder will hang in a vertical plane.
Students are required to perform necessary nondestructive measurements to determine the numerical
values of the following with their error estimates:
i.

position of centre of mass of cylinder with ball inside.


Also provide a schematic drawing of the experimental set-up for measuring the centre of
mass.
[1.0 points]

ii.
iii.
iv.

distance z
ratio M m .
the acceleration due to gravity, g .

[3.5 points]
[3.5 points]
[2.0 points]

Equipment: a cylinder with holes plus a ball inside, a base plate with a thin pin, a pin cap, a ruler, a
stopwatch, thread, a pencil and adhesive tape.

z
O

pivot

xCM

Base plate
to be clamped
to a table top

CM

Thin pin
for pivoting

xCM is the distance from the top of the cylinder to the


centre of mass.
R is the distance from the pivoting point to the centre of
mass.

Experimental Competition:
Problem 2

14 July 2011
Page 2 of 2

Cylinder with holes


plus ball inside

Adhesive
tape

Pin cap

Stopwatch

Thread
(for balancing)

Base plate

Ruler

Caution: The thin pin is sharp. When it is not in use, it should be protected with a pin cap for safety.
Useful information:

d 2!
! " g (M + m ) R! , where ICM is the
dt 2
moment of inertia of the cylinder with a ball about the centre of mass and ! is the angular
displacement.
2. For a long hollow cylinder of length L and mass M , the moment of inertia about the centre of
2
1 !# L $&
mass with the rotational axis perpendicular to the cylinder can be approximated by M ## && .
3 #" 2 &%
1. For such a physical pendulum,

(M + m ) R2 + ICM

3. The parallel axis theorem: I = I centre of mass + Mx 2 , where x is the distance from the rotation
point to the centre of mass, and M is the total mass of the object.
4. The ball can be treated as a point mass and it is located on the central axis of the cylinder.
5. Assume that the cylinder is uniform and the mass of the end-caps is negligible.

Q2_EXPERIMENT_SOLUTION_FINAL.DOCX

Experimental Competition:
Question 2

14 July 2011
Page 1 of 9

Solution: 2 . Mechanical Blackbox: a cylinder with a ball inside

z
O

pivot

xCM
R

CM

m
M

In order to be able to calculate the required values in i, ii, iii, we need to know:
a. the position of the centre of mass of the tubing plus particle (object) which depends on
z, m, M
b. the moment of inertia of the above.
The position of the CM may be found by balancing. The I CM can be calculated from the period
of oscillation of the tubing plus object.
Analytical steps to select parameters for plotting
I.

xCM =

( )

mz + M L 2
m+ M

(1)

L is readily obtainable with a ruler.

xCM is determined by balancing the tubing and object.

Q2_EXPERIMENT_SOLUTION_FINAL.DOCX

Experimental Competition:
Question 2

14 July 2011
Page 2 of 9

II. For small-amplitude oscillation about any point O the period T is given by considering the
equation:

{( M + m) R

T = 2

I CM + ( M + m ) R 2
g ( M + m) R

where

I CM

Note that

T2

+ ICM !!! = ! g M + m Rsin ! ! " g M + m R!

. (2)

. (3)

1 L
L
2

= M + M xCM + m ( z xCM )
3 2
2

1 2
2
2
. (4)
= ML + MxCM
MLxCM + m ( z xCM )
3

g ( M + m)
I
= CM + ( M + m ) R
2
4
R

. (5)

Method (a): (linear graph method)


The equation (5) may be put in the form:

4 2 2
4 2 ICM
T R =
R +
( M + m) g
g
2
2
Hence the plot of T R v.s. R will yield the straight line whose
4 2
Slope =
g
4 2 I CM
and y-intercept =
(M + m) g
2

Hence,

I CM = ( M + m )

4! 2
The value of g is from equation (7): g =
"

. (6)

. (7)
. (8)
. (9)
. (10)

Q2_EXPERIMENT_SOLUTION_FINAL.DOCX

Experimental Competition:
Question 2

14 July 2011
Page 3 of 9

Method (b): minimum point curve method


The equation (5) implies that T has a minimum value at

R = Rmin

ICM
M +m

. (11)

Hence Rmin can be obtained from the graph T v.s. R .


And therefore

2
ICM = ( M + m) Rmin

. (12)

This equation (12) together with equation (1) will allow us to calculate the required values z
and M m .

g ( M + m)
= ( M + m ) Rmin + ( M + m ) Rmin
4 2
2 Rmin
8 2 Rmin
2
. (13)
g =

=
2
2
Tmin
Tmin

2
At the value R = Rmin equation (5) becomes Tmin

from which g can be calculated.

Q2_EXPERIMENT_SOLUTION_FINAL.DOCX

Experimental Competition:
Question 2

14 July 2011
Page 4 of 9

Results
L = 30.0 cm 0.1 cm

xCM = 17.8 cm 0.1 cm (from top)


xCM R
(cm)

time (s) for 20 cycles

T (s)

R (cm)

R2 (cm2)

T 2 R (s2cm)

1.1

18.59

18.78

18.59

0.933

16.7

278.9

14.53

2.1

18.44

18.25

18.53

0.920

15.7

246.5

13.29

3.1

18.10

18.09

18.15

0.906

14.7

216.1

12.06

4.1

17.88

17.78

17.81

0.891

13.7

187.7

10.88

5.1

17.69

17.50

17.65

0.881

12.7

161.3

9.85

6.1

17.47

17.38

17.28

0.869

11.7

136.9

8.83

7.1

17.06

17.06

17.22

0.856

10.7

114.5

7.83

8.1

17.06

17.00

17.06

0.852

9.7

94.1

7.04

9.1

16.97

16.91

16.96

0.847

8.7

75.7

6.25

10.1

17.00

17.03

17.06

0.852

7.7

59.3

5.58

11.1

17.22

17.37

17.38

0.866

6.7

44.9

5.03

12.1

17.78

17.72

17.75

0.888

5.7

32.5

4.49

13.1

18.57

18.59

18.47

0.927

4.7

22.1

4.04

14.1

19.78

19.90

19.75

0.991

3.7

13.7

3.69

15.1

11.16

11.13

11.13

1.114

2.7

7.3

3.34

16.1

13.25

13.40

13.50

1.338

1.7

2.9

3.04

Notes: at xCM ! R = 15.1, 16.1 cm, times for 10 cycles.

Q2_EXPERIMENT_SOLUTION_FINAL.DOCX

Experimental Competition:
Question 2

14 July 2011
Page 5 of 9

Method (a)

T 2 R(s 2cm)
16
14
12
10
8
6
4
2

R2 (cm2 )
0
0

50

100

150

200

250

300

Calculation from straight line graph: slope = 0.04108 0.0007 s2/cm, y-intercept

= 3.10 0.05 s2cm

g =

4 2
giving g = (961 20) cm/s2

3.10
=
= 75.46 cm2 ( 2.5cm2 )

0.04108
I CM = ( M + m )

= ( 75.46 )( M + m )

From equation (4):

I CM

1 L
L
2

= M + M xCM + m ( z xCM )
3 2
2

Q2_EXPERIMENT_SOLUTION_FINAL.DOCX

Experimental Competition:
Question 2
Then

(75.46)( M + m)
7.38

14 July 2011
Page 6 of 9

= 75.0M + 7.84M + m ( z 17.8)

M
2
+ 75.46 = ( z 17.8)
m

. (14)

The centre of mass position gives:

17.8 ( M + m) = 15.0M + mz
M
z 17.8
=
m
2.8

. (15)

From equations (14) and (15):

7.38
2
( z 17.8) + 75.46 = ( z 17.8)
2.8

( z 17.8)

= 7.47

z = 25.27 = 25.3 0.1 cm

And

M
= 2.68 = 2.7
m
Error Estimation
Find error for g :
From (10),

g=

4 2

g =

i)

g = 16.3 cm/s 2 20 cm/s 2

Find error for z :

First, find error for r =

r = (

3.10
=
= 75.46 cm2 .
0.04108
)r = 2.5 cm 2

"
!L !xcm
" !r
~ 0.03 while
Since error from r contributes most $
,
~ 0.005$ , we estimate error
# r
L xcm
#
propagation from r only to simplify the analysis by substituting the min and max values into
equation (4).

Now, we use rmax = r + r = 75.46 + 2.5 = 77.96 . The corresponding quadratic equation is
6

Q2_EXPERIMENT_SOLUTION_FINAL.DOCX

Experimental Competition:
Question 2

14 July 2011
Page 7 of 9

( z 17.8)

+ 1.743 ( z 17.8) 77.96 = 0 The corresponding solution is ( z 17.8)max = 7.55 cm


If we use rmin = r r = 75.46 2.5 = 72.96 , the corresponding quadratic equation is
2

( z 17.8)

+ 3.529 ( z 17.8) 72.96 = 0


The corresponding solution is ( z 17.8)min = 6.96 cm
7.55 6.96
So ( z 17.8) =
= 0.3 cm
2
( z 17.8)
Note that
~ 0.04 . So, we still ignore the error propagation due to L, xcm
z 17.8
The error z can be estimated from z ( z 17.8) = 0.3 cm
2

ii)

Find error for

M
:
m

M z 17.8
=
m
2.8
M ( z 17.8)
=
= 0.11
2.8
m

We know that

Q2_EXPERIMENT_SOLUTION_FINAL.DOCX

Experimental Competition:
Question 2

14 July 2011
Page 8 of 9

Method (b)
Calculation from T-R plot:
T(s)
1.4

1.3

1.2

1.1

0.9

R(cm)
0.8
0

10

12

14

2
Using the minimum position: T = T min at ICM = ( M + m) Rmin
and g =

16

18

8 2 Rmin
2
Tmin

From graph: Rmin = 8.9 0.2 cm and Tmin = 0.846 0.005 s

g = 982 40 cm/s2

ICM = ( M + m)(8.9) = ( 79.21)( M + m)


2

. (16)
8

Q2_EXPERIMENT_SOLUTION_FINAL.DOCX

Experimental Competition:
Question 2

14 July 2011
Page 9 of 9

From equations (14), (15), (16):

(79.21)( M + m)

= 75.0M + 7.84M + m ( z 17.8)

3.63M + 79.21m = m ( z 17.8)

( x 17.8)

3.63
( x 17.8) 79.21 = 0
2.8

( z 17.8)
And

= 8.28

z = 26.08 = 26.1 0.7 cm


M
= 2.95 = 3.0 0.3
m

Error estimation
i)
Find error for g :

8 2 Rmin
Using the minimum position: g =
, we have
2
Tmin

ii)

R
T
g = min + 2 min
Tmin
Rmin
Find error for z :

g = 34 30 cm/s 2

2
First, find error for r = Rmin
= 79.21 cm 2 .

r = 2 Rmin Rmin = 3.56 cm 2


This r is equivalent to r in part 1. So, one can follow the same error analysis.

i)

As a result, we have
z = 26.08 26.1 cm
z = 0.8 cm
Find error for M m :
Following the same analysis as in part I, we found that
M m = 2.96 ; !(M m) = 0.15
NOTE: This minimum curve method is not as accurate as the usual straight line graph.

The 43rd International Physics Olympiad Theoretical Competition


Tartu, Estonia Tuesday, July 17th 2012
The examination lasts for 5 hours. There are 3 problems
worth a total of 30 points. Please note that the point
values of the three theoretical problems are not equal.
You must not open the envelope with the problems before the sound signal indicating the beginning of competition (three short signals).
You are not allowed to leave your working place
without permission.
If you need any assistance
(broken calculator, need to visit a restroom, etc), please
raise the corresponding ag (help or toilet with a
long handle at your seat) above your seat box walls and
keep it raised until an organizer arrives.
Your answers must be expressed in terms of those
quantities, which are highlighted in the problem text,
and can contain also fundamental constants, if needed.
So, if it is written that the box height is a and the
width - b then a can be used in the answer, and b cannot be used (unless it is highlighted somewhere else, see
below). Those quantities which are highlighted in the
text of a subquestion can be used only in the answer to
that subquestion; the quantities which are highlighted
in the introductory text of the Problem (or a Part of a
Problem), i.e. outside the scope of any subquestion, can
be used for all the answers of that Problem (or of that
Problem Part).
Use only the front side of the sheets of paper.

For each problem, there are dedicated Solution Sheets


(see header for the number and pictogramme). Write
your solutions onto the appropriate Solution Sheets. For
each Problem, the Solution Sheets are numbered; use
the sheets according to the enumeration. Always mark
which Problem Part and Question you are dealing with. Copy the nal answers into the appropriate
boxes of the Answer Sheets. There are also Draft papers; use these for writing things which you dont want
to be graded. If you have written something what you
dont want to be graded onto the Solution Sheets (such
as initial and incorrect solutions), cross these out.
If you need more paper for a certain problem, please raise
the ag help and tell an organizer the problem number; you are given two Solution sheets (you can do this
more than once).
You should use as little text as possible: try to
explain your solution mainly with equations, numbers,
symbols and diagrams.
The rst single sound signal tells you that there are 30
min of solving time left; the second double sound signal
means that 5 min is left; the third triple sound signal
marks the end of solving time. After the third sound
signal you must stop writing immediately. Put all
the papers into the envelope at your desk. You are not
allowed to take any sheet of paper out of the room.
If you have nished solving before the nal sound signal,
please raise your ag.

page 1 of 5

p + gh + 12 v 2 = const., assuming that the velocity v is much


less than the speed of sound. Here is the density, h is the
Part A. Ballistics (4.5 points)
A ball, thrown with an initial speed v0 , moves in a homogen- height, g is free fall acceleration and p is hydrostatic pressure.
eous gravitational eld in the x-z plane, where the x-axis is Streamlines are dened as the trajectories of uid particles
horizontal, and the z-axis is vertical and antiparallel to the (assuming that the ow pattern is stationary). Note that the
term 21 v 2 is called the dynamic pressure.
free fall acceleration g . Neglect the eect of air drag.

Problem T1. Focus on sketches (13 points)

i. (0.8 pts) By adjusting the launching angle for a ball thrown In the g. shown below, a cross-section of an aircraft wing is
with a xed initial speed v0 from the origin, targets can be depicted together with streamlines of the air ow around the
wing, as seen in the wings reference frame. Assume that (a)
hit within the region given by
the air ow is purely two-dimensional (i.e. that the velocity
z z0 kx2 .
vectors of air lie in the plane of the gure); (b) the streamline pattern is independent of the aircraft speed; (c) there is
You can use this fact without proving it. Find the constants
no wind; (d) the dynamic pressure is much smaller than the
z0 and k.
atmospheric pressure, p0 = 1.0 105 Pa .
ii. (1.2 pts) The launching point can now be
You can use a ruler to take measurements from the g. on
freely selected on the ground level z = 0, and
the answer sheet.
the launching angle can be adjusted as needed.
The aim is to hit the topmost point of a spherical building of radius R (see g.) with the
minimal initial speed v0 . Bouncing o the roof prior to hitting
the target is not allowed. Sketch qualitatively the shape of
the optimal trajectory of the ball (use the designated box on
the answer sheet). Note that the marks are given only for the
sketch.
iii. (2.5 pts) What is the minimal launching speed vmin needed
to hit the topmost point of a spherical building of radius R ?
i. (0.8 pts) If the aircrafts ground speed is v0 = 100 m/s ,
what is the speed of the air, vP , at the point P (marked in the
g.) with respect to the ground?
ii. (1.2 pts) In the case of high relative humidity, as the
ground speed of the aircraft increases over a critical value vcrit ,
a stream of water droplets is created behind the wing. The
droplets emerge at a certain point Q. Mark the point Q in the
g. on the answer sheet. Explain qualitatively (using formulae
and as little text as possible) how you determined the position
of Q.
iii. (2.0 pts) Estimate the critical speed vcrit using the following data: relative humidity of the air is r = 90% , specic heat
capacity of air at constant pressure cp = 1.00 103 J/kg K ,
pressure of saturated water vapour:

psa = 2.31 kPa

at

the temperature of the unperturbed air Ta = 293 K and


psb = 2.46 kPa at Tb = 294 K . Depending on your approximations, you may also need the specic heat capacity of air at
constant volume cV = 0.717 103 J/kg K . Note that the relLa Geode, Parc de la Villette, Paris. Photo: katchooo/ickr.com
ative humidity is dened as the ratio of the vapour pressure to
the saturated vapour pressure at the given temperature. SatPart B. Air ow around a wing (4 points)
urated vapour pressure is dened as the vapour pressure by
For this part of the problem, the following information may be which vapour is in equilibrium with the liquid.
useful. For a ow of liquid or gas in a tube along a streamline,
page 2 of 5

Part C. Magnetic straws (4.5 points)

There is a magnetic ux through the central cross-section


2
2
2
Consider a cylindrical tube made of a superconducting mater- of the tube, z = 0, x + y < r . A superconductor is a material. The length of the tube is l and the inner radius is r ial which expels any magnetic eld (the eld is zero inside the
with l r. The centre of the tube coincides with the origin, material).
and its axis coincides with the z-axis.
i. (0.8 pts) Sketch ve such magnetic eld lines, which pass
through the ve red dots marked on the axial cross-section of
the tube, on the designated diagram on the answer sheet.
ii. (1.2 pts) Find the tension force T along the z-axis in the
middle of the tube (i.e. the force by which two halves of the
tube, z > 0 and z < 0, interact with each other).
iii. (2.5 pts) Consider another tube, identical and parallel to
the rst one.

The second tube has the same magnetic eld but in the opposite direction and its centre is placed at y = l , x = z = 0 (so
that the tubes form opposite sides of a square). Determine the
magnetic interaction force F between the two tubes.

page 3 of 5

Problem T2. Kelvin water dropper (8 points)


The following facts about the surface tension may turn out
to be useful for this problem. For the molecules of a liquid,
the positions at the liquid-air interface are less favourable as
compared with the positions in the bulk of the liquid. This
interface is described by the so-called surface energy, U = S,
where S is the surface area of the interface and is the surface
tension coecient of the liquid. Moreover, two fragments of
the liquid surface pull each other with a force F = l, where l
is the length of a straight line separating the fragments.

Part B. Two pipes (4 points)


An apparatus called the Kelvin water dropper consists of
two pipes, each identical to the one described in Part A, connected via a T-junction, see g. The ends of both pipes are
at the centres of two cylindrical electrodes (with height L and
diameter D with L D r). For both tubes, the dripping
rate is n droplets per unit time. Droplets fall from height H
into conductive bowls underneath the nozzles, cross-connected
to the electrodes as shown in the diagram. The electrodes are
connected via a capacitance C . There is no net charge on
the system of bowls and electrodes. Note that the top water
container is earthed as shown. The rst droplet to fall will
have some microscopic charge which will cause an imbalance
between the two sides and a small charge separation across the
capacitor.

A long metallic pipe with internal diameter d is pointing directly downwards. Water is slowly dripping from a nozzle at its
lower end, see g. Water can be considered to be electrically
conducting; its surface tension is and its density is . A
droplet of radius r hangs below the nozzle. The radius grows
slowly in time until the droplet separates from the nozzle due
to the free fall acceleration g . Always assume that d r.
i. (1.2 pts) Express the absolute value of the charge Q0 of the
Part A. Single pipe (4 points)
drops as they separate from the tubes, and at the instant when
i. (1.2 pts) Find the radius rmax of a drop just before it septhe capacitors charge is q . Express Q0 in terms of rmax
arates from the nozzle.
(from Part A-i) and neglect the eect described in Part A-iii.
ii. (1.2 pts) Relative to the far-away surroundings, the pipes
electrostatic potential is . Find the charge Q of a drop when ii. (1.5 pts) Find the dependence of q on time t by approximating it with a continuous function q(t) and assuming that
its radius is r .
q(0) = q0 .
iii. (1.6 pts) Consider the situation in which r is kept constant and is slowly increased. The droplet becomes unstable
and breaks into pieces if the hydrostatic pressure inside the
droplet becomes smaller than the atmospheric pressure. Find
the critical potential max at which this will happen.

iii. (1.3 pts) The droppers functioning can be hindered by


the eect shown in Part A-iii. In addition, a limit Umax to
the achievable potential between the electrodes is set by the
electrostatic push between a droplet and the bowl beneath it.
Find Umax .

page 4 of 5

Problem T3. Protostar formation (9 points)


Let us model the formation of a star as follows. A spherical
cloud of sparse interstellar gas, initially at rest, starts to collapse due to its own gravity. The initial radius of the ball is
r0 and the mass is m . The temperature of the surroundings
(much sparser than the gas) and the initial temperature of the
gas is uniformly T0 . The gas may be assumed to be ideal.
The average molar mass of the gas is and its adiabatic
index is >

4
3.

iii. (2.5 pts) Assuming that the pressure remains negligible,


nd the time tr0 needed for the ball to collapse from r0 down
to a much smaller radius, using Keplers Laws.
iv. (1.7 pts) At some radius r3 r0 , the gas becomes dense
enough to be opaque to the heat radiation. Calculate the
amount of heat Q radiated away during the collapse from the
radius r0 down to r3 .

Assume that G m
r0 RT0 , where R is the

v. (1 pt) For radii smaller than r3 you may neglect heat loss
due to radiation. Determine how the temperature T of the ball
i. (0.8 pts) During much of the collapse, the gas is so transpar- depends on its radius for r < r3 .
ent that any heat generated is immediately radiated away, i.e.
the ball stays in thermodynamic equilibrium with its surround- vi. (2 pts) Eventually we cannot neglect the eect of the presings. What is the number of times, n, by which the pressure sure on the dynamics of the gas and the collapse stops at r = r4
increases when the radius is halved to r1 = 0.5r0 ? Assume (with r4 r3 ). However, the radiation loss can still be negthat the gas density remains uniform.
lected and the temperature is not yet high enough to ignite
nuclear fusion. The pressure of such a protostar is not uniform
ii. (1 pt) Estimate the time t2 needed for the radius to shrink anymore, but rough estimates with inaccurate numerical prefrom r0 to r2 = 0.95r0 . Neglect the change of the gravity eld factors can still be done. Estimate the nal radius r4 and the
at the position of a falling gas particle.
respective temperature T4 .
gas constant and G is the gravitational constant.

page 5 of 5

Problem T1. Focus on sketches (13 points)


Part A. Ballistics (4.5 points)
i. (0.8 pts) When the stone is thrown vertically upwards, it
can reach the point x = 0, z = v02 /2g (as it follows from the
energy conservation law). Comparing this with the inequality
z z0 kx2 we conclude that
z0 = v02 /2g.

[0.3 pts]

Note that the interior of the building needs to lie inside the
region where the targets can be hit with a stone thrown from
the top with initial speed vmin . Indeed, if we can throw over
the building, we can hit anything inside by lowering the throwing angle. On the other hand, the boundary of the targetable
region needs to touch the building. Indeed, if there were a
gap, it would be possible to hit a target just above the point
where the optimal trajectory touches the building; the trajectory through that target wouldnt touch the building anywhere,
hence we arrive at a contradiction.
So, with v0 corresponding to the optimal trajectory, the targetable region touches the building; due to symmetry, overall
there are two touching points (for smaller speeds, there would
be four, and for larger speeds, there would be none). With the
origin at the top of the building, the intersection points are
defined by the following system of equations:

Let us consider the asymptotics z ; the trajectory of


the stone is a parabola, and at this limit, the horizontal displacement (for the given z) is very sensitive with respect to the
curvature of the parabola: the flatter the parabola, the larger
the displacement. The parabola has the flattest shape when
the stone is thrown horizontally, x = v0 t and z = gt2 /2, i.e.
its trajectory is given by z = gx2 /2v02 . Now, let us recall
that z z0 kx2 , i.e. gx2 /2v02 z0 kx2 k g/2v02 .
v2
gx2
Note that k < g/2v02 would imply that there is a gap between
x2 + z 2 + 2zR = 0, z = 0 2 .
2g
2v0
the parabolic region z z0 kx2 and the given trajectory
2
2
z = gx /2v0 . This trajectory is supposed to be optimal for Upon eliminating z, this becomes a biquadratic equation for x:
2

  2
 2

hitting targets far below (z ), so there should be no such
1 gR
v0
v0
g
2
4
2
+
+
x

+
R
= 0.
x
a gap, and hence, we can exclude the option k < g/2v0 . This
2
2
2v0
2
v0
4g
g
leaves us with
Hence the speed by which the real-valued solutions disappear
k = g/2v02 .
[0.5 pts]
can be found from the condition that the discriminant vanishes:
2

ii. (1.2 pts) Let us note that the
1 gR
gR
1 gR
= + 2 = 2 = 2.
2
stone trajectory is reversible and due
2
v0
4
v0
v0
to the energy conservation law, one
Bearing in mind that due to the energy conservation law, at
can equivalently ask, what is the minthe ground level the squared speed is increased by 4gR. Thus
imal initial speed needed for a stone
we finally obtain
to be thrown from the topmost point
r
q
gR
of the spherical building down to the
vmin = v02 + 4gR = 3
.
2
ground without hitting the roof, and what is the respective trajectory. It is easy to understand that the trajectory either needs Part B. Mist (4 points)
to touch the roof, or start horizontally from the topmost point i. (0.8 pts) In the planes reference frame, along the channel
with the curvature radius equal to R. Indeed, if neither were between two streamlines the volume flux of air (volume flow
the case, it would be possible to keep the same throwing angle rate) is constant due to continuity. The volume flux is the
and just reduce the speed a little bit the stone would still product of speed and channels cross-section area, which, due
reach the ground without hitting the roof. Further, if it were to the two-dimensional geometry, is proportional to the channel
tangent at the topmost point, the trajectory wouldnt touch width and can be measured from the Fig. Due to the absence of
nor intersect the roof anywhere else, because the curvature of wind, the unperturbed airs speed in the planes frame is just v0 .
the parabola has maximum at its topmost point. Then, it So, upon measuring the dimensions a = 10 mm and b = 13 mm
would be possible to keep the initial speed constant, and in- (see Fig), we can write v0 a = ub and hence u = v0 ab . Since at
crease slightly the throwing angle (from horizontal to slightly point P , the streamlines are horizontal where all the velocities
upwards): the new trajectory wouldnt be neither tangent at are parallel, the vector addition is reduced to the scalar addithe top nor touch the roof at any other point; now we can re- tion: the airs ground speed vP = v0 u = v0 (1 ab ) = 23 m/s.
duce the initial speed as we argued previously. So we conclude ii. (1.2 pts) Although the dynamic pressure 21 v 2 is relatively
that the optimal trajectory needs to touch the roof somewhere, small, it gives rise to some adiabatic expansion and compresas shown in Fig.
sion. In expanding regions the temperature will drop and hence,
iii. (2.5 pts) The brute force approach would be writing down the pressure of saturated vapours will also drop. If the dew
the condition that the optimal trajectory intersects with the point is reached, a stream of droplets will appear. This process
building at two points and touches at one. This would be de- will start in a point where the adiabatic expansion is maximal,
scribed by a fourth order algebraic equation and therefore, it is i.e. where the hydrostatic pressure is minimal and consequently,
not realistic to accomplish such a solution within a reasonable as it follows from the Bernoullis law p + 12 v 2 = const, the dytime frame.
namic pressure is maximal: in the place where the air speed in
page 1 of 5

wings frame is maximal and the streamline distance minimal. Part C. Magnetic straws (4.5 points)
i. (0.8 pts) Due to the superconductSuch a point Q is marked in Fig.
ing walls, the magnetic field lines cannot
cross the walls, so the flux is constant
along the tube. For a closed contour inside the tube, there should be no circulation of the magnetic field, hence the
field lines cannot be curved, and the field
needs to be homogeneous. The field lines
close from outside the tube, similarly to a solenoid.
ii. (1.2 pts) Let us consider the change of the magnetic energy
when
the tube is stretched (virtually) by a small amount l.
iii. (2 pts) First we need to calculate the dew point for the air
of given water content (since the relative pressure change will Note that the magnetic flux trough the tube is conserved: any
d
be small, we can ignore the dependence of the dew point on change of flux would imply a non-zero electromotive force dt ,
resistivity, an infinite current. So, the inducpressure). The water vapour pressure is pw = psa r = 2.08 kPa. and for a zero

B2
.
tion
B
=
.
The
energy density of the magnetic field is 2
r 2
0
The relative change of the pressure of the saturated vapour is
Thus, the change of the magnetic energy is calculated as
small, so we can linearize its temperature dependence:
psa pw
psb psa
(1 r)psa
=
= Ta T = (Tb Ta )
;
Ta T
Tb Ta
psb psa
numerically T 291.5 K. Further we need to relate the air
speed to the temperature. To this end we need to use the energy conservation law. A convenient ready-to-use form of it is
provided by the Bernoullis law. Applying this law will give
a good approximation of the reality, but strictly speaking, it
needs to be modified to take into account the compressibility
of air and the associated expansion/contraction work. Consider one mole of air, which has the mass and the volume
V = RT /p. Apparently the process is fast and the air parcels are large, so that heat transfer across the air parcels is
negligible. Additionally, the process is subsonic; all together
we can conclude that the process is adiabatic. Consider a segment of a tube formed by the streamlines. Let us denote the
physical quantities at its one end by index 1, and at the other
end by index 2. Then, while one mole of gas flows into
the tube at one end, as much flows out at the other end. The
inflow carries in kinetic energy 21 v12 , and the outflow carries
out 12 v22 . The inflowing gas receives work due to the pushing
gas equal to p1 V1 = RT1 , the outflowing gas performs work
p2 V2 = RT2 . Lets define molar heat capacities CV = cV and
Cp = cp . The inflow carries in heat energy CV RT1 , and the
outflow carries out CV RT2 . All together, the energy balance
can be written as 12 v 2 + Cp T = const. From this we can
2
2
2
easily express v2 = C1 vcrit
( ac2 1) = cp T , where c is the
streamline distance at the point Q, and further
r
2cp T
vcrit = c
23 m/s,
a2 c2
where we have used c 4.5 mm and T = 1.5 K. Note that
in reality, the required speed is probably somewhat higher, because for a fast condensation, a considerable over-saturation is
needed. However, within an order of magnitude, this estimate
remains valid.

W =

B2 2
2
r l =
l.
20
20 r2

This energy increase is achieved owing to the work done by the


stretching force, W = T l. Hence, the force
T =

2
.
20 r2

iii. (2.5 pts) Let us analyse, what would be the change of


the magnetic energy when one of the straws is displaced to a
small distance. The magnetic field inside the tubes will remain
constant due to the conservation of magnetic flux, but outside,
the magnetic field will be changed. The magnetic field outside the straws is defined by the following condition: there is
~ (because there are no currents outside the
no circulation of B
straws); there are no sources of the field lines, other than the
endpoints of the straws; each of the endpoints of the straws is
a source of streamlines with a fixed magnetic flux . These
are exactly the same condition as those which define the electric field of four charges Q. We know that if the distance
between charges is much larger than the geometrical size of
a charge, the charges can be considered as point charges (the
electric field near the charges remains almost constant, so that
the respective contribution to the change of the overall electric
field energy is negligible). Therefore we can conclude that the
endpoints of the straws can be considered as magnetic point
charges. In order to calculate the force between two magnetic
charges (magnetic monopoles), we need to establish the correspondence between magnetic and electric quantities.
For two electric charges Q separated by a distance a, the
2
1 Q
force is F = 4
2 , and at the position of one charge, the elec0 a
2

tric field of the other charge has energy density w = 3212 0 Q


a4 ;
2
hence we can write F = 8wa . This is a universal expression
for the force (for the case when the field lines have the same
shape as in the case of two opposite and equal by modulus electric charges) relying only on the energy density, and not related
to the nature of the field; so we can apply it to the magnetic

page 2 of 5

field. Indeed, the force can be calculated as a derivative of


the full field energy with respect to a virtual displacement of
a field line source (electric or magnetic charge); if the energy
densities of two fields are respectively equal at one point, they
are equal everywhere, and so are equal the full field energies.
As it follows from the Gauss law, for a point source of a fixed
1
magnetic flux at a distance a, the induction B = 4
a2 . So,
1
2
B2
the energy density w = 20 = 322 0 a4 , hence

For the two straws, we have four magnetic charges. The longitudinal (along a straw axis) forces cancel out (the diagonally
positioned pairs of same-sign-charges push in opposite directions). The normal force is a superposition of the attraction
1 2
due to the two pairs of opposite charges, F1 = 4
2 , and
0 l
the repulsive forces of diagonal pairs, F2 =
attractive force will be

2 2
80 2l2 .

4 2 2
.
F = 2(F1 F2 ) =
80 l2

1 2
F =
.
40 a2

page 3 of 5

The net

Problem T2. Kelvin water dropper (8 points)

where the droplets capacitance Cd = 40 r; the electrical work


dWel = max dq = 40 2max dr. Putting dW = 0 we obtain an
equation for max , which recovers the earlier result.
Part B. Two pipes (4 points)
i. (1.2 pts) This is basically the same as Part A-ii, except
that the surroundings potential is that of the surrounding
electrode, U/2 (where U = q/C is the capacitors voltage)
and droplet has the ground potential (0). As it is not defined
which electrode is the positive one, opposite sign of the potential may be chosen, if done consistently. Note that since
the cylindrical electrode is long, it shields effectively the environments (ground, wall, etc) potential. So, relative to its
surroundings, the droplets potential is U/2. Using the result
ii. (1.2 pts) Since d r, we can neglect the change of the of Part A we obtain
droplets capacitance due to the tube. On the one hand, the
1 Q
Q = 20 U rmax = 20 qrmax /C.
droplets potential is ; on the other hand, it is 4
. So,
0 r

Part A. Single pipe (4 points)


i. (1.2 pts) Let us write the force balance for the droplet.
Since d r, we can neglect the force 4 pd2 due to the excess
3
pressure p inside the tube. So, the gravity force 43 rmax
g
is balanced by the capillary force. When the droplet separates
from the tube, the water surface forms in the vicinity of the
nozzle a neck, which has vertical tangent. In the horizontal
cross-section of that neck, the capillary force is vertical and
can be calculated as d. So,
s
3d
.
rmax = 3
4g

Q = 40 r.

ii. (1.5 pts) The sign of the droplets charge is the same as
iii. (1.6 pts) Excess pressure inside the droplet is caused by that of the capacitors opposite plate (which is connected to
the capillary pressure 2/r (increases the inside pressure), and the farther electrode). So, when the droplet falls into the bowl,
by the electrostatic pressure 21 0 E 2 = 21 0 2 /r2 (decreases the it will increase the capacitors charge by Q:
pressure). So, the sign of the excess pressure will change, if
q
dq = 20 U rmax dN = 20 rmax ndt ,
1
2
2

/r
=
2/r,
hence
C
2 0 max
p
where dN = ndt is the number of droplets which fall during
max = 2 r/0 .
the time dt This is a simple linear differential equation which
The expression for the electrostatic pressure used above can
is solved easily to obtain
be derived as follows. The electrostatic force acting on a surface
s

charge of density and surface area S is given by F = S E,

n
2
r
n
0
0
max
3 6d
t
is the field at the site without the field created by the
=
.
q = q0 e , =
where E
C
C
g
surface charge element itself. Note that this force is perpendicular to the surface, so F/S can be interpreted as a pressure. iii. (1.3 pts) The droplets can reach the bowls if their mechThe surface charge gives rise to a field drop on the surface equal anical energy mgH (where m is the droplets mass) is large
to E = /0 (which follows from the Gauss law); inside the enough to overcome the electrostatic push: The droplet starts
droplet, there is no field due to the conductivity of the droplet: at the point where the electric potential is 0, which is the sum of
1 E = 0; outside the droplet, there is field E = E
+ 1 E,
E
2
2
the potential U/2, due to the electrode, and of its self-generated
1
1
= E = E. Bringing everything together, we
therefore E
2
2
potential U/2. Its motion is not affected by the self-generated
obtain the expression used above.
field, so it needs to fall from the potential U/2 down to the poNote that alternatively, this expression can be derived by tential U/2, resulting in the change of the electrostatic energy
considering a virtual displacement of a capacitors surface and equal to U Q mgH, where Q = 2 U r
0
max (see above). So,
comparing the pressure work pV with the change of the elecmgH
trostatic field energy 12 0 E 2 V .
Umax =
,
20 Umax rmax
Finally, the answer to the question can be also derived from
the requirement that the mechanical work dA done for an ins
r
Hd
H 3 g 2 d2
finitesimal droplet inflation needs to be zero. From the en.
Umax =
= 6
1 2
2
ergy conservation law, dW + dWel = d(4r ) + 2 max dCd ,
20 rmax
630

page 4 of 5

Problem T3. Protostar formation (9 points)


i. (0.8 pts)
T = const = pV = const
V r3
p r3 =

p(r1 )
= 23 = 8.
p(r0 )

ii. (1 pt) During the period considered the pressure is negligible. Therefore the gas is in free fall. By Gauss theorem and
symmetry, the gravitational field at any point in the ball is
equivalent to the one generated when all the mass closer to the
center is compressed into the center. Moreover, while the ball
has not yet shrunk much, the field strength on its surface does
not change much either. The acceleration of the outermost
layer stays approximately constant. Thus,
s
2(r0 r2 )
t
g
where

q
2E + 2Gm
= and
be sketched as follows. Substituting
r

2E = , one gets
Z
d
t
=
2
2
4Gm
( 2 )
0


Z
1

1
1
= 3
d.
+
+
+
4 0
( )2
( + )2

+
R
Here (after shifting the variable) one can use d
= ln and
R d
1
2 = , finally getting the same answer as by Keplers laws.
iv. (1.7 pts) By ClapeyronMendeleyev law,
p=

mRT0
.
V

Work done by gravity to compress the ball is


W =

p dV =

mRT0

3
4
3 r3
4
3
3 r0

dV
3mRT0 r0
=
ln .
V

r3

Gm
,
r2
r
r0
2r02 (r0 r2 )
0.1r03
=
.
t
Gm
Gm

The temperature stays constant, so the internal energy does not


change; hence, according to the 1st law of thermodynamics, the
compression work W is the heat radiated.
v. (1 pt) The collapse continues adiabatically.

iii. (2.5 pts) Gravitationally the outer layer of the ball is influenced by the rest just as the rest were compressed into a
point mass. Therefore we have Keplerian motion: the fall of
any part of the outer layer consists in a halfperiod of an ultraelliptical orbit. The ellipse is degenerate into a line; its foci are
at the ends of the line; one focus is at the center of the ball (by
Keplers 1st law) and the other one is at r0 , see figure (instead
of a degenerate ellipse, a strongly elliptical ellipse is depicted).
The period of the orbit is determined by the longer semiaxis of
the ellipse (by Keplers 3rd law). The longer semiaxis is r0 /2
and we are interested in half a period. Thus, the answer is
equal to the halfperiod of a circular orbit of radius r0 /2:
r

2
2
Gm
r0
r03
=
t
=

=
.
r0
2tr0
2
(r0 /2)2
8Gm

pV = const = T V 1 = const.

T V 1 r33
 r 33
3
.
T = T0
r

vi. (2 pts) During the collapse, the gravitational energy is converted into heat. Since r3 r4 , The released gravitational energy can be estimated as = Gm2 (r41 r31 ) Gm2 /r4
(exact calculation by integration adds a prefactor 35 ); the terminal heat energy is estimated as Q = cV m
(T4 T0 )
m
cV T4 (the approximation T4 T0 follows from the result
of the previous question, when combined with r3 r4 ). So,
R m
m
Q = 1
T4 RT4 . For the temperature T4 , we can use
 33
the result of the previous question, T4 = T0 rr34
. Since
initial full energy was approximately zero, Q + 0, we
obtain
m
Gm2
RT0
r4

r3
r4

33

= r4 r3

RT0 r3
mG

1
 34

Alternatively, one may write the energy conservation law Therefore,


 33

Gm
r 2
= E (that in turn is obtainable from Newtons
RT0 r3 43
2 r
.
T4 T0
Gm
II law r = Gm
mG
r 2 ) with E = r0 , separate the variables
q
R
2Gm
dr 2Gm . Alternatively, one can obtain the result by approximately
( dr
dt = 2E + r ) and write the integral t =
2E+ r
Gm
This integral is probably not calculable during the limitted equating the hydrostatic pressure r4 r42 to the gas pressure
time given during the Olympiad, but a possible approach can p4 = RT4 ; the result will be exactly the same as given above.

page 5 of 5

Problem T1. Focus on sketches (13 points)


Part A. Ballistics (4.5 points)
i. (0.8 pts)
z0 = v02 /2g

k = g/2v02

ii. (1.2 pts) The sketch of the trajectory:

iii. (2.5 pts)


vmin = 3

gR
2

page 1 of 5

Part B. Air flow around a wing (4 points)


i. (0.8 pts)
vP = 23 m/s

ii. (1.2 pts) Mark on this fig. the point Q. Use it also for taking measurements (questions i and iii).

Formulae motivating
the choice of point Q:

av = const
1
p + v 2 = const
2
p1 T = const

iii. (2.0 pts)


Formula: vcrit = c

2cp T
a 2 c2

Numerical: vcrit 23 m/s

page 2 of 5

Part C. Magnetic straws (4.5 points)


i. (0.8 pts)
Sketch here five
magnetic field lines.

ii. (1.2 pts)


T =

2
20 r2

iii. (2.5 pts)

4 2 2
F =
80 l2

page 3 of 5

Problem T2. Kelvin water dropper (8 points)


Part A. Single pipe (4 points)
i. (1.2 pts)
s
3d
rmax = 3
4g

ii. (1.2 pts)


Q = 40 r

iii. (1.6 pts)


p
max = 2 r/0

Part B. Two pipes (4 points)


i. (1.2 pts)
Q0 = 20 qrmax /C

ii. (1.5 pts)


0 n
q(t) = q0 et , =
C

s
3

6d
.
g

iii. (1.3 pts)


Umax =

s
6

H 3 g 2 d2
630

page 4 of 5

Problem T3. Protostar formation (9 points)


i. (0.8 pts)
n=8

ii. (1 pt)
t2

0.1r03
Gm

iii. (2.5 pts)


tr0 =

r03
8Gm

iv. (1.7 pts)


Q=

3mRT0 r0
ln

r3

v. (1 pt)
T (r) = T0

 r 33
3

vi. (2 pts)
r4 r3

RT0 r3
mG

1
 34

T4 T0

RT0 r3
mG

33
 43

page 5 of 5

The 43rd International Physics Olympiad Experimental Competition


Tartu, Estonia Thursday, July 19th 2012
The examination lasts for 5 hours. There are 2 problems
worth in total 20 points. There are two tables in your
disposal (in two neighbouring cubicles), the apparatus of
Problem E1 is on one table and the apparatus of Problem E2 is on the other table; you can move freely between
these tables. However, you are not allowed to move
any piece of experimental setup from one table to
the other.
Initially the experimental equipment on one table is
covered and on the other table is boxed. You must
neither remove the cover nor open the box nor
open the envelope with the problems before the
sound signal of the beginning of competition
(three short signals).
You are not allowed to leave your working place
without permission. If you need any assistance (malfunctioning equipment, broken calculator, need to
visit a restroom, etc), please raise the corresponding ag
(help or toilet with a long handle at your seat)
above your seat box walls and keep it raised until an organizer arrives.
Use only the front side of the sheets of paper.
For each problem, there are dedicated Solution Sheets
(see header for the number and pictogramme). Write
your solutions onto the appropriate Solution Sheets. For
each Problem, the Solution Sheets are numbered; use the
sheets according to the enumeration. Copy the nal answers into the appropriate boxes of the Answer Sheets.

There are also Draft papers; use these for writing things
which you dont want to be graded. If you have written
something what you dont want to be graded onto the
Solution Sheets (such as initial and incorrect solutions),
cross these out.
If you need more paper for a certain problem, please raise
the ag help and tell an organizer the problem number; you are given two Solution sheets (you can do this
more than once).
You should use as little text as possible: try to
explain your solution mainly with equations, numbers,
tables, symbols and diagrams.
Avoid unnecessary movements during the experimental
examination and do not shake the walls of your cubicle the laser experiment requires stability.
Do not look into the laser beam or its reections! It may
permanently damage your eyes!
The rst single sound signal tells you that there are 30
min of solving time left; the second double sound signal
means that 5 min is left; the third triple sound signal
marks the end of solving time. After the third sound
signal you must stop writing immediately. Put all
the papers into the envelope at your desk. You are not
allowed to take any sheet of paper out of the room.
If you have nished solving before the nal sound signal,
please raise your ag.

page 1 of 5

Problem E1. The magnetic permeability of water The remaining legend for the gure is as follows: 6 the point
(10 points)
where the laser beam hits the screen; 11 the LCD screen of
The eect of a magnetic eld on most of substances other
than ferromagnetics is rather weak. This is because the energy
density of the magnetic eld in substances of relative magnetic
B2
permeability is given by the formula w = 2
, and typically
0
is very close to 1. Still, with suitable experimental techniques
such eects are rmly observable. In this problem we study the
eect of a magnetic eld, created by a permanent neodymium
magnet, on water and use the results to calculate the magnetic
permeability of water. You are not asked to estimate any
uncertainties throughout this problem and you do not
need to take into account the eects of surface tension.
The setup comprises of 1 a stand (the highlighted numbers
correspond to the numbers in the g.), 3 a digital caliper,
4 a laser pointer, 5 a water tray and 7 a cylindrical
permanent magnet in the water tray (the magnet is axially
magnetised). The water tray is xed to the base of the stand
by the magnets pull. The laser is xed to the caliper, the base
of which is fastened to the stand; the caliper allows horizontal
displacement of the laser. The on-o button of the laser can be
kept down with the help of 13 the white conical tube. Do not
leave the Laser switched on unnecessarily. The depth of the
water above the magnet should be reasonably close to 1 mm (if
shallower, the water surface becomes so curved that it will be
dicult to take readings from the screen). 15 A cup of water
and 16 a syringe can be used for the water level adjustment
(to raise the level by 1 mm, add 13 ml of water). 2 A sheet of
graph paper (the screen) is to be xed to the vertical plate
with 14 small magnetic tablets. If the laser spot on the screen
becomes smeared, check for a dust on the water surface (and
blow away).

the caliper, 10 the button which switches the caliper units


between millimeters and inches; 8 on-o switch; 9 button
for setting the origin of the caliper reading. Beneath the laser
pointer, there is one more button on the caliper, which temporarily re-sets the origin (if you pushed it inadvertently, push
it once again to return to the normal measuring mode).
Numerical values for your calculations:
Horizontal distance between the magnets centre and the
screen L0 = 490 mm. Check (and adjust, if needed) the alignment of the centre of the magnet in two perpendicular directions. The vertical axis of the magnet must intersect with the
laser beam, and it must also intersect with 12 the black line
on the support plate.
Magnetic induction (magnetic eld strength) on the
magnets axis, at a height of 1 mm from the at surface,
B0 = 0.50 T
Density of water w = 1000 kg/m3
Acceleration of free-fall g = 9.8 m/s2
Permeability of a vacuum 0 = 4 107 H/m
WARNINGS:
The laser orientation is pre-adjusted, do not move it!
Do not look into the laser beam or its reections!
Do not try to remove the strong neodymium magnet!
Do not put magnetic materials close to the magnet!
Turn o the laser when not used, batteries drain in 1 h!

page 2 of 5

Part A. Qualitative shape of the water surface (1 points)


iv. (1.4 pts) please note that the slope (tan ) of the water
When a cylindrical magnet is placed below water surface, the surface can be expressed as follows:
latter becomes curved. By observation, determine the shape
of the water surface above the magnet. Based on this observacos2 0 y y0 (x x0 ) tan 0
tion, decide if the water is diamagnetic ( < 1) or paramagnetic
tan

,
2
L0 + x x0
( > 1).
where y0 is the vertical position of the laser spot on the screen
when the beam is reected from the water surface at the axis
of the magnet, and x0 is the respective position of the caliper.
Calculate the values of the slope of the water surface and enter
them into the Table on the Answer Sheet. Please note that it
may be possible to simplify your calculations if you substitute
Write the letter corresponding to the correct option into the some combination of terms in the given expression for the slope
Answer Sheet, together with an inequality > 1 or < 1.
with a reading from the last graph.
For this part, you do not need to justify your answer.

v. (1.6 pts) Calculate the height of the water surface relative


to the surface far from the magnet, as a function of x, and
write it into the Table on the Answer Sheet.

Part B. Exact shape of the water surface (7 points)


Curving of the water surface can be checked with high sensitivity by measuring the reection of the laser beam from the vi. (1.0 pts) Draw the graph of the latter dependence. Insurface. We use this eect to calculate the dependence of the dicate on it the region where the beam hits the water surface
depth of the water on the horizontal position above the magnet. directly above the magnet.
i. (1.6 pts) Measure the dependence of the vertical position
y of the laser spot on the screen on the caliper reading x (see
gure). You should cover the whole usable range of caliper
displacements. Write the results into the Table in the Answer
Sheet.

Part C. Magnetic permeability (2 points)


Using the results of Part B, calculate the value of 1 (the
so-called magnetic susceptibility), where is the relative magnetic permeability of the water. Write your nal formula and
the numerical result into the Answer Sheet.

ii. (0.7 pts) Draw the graph of the measured dependence.


iii. (0.7 pts) Using the obtained graph, determine the angle
0 between the beam and the horizontal surface of the water.

page 3 of 5

Problem E2. Nonlinear Black Box (10 points)


In simple problems, electrical circuits are assumed to consist of linear elements, for which electrical characteristics are
directly proportional to each other. Examples include resistance (V = RI), capacitance (Q = CV ) and inductance
(V = LI = L dI
dt ), where R, C and L are constants. In this
problem, however, we examine a circuit containing nonlinear
elements, enclosed in a black box, for which the assumption of
proportionality no longer holds.
The setup
comprises a multimeter (labelled IPhOmeasure), a black box that acts as a current source, a black
box containing nonlinear elements, and four test leads with
stackable connectors for wiring. Be careful not to break the
seal on the black box.
The multimeter can measure current and voltage simultaneously. You can store with it up to 2000 data points, each
consisting of: voltage V , current I, power P = IV , resistance
R = V /I, voltage time derivative V (= dV
dt ), current time derivative I (= dI
)
and
time
t.
See
multimeter
manual for details.
dt
If you go beyond 2000 stored data points, the oldest data will
be overwritten.
IN

OUT

GND

ours on the black box and the current source are connected (you are allowed to use negative voltages).

C(V)

Nonlinear
device

Black box
It is safe to discharge the capacitor in the black box by shorting
its inputs, either by itself or through the IN and OUT terminals on multimeter: the internal resistance of this capacitor is
enough to keep the current from damaging anything.
You are not asked to estimate any uncertainties
throughout this problem.
Part A. Circuit without inductance (7 points)
In this part, keep the switch on the black box closed (push I
down), so that the inductance is shorted. Please note that
some measurements may take a considerable time, therefore it
is recommended that you read through all the tasks of part A
to avoid unnecessary work.

i. (1.0 pts) Conrm that the output current of the current


source is approximately 6 mA, and determine the range within
which it varies for voltages between 0 and +480 mV. Document
The constant current source supplies stable current as long as the circuit diagram used.
the voltage across its terminals stays between 0.6125 V and
0.6125 V. When switched o, the constant current source be- ii. (1.2 pts) Show that the dierential capacitance C(V ) used
in the black box is approximately 2 F by measuring its value
haves as a large (essentially innite) resistance.
for a single voltage of your choice C(V0 ) = C0 . Document the
circuit diagram.
-

Multimeter

+
I=6mA
U=-612.5mV...612.5mV

iii. (2.2 pts) Neglecting the nonlinearity of the capacitance


[C(V ) C0 ], determine the currentvoltage characteristic of
the nonlinear element used in the black box. Plot the I(V )
curve for obtainable positive voltages on the black box onto
the answer sheet. Document the circuit diagram.

iv. (2.6 pts) Using measurements taken from the whole range
of obtainable voltages, calculate and plot the C(V ) curve for
The black box contains an electric double layer capacitor obtainable positive voltages from the black box on the answer
(which is a slightly nonlinear high capacitance capacitor), an sheet. Write down the minimal and maximal values of dierunknown nonlinear element, and an inductor L = 10 H of ential capacitance Cmin , Cmax . Document the circuit diagram.
negligible resistance, switchable as indicated on the circuit diagram. The nonlinear element can be considered as a resistance Part B. Circuit with inductance (3 points)
with a nonlinear dependence between the voltage and the cur- Enable the inductance by opening the switch on the black box
rent [I(V ) is a continuous function of V with I(0) = 0]. Like- (push 0 down). Using the same method as in pt. A-iii, measwise, for the capacitor, the dierential capacitance C(V ) = ure and plot the current-voltage characteristic of the nonlinear element. Describe any signicant dierences between the
dQ/dV is not exactly constant.
We say that the voltage on the black box is positive curves of parts A and B and suggest a reason using qualitative
when the potential on its red terminal is higher than arguments. You need to know that the nonlinear element also
the potential on the black terminal. Positive voltage has a capacitance ( 1 nF) which is connected in parallel to
will be acquired when the terminals of matching col- the nonlinear resistance.

Current source

page 4 of 5

IPhO-measure: short manual

Display

IPhO-measure is a multimeter capable of measuring voltage V


and current I simultaneously. It also records their time derivat their product P = V I, ratio R = V /I, and time
ives V and I,
t of the sample. Stored measurements are organized into separate sets; every stored sample is numbered by the set number
s and a counter n inside the set. All saved samples are written
to an internal ash memory and can later be retrieved.
Electrical behaviour
The device behaves as an ammeter and a voltmeter connected
as follows.
Internal
Range resistance
Voltmeter
0...2V
1 M
A
V
Voltmeter 2 . . . 10 V
57 k
Multimeter
Ammeter
0...1A
1
IN

OUT

GND

Basic usage
Push Power to switch the IPhO-measure on. The
device is not yet measuring; to start measuring, push
start. Alternatively, you can now start browsing your
stored data. See below.
To browse previously saved samples (through all sets),
press Previous or Next. Hold them down longer
to jump directly between sets.
A displayed sample consists of nine variables:
While not measuring, push Start to start measuring
a new set.
1. index n of the sample in the set;
2. index s of the set;
While measuring, push Sample each time you want to
3. time t since starting the set;
store a new set of data (i.e. of the readings shown on the
4. voltmeter output V ;
display).
5. rate of change of V (the time derivative V ); if derivative
cannot be reliably taken due to uctuations, +nan/s is
While measuring, you can also browse other samples of
shown;
the current set, using Previous and Next.
6. ammeter output I;
Press Stop to end a set and stop measuring. The
if derivative
7. rate of change of I (the time derivative I);
device is still on. You are ready to start a new meascannot be reliably taken due to uctuations, +nan/s is
uring session or start browsing stored data.
shown;
8.
product P = V I;
Pushing Power turns the device o. The device will
9.
ratio R = V /I.
show text my mind is going ...; do not worry, all the
data measurements will be stored and you will be able to
browse them after you switch the device on again. Saved If any of the variables is out of its allowed range, its display
samples will not be erased.
shows +inf or -inf.

page 5 of 5

Problem E1. The magnetic permeability of water is essentially unperturbed; connecting the respective points on
the graph, we obtain a line corresponding to a flat water sur(10 points)
Part A. Qualitative shape of the water surface (1 points)
Observing reflections from the water surface (in particular,
those of straight lines, such as the edge of a sheet of paper),
it is easy to see that the profile has one minimum and has a
relatively flat bottom, ie. the correct answer is Option D (full
marks are given also for Option B). This profile implies that
water is pushed away from the magnet, which means < 1
(recall that ferromagnets with > 1 are pulled).
Part B. Exact shape of the water surface (7 points)
i. (1.6 pts) The height of the spot on the screen y is tabulated
below as a function of the horizontal position x of the caliper.
Note that the values of y in millimetres can be rounded to integers (this series of measurements aimed as high as possible
precision).
x (mm) 10
15
20
25
30
32
34
36
y (mm) 11.5 15.6 19.8 24.3 30.2 33.2 37.2 40.5
x (mm) 38
40
42
44
46
48
50
52
y (mm) 42.2 41.4 40.3 40.3 40.8
42
43.2 44.4
x (mm) 54
56
58
60
62
64
66
68
y (mm) 45.3 45.8 45.4 44.4 43.6 46.2
50
53.6
x (mm) 70
72
74
76
78
80
85
90
y (mm) 56.7 59.5 61.6 63.5 65.3
67
70.9 74.9
ii. (0.7 pts)

On this graph, the data of to two different water levels are


depicted; blue curve corresponds to a water depth of ca 2 mm
(data given in the table above); the violet one to 1 mm.
iii. (0.5 pts) If the water surface were flat, the dependence of x
on y would be linear, and the tangent of the angle 0 would be
y
given by tan 0 = x
, where x is a horizontal displacement of
the pointer, and y the respective displacement of the spot
height. For the extreme positions of the pointer, the beam hits
the water surface so far from the magnet that there, the surface

face the red line. Using these two extreme data points we

can also easily calculate the angle 0 = arctan 74.911.5


9010 38 .

iv. (1.4 pts) For faster calculations, y y0 (x x0 ) tan 0


(appearing in the formula given) can be read from the previous
graph as the distance between red and blue line; the red line
is given by equation yr = y0 + (x x0 ) tan 0 . One can also
precalculate 21 cos2 0 0.31. The calculations lead to the following table (with z = tan 105 ; as mentioned above, during
the competition, lesser precision with two significant numbers
is sufficient).

x (mm) 10
z
0
x (mm) 38
z
597
x (mm) 54
z
-72
x (mm) 70
z
-154

page 1 of 4

15
10
40
428
56
-145
72
-74

20
27
42
239
58
-278
74
-40

25
66
44
128
60
-449
76
-20

30
204
46
53
62
-606
78
-6

32
303
48
26
64
-536
80
2

34
473
50
0
66
-388
85
-2

36
591
52
-26
68
-254
90
0

v. (1.6 pts) The water height can be obtained as the integral


R
h = tan dx. Thus, we calculate the water height row-byrow, by adding to the height in the previous row the product
of the horizontal displacement xi+1 xi with the average slope
1
2 (tan i+1 + tan i ).
x (mm) 10 15 20 25 30 32 34 36
h (m) 0
0
1
4 10 15 23 34
x (mm) 38 40 42 44 46 48 50 52
h (m) 46 56 63 66 68 69 69 69
x (mm) 54 56 58 60 62 64 66 68
h (m) 68 66 61 54 44 32 23 17
x (mm) 70 72 74 76 78 80 85 90
h (m) 12 10 9
8
8
8
8
8
Note that the water level height at the end of the table should
be also 0 (this corresponds also to an unperturbed region); the
non-zero result is explained by the measurement uncertainties.
One can improve the result by subtracting from h a linear trend
mm
8 m x10
80 mm .
If the water level above the magnet is 1 mm, the water level
descends below its unperturbed level at the axis of the magnet
by ca 120 m.
vi. (1 pt)

Similarly to the previous figure, blue curve corresponds to


a water depth of ca 2 mm, (data given in the table above), and
the violet one to 1 mm.
The position of the magnet can be found by measuring the
caliper (find the positions when the laser beam hits the edges of
the magnet and determine the distance between these positions
the result is ca 24 mm), and using the symmetry: magnet
is placed symmetrically with respect to the surface elevation
curve.
Part C. Magnetic permeability (2 points)
Water surface takes an equipotential shape; for a unit volume of
water, the potential energy associated with the magnetic interB2
action is 2
(1 1) B 2 1
20 ; the potential energy associated
0
with the Earths gravity is gh. At the water surface, the sum
of those two needs to be constant; for a point at unperturbed
surface, this expression equals to zero, so B 2 1
20 + gh = 0
2
and hence, 1 = 20 gh/B . Here, h = 120 m stands for
the depth of the water surface at the axis of the magnet; note
that we have compensated the cumulative error as described at
the end of the previous task and obtained h as the difference
between the depth at the magnets axis (121 m) and the halfdepth at the right-hand-side of the graph (1 m). Putting in
the numbers, we obtain 1 = 1.2 105 .

page 2 of 4

Problem E2. Nonlinear Black Box (10 points)


Part A. Circuit without inductance (7 points)
It is possible to make all the measurements needed for this
problem with a single circuit as shown in the figure. While the
current source is switched on, we are charging the capacitor in
the black box, until the current I(Vmax ) through the nonlinear
element equals to the output current I0 of the current source.
Vmax = 540 40 mVs varies from one experimental setup to another. When the current source is switched off or disconnected,
the capacitor will discharge through the nonlinear element.

Multimeter
IN
OUT

GND

When the voltage on the black box is Vmax , the current


through the nonlinear element is I0 . Switching the current source off, we will have the capacitor discharging
with the same current.
C0 = I0 /V (V = Vmax )

We can also measure the capacitance for any intermediate


voltage as in A-iv.
iii. (2.2 pts) If we neglect the nonlinearity of the capacitor,
there are (at least) two ways to obtain the currentvoltage characteristic of the nonlinear element in the black box.
Applying Kirchhoffs I law to the charging capacitor,
I(V ) = Ic C0 V (V ).

Black box

Current source
Switch
+

Switch

An I(V ) characteristic obtained by charging the capacitor


is shown on the following figure.

O
X I

O
I X

Applying Kirchhoff I law to the discharging capacitor,


i. (1 pt) During charging of the capacitor from V = 0 to
V = Vmax we note that the output of the current source is constant (I0 = 6.0 mA) close to the precision of the multimeter.
ii. (1.2 pts) Using the definition of differential capacitance,
we can calculate the current through the capacitor in the black
box from the time derivative of the voltage on the black box.

I(V ) = C0 V (V ).

6
Part A
Part B

dQ
dQ dV
Ic =
=
= C(V )V
dt
dV dt

There are several ways to determine the capacitance used in


the black box based on chosen voltage.
I (mA)

When the voltage on the black box is close to zero, the


current through the nonlinear element is also close to
zero, because I(V = 0) = 0. After switching the current
source on, most of the input current I0 will at first go
through the capacitor.

C0 = I0 /V (V = 0)
0

0
0.1
0.2
0.3
0.4
0.5
This can be measured more precisely after first reversing
V (V)
the polarity of the current source and charging the capacitor backwards, because the multimeter does not display iv. (2.6 pts) In order to obtain the differential capacitance,
derivatives when they change sharply (as in few moments we solve a system of linear equations by eliminating I(V ):
after switching the current source on).
(
Example measurements taken this way follow.
I0 = V C(V ) + I(V )
I0
= C(V ) =
.

V V
I(V ) = V C(V );
V (0) (mV/s) 3.51 3.32 3.55

C0 (F)

1.71
C0 = 1.74 F

1.81

1.69
Therefore we need to take measurements during both charging
and discharging the capacitor in the black box at the same
voltages. A graph of measurement results follows.
page 3 of 4

Part B. Circuit with inductance (3 points)


Measuring and plotting the currentvoltage characteristic of
the nonlinear element in the same way as in part A-iii, we
obtain a graph that differs only in the negative differential resistance (I (V ) < 0) region, in our case 70 mV < V < 330 mV.
This is the region where, when we look at small-signal oscillations, the nonlinear element behaves as a negative-valued
Ohmic resistance. After enabling the inductance we have a
LC circuit whose oscillations are amplified (instead of being
dampened) by the negative
q differential resistance. Because the
1
resonant frequency = LC
30 MHz (with Cp being the
p

2.1
2.05
2

C (F)

1.95

1.9
1.85

1.8

1.75
1.7

0.1

0.2

0.3
V (V)

0.4

0.5

capacitance of the nonlinear element) is high, we actually measure the average current through the nonlinear element, while
the real current oscillates all over the region of negative differential resistance.

page 4 of 4

The Maribo Meteorite

T1

Introduction
A meteoroid is a small particle (typically smaller than 1 m) from a comet or an asteroid. A
meteoroid that impacts the ground is called a meteorite.
On the night of 17 January 2009 many people near the Baltic Sea saw the glowing trail or fireball of
a meteoroid falling through the atmosphere of the Earth. In Sweden a surveillance camera recorded
a video of the event, see Fig. 1.1(a). From these pictures and eyewitness accounts it was possible to
narrow down the impact area, and six weeks later a meteorite with the mass 0.025 kg was found in
the vicinity of the town Maribo in southern Denmark. Measurements on the meteorite, now named
Maribo, and its orbit in the sky showed interesting results. Its speed when entering the atmosphere
had been exceptionally high. Its age,
year, shows that it had been formed shortly after
the birth of the solar system. The Maribo meteorite is possibly a part of Comet Encke.

The speed of Maribo


The fireball was moving in westerly direction, heading 285 relative to north, toward the location
where the meteorite was subsequently found, as sketched in Fig. 1.1. The meteorite was found at a
distance 195 km from the surveillance camera in the direction 230 relative to north.
Use this and the data in Fig. 1.1 to calculate the average speed of Maribo in the time
1.1 interval between frames 155 and 161. The curvature of the Earth and the gravitational 1.3
force on the meteoroid can both be neglected.

Through the atmosphere and melting?


The friction from the air on a meteoroid moving in the higher atmosphere depends in a complicated
way on the shape and velocity of the meteoroid, and on the temperature and density of the atmosphere. As a reasonable approximation the friction force in the upper atmosphere is given by the
expression
, where is a constant,
the density of the atmosphere, the projected cross-section area of the meteorite, and its speed.
The following simplifying assumptions are made to analyze the meteoroid: The object entering the
atmosphere was a sphere of mass
, radius
, temperature
, and
speed
. The density of the atmosphere is constant (its value 40 km above the
surface of the Earth),
, and the friction coefficient is
.
1.2a Estimate how long time after entering the atmosphere it takes the meteoroid to have its
speed reduced by 10 % from
to
. You can neglect the gravitational force on
the meteoroid and assume, that it maintains its mass and shape.

0.7

1.2b Calculate how many times larger the kinetic energy


0.3
of the meteoroid entering the
atmosphere is than the energy
necessary for melting it completely (see data sheet).

Page 1 of 4

The Maribo Meteorite


(a)

(b)

Frame Time Azimuth Altitude


155
1.46 s
215
19.2
161
2.28 s
221
14.7
Landing at M
230
0.0

(c)

Figure 1.1 (a) Azimuth is the clockwise angular position from north in the horizontal plane, and
altitude is the angular position above the horizon. A series of frames recorded by the surveillance
camera in Sweden, showing the motion of Maribo as a fireball on its way down through the
atmosphere. (b) The data from two frames indicating the time, the direction (azimuth) in degrees,
as seen by the camera (C), and the height above the horizon (altitude) in degrees. (c) Sketch of the
directions of the path (magenta arrow) of Maribo relative to north (N) and of the landing site (M)
in Denmark as seen by the camera (C).

Heating of Maribo during its fall in the atmosphere


When the stony meteoroid Maribo entered the atmosphere at supersonic speed it appeared as a fireball because the surrounding air was glowing. Nevertheless, only the outermost layer of Maribo was
heated. Assume that Maribo is a homogenous sphere with density
, specific heat capacity
,
and thermal conductivity
(for values see the data sheet). Furthermore, when entering the
atmosphere, it had the temperature
. While falling through the atmosphere its surface
temperature was constant
due to the air friction, thus gradually heating up the interior.

Page 2 of 4

T1

The Maribo Meteorite

T1

After falling a time in the atmosphere, an outer shell of Maribo of thickness will have been
heated to a temperature significantly larger than . This thickness can be estimated by dimensional
analysis as the simple product of powers of the thermodynamic parameters:
.
1.3a Determine by dimensional (unit) analysis the value of the four powers , , , and .

0.6

1.3b Calculate the thickness

0.4

after a fall time

, and determine the ratio

The age of a meteorite


The chemical properties of radioactive elements may be different, so during the crystallization of
the minerals in a given meteorite, some minerals will have a high content of a specific radioactive
element and others a low content. This difference can be used to determine the age of a meteorite by
radiometric dating of its radioactive minerals.
As a specific example, we study the isotope 87Rb (element no. 37), which decays into the stable
isotope 87Sr (element no. 38) with a half-life of
year, relative to the stable isotope
86
Sr. At the time of crystallization the ratio 87Sr/86Sr was identical for all minerals, while the ratio
87
Rb/86Sr was different. As time passes on, the amount of 87Rb decreases by decay, while
consequently the amount of 87Sr increases. As a result, the ratio 87Sr/86Sr will be different today. In
Fig. 1.2(a), the points on the horizontal line refer to the ratio 87Rb/86Sr in different minerals at the
time, when they are crystallized.

Figure 1.2 (a) The ratio 87Sr/86Sr in different minerals at the time
of crystallization (open
circles) and at present time (filled circles). (b) The isochron-line for three different mineral samples
taken from a meteorite at present time.
1.4a Write down the decay scheme for the transformation of
87

to

0.3

86

87

86

Show that the present-time ratio Sr/ Sr plotted versus the present-time ratio Rb/ Sr
in different mineral samples from the same meteorite forms a straight line, the so-called
1.4b
isochron-line, with slope
(
). Here is the time since the formation of the
minerals, while is the decay constant inversely proportional to half-life .

0.7

1.4c Determine the age

0.4

of the meteorite using the isochron-line of Fig. 1.2(b).

Page 3 of 4

The Maribo Meteorite

T1

Comet Encke, from which Maribo may originate


In its orbit around the Sun, the minimum and maximum distances between comet Encke and the
Sun are
and
, respectively.
1.5 Calculate the orbital period

0.6

of comet Encke.

Consequences of an asteroid impact on Earth


65 million years ago Earth was hit by a huge asteroid with density
, radius
, and final speed of
. This impact resulted in the extermination
of most of the life on Earth and the formation of the enormous Chicxulub Crater. Assume that an
identical asteroid would hit Earth today in a completely inelastic collision, and use the fact that the
moment of inertia of Earth is 0.83 times that for a homogeneous sphere of the same mass and
radius. The moment of inertia of a homogeneous sphere with mass
and radius
is
.
Neglect any changes in the orbit of the Earth.
1.6a

Let the asteroid hit the North Pole. Find the maximum change in angular orientation of
0.7
the axis of Earth after the impact.

1.6b

Let the asteroid hit the Equator in a radial impact. Find the change
of one revolution of Earth after the impact.

1.6c

Let the asteroid hit the Equator in a tangential impact in the equatorial plane. Find the
0.7
change
in the duration of one revolution of Earth after the impact.

in the duration

0.7

Maximum impact speed


Consider a celestial body, gravitationally bound in the solar system, which impacts the surface of
Earth with a speed
. Initially the effect of the gravitational field of the Earth on the body can be
neglected. Disregard the friction in the atmosphere, the effect of other celestial bodies, and the
rotation of the Earth.
1.7 Calculate

, the largest possible value of

Page 4 of 4

1.6

The Maribo Meteorite

T1

Solutions
Top
view

G
Frame 161

Turned sideview, not to


scale

Top view: Triangle MBC: |


, so
(
)
|
|
Then | |
(
)
Triangle DBC: | |
so
(
| |
Then | |
(

, and
.

1.3
,and

, and

)
)

,
.

)
(

Triangle EBC: | |
so
(
| |
Then | |
(
|

Triangle EDC:
Maribo:| |

1.1

195 km

Frame 155
C

Horizontal distance traveled by

)
)

(
)
Side view: Triangle CDF: | | | |
(
)
Triangle CEG: | | | |
Thus vertical distance travelled by Maribo: | | | |
Total distance travelled by Maribo from frame 155 to 161:
| | | |
(| | | |)
.

The speed of Maribo is

Newton's second law:


By integration

yields
(

1.2a Alternative solution: The average force on the meteoroid when the speed decreases from 0.7
(
) . Using that the acceleto
can be estimated to
(
)
ration is approximately constant,
, results in
.

Page 1 of 3

The Maribo Meteorite


1.2b

[ ] [ ] [
] [ ] [
so [ ] [ ]
[ ]
1.3a Thus
,

1.3b

[ ] [
[ ]

] [
[ ]
.

] [

] ,

, and

) and ( )

( )

and Rb Sr:
( )
( ) (
Thus
)
1.4b equation of a straight line:
( )

Slope:
)

0.4
0.3

( )
( ) [
( ), and dividing by
( )

( )
( )].
we obtain the

0.7

( )

( )

and
(

0.6

( )

1.4c

.
.

1.6 mm

1.4a Rb-Sr decay scheme:

So

0.3

From which (

T1

.
0.4

( )

Kepler's 3rd law on comet Encke and Earth, with the orbital semi-major axis of Encke
1.5

given by

). Thus

( )

For Earth around its rotation axis: Angular velocity

Moment of inertia
Angular momentum
1.6a
Asteroid:

.
and angular momentum
.
is perpendicular to , so by
conservation angular momentum:
( )
=
.The axis tilt
(so the North Pole moves
).
At vertical impact

1.6b
in rotation period is
1.6c

At tangential impact
thus
(

0.6

so (
(

)
. Thus
we obtain
)

is parallel to
)
(

so
)

Page 2 of 3

, and since
. The change
.

)(

0.7

0.7

) and
.

0.7

The Maribo Meteorite

Maximum impact speed


(I) The velocity

arises from three contributions:

of the body at distance

(Earth orbit radius) from the Sun,

.
1.7 (II) The orbital velocity of the Earth,
(III) Gravitational attraction from the Earth and kinetic energy seen from the Earth:
(
)
(
) .
In conclusion:

T1

1.6

Total

9.0

Page 3 of 3

Plasmonic Steam Generator

T2

Introduction
In this problem we study an efficient process of steam production that has been demonstrated to
work experimentally. An aqueous solution of spherical nanometer-sized silver spheres
(nanoparticles) with only about
particles per liter is illuminated by a focused light beam. A
fraction of the light is absorbed by the nanoparticles, which are heated up and generate steam
locally around them without heating up the entire water solution. The steam is released from the
system in the form of escaping steam bubbles. Not all details of the process are well understood at
present, but the core process is known to be absorption of light through the so-called collective
electron oscillations of the metallic nanoparticles. The device is known as a plasmonic steam
generator.

Figure 2.1 (a) A spherical charge-neutral nanoparticle of radius R placed at the center of the coordinate
system. (b) A sphere with a positive homogeneous charge density (red), and containing a smaller spherical
charge-neutral region (0, yellow) of radius , with its center displaced by
. (c) The sphere with
positive charge density of the nanoparticle silver ions is fixed in the center of the coordinate system. The
center of the spherical region with negative spherical charge density (blue) of the electron cloud is
displaced by
, where
. (d) An external homogeneous electric field
. For time)
dependent
, the electron cloud moves with velocity
. (e) The rectangular vessel (
containing the aqueous solution of nanoparticles illuminated by monochromatic light propagating along the
-axis with angular frequency
and intensity .

A single spherical silver nanoparticle


Throughout this problem we consider a spherical silver nanoparticle of radius
and
with its center fixed at the origin of the coordinate system, see Fig. 2.1(a). All motions, forces and
driving fields are parallel to the horizontal -axis (with unit vector ). The nanoparticle contains
free (conduction) electrons moving within the whole nanoparticle volume without being bound to
any silver atom. Each silver atom is a positive ion that has donated one such free electron.
Find the following quantities: The volume and mass of the nanoparticle, the
2.1 number and charge density of silver ions in the particle, and for the free electrons
their concentration , their total charge , and their total mass
.

Page 1 of 4

0.7

Plasmonic Steam Generator

T2

The electric field in a charge-neutral region inside a charged sphere


For the rest of the problem assume that the relative dielectric permittivity of all materials is
.
Inside a charged sphere of homogeneous charge density and radius R is created a small spherical
charge-neutral region of radius
by adding the opposite charge density
, with its center displaced by
from the center of the R-sphere, see Fig. 2.1(b).
2.2

Show that the electric field inside the charge-neutral region is homogenous of the form
(
) , and determine the pre-factor .

1.2

The restoring force on the displaced electron cloud


In the following, we study the collective motion of the free electrons, and therefore model them as a
single negatively charged sphere of homogeneous charge density
with a center position ,
which can move along the x-axis relative to the center of the positively charged sphere (silver ions)
fixed at the origin of the coordinate system, see Fig. 2.1(c). Assume that an external force
displaces the electron cloud to a new equilibrium position
with
. Except for
tiny net charges at opposite ends of the nanoparticle, most of its interior remains charge-neutral.
2.3

Express in terms of
and n the following two quantities: The restoring force exerted
on the electron cloud and the work
done on the electron cloud during displacement.

1.0

The spherical silver nanoparticle in an external constant electric field


A nanoparticle is placed in vacuum and influenced by an external force
due to an applied static
homogeneous electric field
, which displaces the electron cloud the small distance
,
where
.
Find the displacement
of the electron cloud in terms of
, and determine the
2.4 amount
of electron charge displaced through the yz-plane at the center of the 0.6
nanoparticle in terms of
.

The equivalent capacitance and inductance of the silver nanoparticle


For both a constant and a time-dependent field , the nanoparticle can be modeled as an equivalent
electric circuit. The equivalent capacitance can be found by relating the work
, done on the
separation of charges
, to the energy of a capacitor, carrying charge
. The charge separation
will cause a certain equivalent voltage across the equivalent capacitor.
2.5a Express the systems equivalent capacitance C in terms of
2.5b

and , and find its value.

For this capacitance, determine in terms of


and
the equivalent voltage
should be connected to the equivalent capacitor in order to accumulate the charge

Page 2 of 4

0.7

that
0.4
.

Plasmonic Steam Generator

T2

For a time-dependent field , the electron cloud moves with velocity


, Fig. 2.1(d). It has
the kinetic energy
and forms an electric current flowing through the fixed yz-plane. The
kinetic energy of the electron cloud can be attributed to the energy of an equivalent inductor of
inductance carrying the current .
2.6a Express both
2.6b

and

0.7

in terms of the velocity .

Express the equivalent inductance in terms of particle radius , the electron charge
and mass
, the electron concentration , and calculate its value.

0.5

The plasmon resonance of the silver nanoparticle


From the above analysis it follows that the motion, arising from displacing the electron cloud from
its equilibrium position and then releasing it, can be modeled by an ideal LC-circuit oscillating at
resonance. This dynamical mode of the electron cloud is known as the plasmon resonance, which
oscillates at the so-called angular plasmon frequency .
2.7a

Find an expression for the angular plasmon frequency


of the electron cloud in terms
0.5
of the electron charge and mass
, the electron density , and the permittivity .

2.7b

Calculate
frequency

in rad/s and the wavelength


.

in nm of light in vacuum having angular

0.4

The silver nanoparticle illuminated with light at the plasmon frequency


In the rest of the problem, the nanoparticle is illuminated by monochromatic light at the angular
plasmon frequency
with the incident intensity
. As the wavelength
is large,
, the nanoparticle can be considered as being placed in a homogeneous harmonically oscillating field
(
) . Driven by , the center ( ) of the electron cloud
oscillates at the same frequency with velocity
and constant amplitude
. This
oscillating electron motion leads to absorption of light. The energy captured by the particle is either
converted into Joule heating inside the particle or re-emitted by the particle as scattered light.
Joule heating is caused by random inelastic collisions, where any given free electron once in a
while hits a silver ion and loses its total kinetic energy, which is converted into vibrations of the
silver ions (heat). The average time between the collisions is
, where for silver
nanoparticle we use
.
Find an expression for the time-averaged Joule heating power
in the nanoparticle
2.8a as well as the time-averaged current squared , which includes explicitly the time- 1.0
averaged velocity squared of the electron cloud.
Find an expression for the equivalent ohmic resistance
2.8b model of the nanoparticle having the Joule heating power
current I. Calculate the numerical value of
.

Page 3 of 4

in an equivalent resistordue to the electron cloud 1.0

Plasmonic Steam Generator

T2

The incident light beam loses some time-averaged power


by scattering on the oscillating electron cloud (re-emission).
depends on the scattering source amplitude , charge , angular frequency
and properties of the light (the speed of light and permittivity in vacuum). In terms
of these four variables,

2.9

By use of
logy with

is given by

, find an expression of the equivalent scattering resistance


) in an equivalent resistor-model, and calculate its value.

(in ana-

1.0

The above equivalent circuit elements are combined into an LCR series circuit model of the silver
nanoparticle, which is driven by a harmonically oscillating equivalent voltage
(
)
determined by the electric field
of the incident light.
Derive expressions for the time-averaged power losses
and
involving the
of the electric field in the incident light at the plasmon resonance 1.2
2.10a amplitude
.
2.10b Calculate the numerical value of

, and

0.3

Steam generation by light


An aqueous solution of silver nanoparticles is prepared with a concentration
.
It is placed inside a rectangular transparent vessel of size
and
illuminated by light at the plasmon frequency with the same intensity
at normal
incidence as above, see Fig. 2.1(e). The temperature of the water is
and we assume, in
fair agreement with observations, that in steady state all Joule heating of the nanoparticle goes to
the production of steam of temperature
, without raising the temperature of the water.
The thermodynamic efficiency of the plasmonic steam generator is defined by the power ratio
, where
is the power going into the production of steam in the entire vessel, while
is the total power of the incoming light that enters the vessel.
Most of the time any given nanoparticle is surrounded by steam instead of water, and it can thus be
described as being in vacuum.
2.11a

Calculate the total mass per second


of steam produced by the plasmonic steam ge0.6
nerator during illumination by light at the plasmon frequency and intensity .

2.11b

Calculate the numerical value of the thermodynamic efficiency


steam generator.

Page 4 of 4

of the plasmonic

0.2

Plasmonic Steam Generator

T2

Solutions
A single spherical silver nanoparticle
Volume of the nanoparticle:
Mass of the nanoparticle:

.
.

Number of ions in the nanoparticle:

2.1 Charge density

, charge density

Electrons concentration
Total charge of free electrons
Total mass of free electrons

0.7

.
.
.

The electric field in a charge-neutral region inside a charged sphere


For a sphere with radius and constant charge density , for any point inside the sphere
designated by radius-vector
) Gauss's law yields directly
, where
is the unit radial vector pointing away from the center of the
sphere. Thus,

Likewise, inside another sphere of radius


and charge density
the field is
, where is the radius-vector of the point in the coordinate system with the origin 1.2
2.2
in the center of this sphere.
Superposition of the two charge configurations gives the setup we want with
. So
inside the charge-free region |
|
the field is
or

with pre-factor

The restoring force on the displaced electron cloud


With
and
inside the particle is
produced
2.3

we have from above that approximately the field induced


. The number of electrons on the particles border that

is negligibly smaller than the number of electrons inside the particle, so


(note the antiparallel attractive force

is proportional to the displacement that it is similar to Hookes law).


The
work
done
on
the
electron
cloud
to

shift

it

1.0

is

The spherical silver nanoparticle in an external constant electric field


Inside the metallic particle in the steady state the electric field must be equal to 0. The
2.4 induced field (from 2.2 or 2.3) compensates the external field:
, so 0.6

Page 1 of 3

Plasmonic Steam Generator

T2

.
Charge displaced through the
radius and height :

-plane is the total charge of electrons in the cylinder of


.

The equivalent capacitance and inductance of the silver nanoparticle


The electric energy
2.5a

of a capacitor with capacitance

holding charges

. The energy of such capacitor is equal to the work (see 2.3) done to separate
the charges (see 2.4), thus

2.5b

is
0.7

Equivalent scheme for a capacitor reads:


capacitance from (2.5a) gives

. Combining charge from (2.4) and


.

0.4

The kinetic energy of the electron cloud is defined as the kinetic energy of one electron
multiplied
by
the
number
of
electrons
in
the
cloud
(
).
2.6a
0.7
The current is the charge of electrons in the cylinder of area
and height
divided by time
(or simply the time derivative of charge
), thus
.
The energy carried by current in the equivalent circuit with inductance is
2.6b is, in fact, the kinetic energy of electrons
. Taking the energy and current from 0.5
(2.6a) gives
.

The plasmon resonance of the silver nanoparticle


From the LC-circuit analogy we can directly derive

.
2.7a Alternatively it is possible to use the harmonic law of motion in (2.3) and get the same 0.5
result for the frequency.
rad/s, for light with angular frequency
.

2.7b

the wavelength is

0.4

The silver nanoparticle illuminated with light at the plasmon frequency


The velocity of an electron
kinetic energy on the electron
2.8a

. The time-averaged

. During time

each electron

hits an ion one time. So the energy lost in the whole nanoparticle during time

. Time-averaged Joule heating power


).

The expression for current is taken from (2.6a), squared and averaged

Page 2 of 3

is

1.0

Plasmonic Steam Generator


( )

T2

The average time between the collisions is


, so each electron oscillates many
times before it collides with an ion. The oscillating current
2.8b
, 1.0
produces the heat in the resistance
equal to
that together with results from (2.8a) leads to
.

For equivalent scattering resistance

and for harmonic oscillations we can

2.9 average the velocity squared over one period of oscillations, so


Together it yields

. At the resonance

frequency time-averaged voltage squared is

. 1.0

Ohms law for a LCR serious circuit is

2.10a And from (2.5b)

, so Ohms law results in


The time-averaged power losses are

1.2

and

2.10b Starting with the electric field amplitude


and
.

, we calculate 0.3

Steam generation by light


Total number of nanoparticles in the vessel:
Then the
total time-averaged Joule heating power:
kW. This power goes
, with
2.11a into the steam generation:
. Thus the mass of steam produced in one second
is:
.

0.6

The power of light incident on the vessel


, and the power directed for steam production by nanoparticles is given in
2.11b
2.11a. Efficiency of the process is
= 0.498.

0.2

Total

12.0

Page 3 of 3

The Greenlandic Ice Sheet


Introduction
This problem deals with the physics of the Greenlandic ice sheet, the second largest glacier in the
world, Fig. 3.1(a). As an idealization, Greenland is modeled as a rectangular island of width
and
length
with the ground at sea level and completely covered by incompressible ice (constant
density
), see Fig. 3.1(b). The height profile
of the ice sheet does not depend on the coordinate and it increases from zero at the coasts
to a maximum height
along the
middle north-south axis (the -axis), known as the ice divide, see Fig. 3.1(c).

(c)

Figure 3.1 (a) A map of Greenland showing the extent of the ice sheet (white), the ice-free, coastal regions
(green), and the surrounding ocean (blue). (b) The crude model of the Greenlandic ice sheet as covering a
rectangular area in the -plane with side lengths
and . The ice divide, the line of maximum ice sheet
height
runs along the -axis. (c) A vertical cut ( -plane) through the ice sheet showing the height
profile
(blue line).
is independent of the -coordinate for
, while it drops abruptly
to zero at
and
. The -axis marks the position of the ice divide. For clarity, the vertical
dimensions are expanded compared to the horizontal dimensions. The density
of ice is constant.

Page 1 of 4

T3

The Greenlandic Ice Sheet

T3

Two useful formulas


In this problem you can make use of the integral:

and the approximation

, valid for |

The height profile of the ice sheet


On short time scales the glacier is an incompressible hydrostatic system with fixed height profile
.
Write down an expression for the pressure
inside the ice sheet as a function of
vertical
height
z
above
the
ground
and
distance
from
the ice divide. Neglect the
3.1
0.3
atmospheric pressure.
Consider a given vertical slab of the ice sheet in equilibrium, covering a small horizontal base area
between and
, see the red dashed lines in Fig. 3.1(c). The size of
does not matter.
The net horizontal force component
on the two vertical sides of the slab, arising from the
difference in height on the center-side versus the coastal-side of the slab, is balanced by a friction
force
from the ground on the base area
, where
.
3.2a For a given value of , show that in the limit
,
, and determine k 0.9
Determine an expression for the height profile
in terms of
, , , and
3.2b distance from the divide. The result will show, that the maximum glacier height
scales with the half-width as
.
3.2c

Determine the exponent with which the total volume


the area of the rectangular island,
.

of the ice sheet scales with

0.8

0.5

A dynamical ice sheet


On longer time scale, the ice is a viscous incompressible fluid, which by gravity flows from the
center part to the coast. In this model, the ice maintains its height profile
in a steady state,
where accumulation of ice due to snow fall in the central region is balanced by melting at the coast.
In addition to the ice sheet geometry of Fig. 3.1(b) and (c) make the following model assumptions:
1)
2)
3)
4)
5)

Ice flows in the


-plane away from the ice divide (the -axis).
The accumulation rate (m/year) in the central region is a constant.
Ice can only leave the glacier by melting near the coasts at
.
The horizontal ( -)component
of the ice-flow velocity is independent of .
The vertical -)component
of the ice-flow velocity is independent of .

Consider only the central region | |


close to the middle of the ice sheet, where height
variations of the ice sheet are very small and can be neglected altogether, i.e.
.
3.3

Use mass conservation to find an expression for the horizontal ice-flow velocity
in terms of , , and
.

Page 2 of 4

0.6

The Greenlandic Ice Sheet

T3

From the assumption of incompressibility, i.e. the constant density


of the ice, it follows that
mass conservation implies the following restriction on the ice flow velocity components

3.4

Write down an expression for the


ice-flow velocity.

dependence of the vertical component

of the

0.6

A small ice particle with the initial surface position


will, as time passes, flow as part of the
ice sheet along a flow trajectory
in the vertical -plane.
3.5 Derive an expression for such a flow trajectory
0.9
.

Age and climate indicators in the dynamical ice sheet


Based on the ice-flow velocity components
and
, one can estimate the age
ice in a specific depth
from the surface of the ice sheet.
3.6

Find an expression for the age


right at the ice divide
.

of the ice as a function of height

of the

above ground,

1.0

An ice core drilled in the interior of the Greenland ice sheet will penetrate through layers of snow
from the past, and the ice core can be analyzed to reveal past climate changes. One of the best
indicators is the so-called
, defined as

[
] [
] denotes the relative abundance of the two stable isotopes
where
and
oxygen. The reference
is based on the isotopic composition of the oceans around Equator.

of

Figure 3.2 (a) Observed relationship between


in snow versus the mean annual surface temperature .
(b) Measurements of
versus depth
from the surface, taken from an ice core drilled from surface
to bedrock at a specific place along the Greenlandic ice divide where
m.

Page 3 of 4

The Greenlandic Ice Sheet

T3

Observations from the Greenland ice sheet show that


in the snow varies approximately
linearly with temperature, Fig. 3.2(a). Assuming that this has always been the case,
retrieved
from an ice core at depth
leads to an estimate of the temperature near Greenland at the
age
.
Measurements of
in a 3060 m long Greenlandic ice core show an abrupt change in
at a
depth of 1492 m, Fig. 3.2(b), marking the end of the last ice age. The ice age began 120,000 years
ago, corresponding to a depth of 3040 m, and the current interglacial age began 11,700 years ago,
corresponding to a depth of 1492 m. Assume that these two periods can be described by two
different accumulation rates,
(ice age) and
(interglacial age), respectively. You can assume
to be constant throughout these 120,000 years.
3.7a Determine the accumulation rates

and

3.7b Use the data in Fig. 3.2 to find the temperature change at the transition from the ice age
to the interglacial age.

0.8
0.2

Sea level rise from melting of the Greenland ice sheet


A complete melting of the Greenlandic ice sheet will cause a sea level rise in the global ocean. As a
crude estimate of this sea level rise, one may simply consider a uniform rise throughout a global
ocean with constant area
.
Calculate the average global sea level rise, which would result from a complete melting
3.8 of the Greenlandic ice sheet, given its present area of
and 0.6
.
The massive Greenland ice sheet exerts a gravitational pull on the surrounding ocean. If the ice
sheet melts, this local high tide is lost and the sea level will drop close to Greenland, an effect
which partially counteracts the sea level rise calculated above.
To estimate the magnitude of this gravitational pull on the water, the Greenlandic ice sheet is now
modeled as a point mass located at the ground level and having the total mass of the Greenlandic ice
sheet. Copenhagen lies at a distance of 3500 km along the Earth surface from the center of the point
mass. One may consider the Earth, without the point mass, to be spherically symmetric and having
a global ocean spread out over the entire surface of the Earth of area
. All
effects of rotation of the Earth may be neglected.
3.9

Within this model, determine the difference


between sea levels in
Copenhagen (
) and diametrically opposite to Greenland (
).

Page 4 of 4

1.8

The Greenlandic Ice Sheet


Solutions
3.1

The pressure is given by the hydrostatic pressure (, ) = ice (() ), which is


0.3
zero at the surface.
The outward force on a vertical slice at a distance from the middle and of a given
width is obtained by integrating up the pressure times the area:
()
1
() =
ice (() ) d = ice ()2
2
0
d

3.2a which implies that = () ( + ) = d = ice () d .


0.9
This finally shows that

d
b =
= ice ()

d
Notice the sign, which must be like this, since was defined as positive and () is a
decreasing function of .
To find the height profile, we solve the differential equation for ():
b
d 1 d

= ()
=
()2
ice
d 2 d
with the boundary condition that () = 0. This gives the solution:
2
() =
1 /
ice
2

Which gives the maximum height m =

ice

Alternatively, dimensional analysis could be used in the following manner. First notice

that = [m ] = ice
b . Using that ice = 3, [] = 2, [ ] =


1 2
+ 3++ 22
[ ]

, 0.8
3.2b , demands that = m = =
which again implies + = 0, 3 + + = 1, 2 + 2 = 0. These three
equations are solved to give = = = 1, which shows that

ice

Since we were informed that m , it follows that = 1/2. With the boundary
condition () = 0, the solution then take the form
()

ice

1/2

The proportionality constant of 2 cannot be determined in this approach.

Page 1 of 6

T3

The Greenlandic Ice Sheet

T3

For the rectangular Greenland model, the area is equal to = 102 and the volume is
found by integrating up the height profile found in problem 3.2b:
3.2c

1/2

G,ice = (5)2 0 () d = 10 0 b
2

= 10m 3 (1 )

3/2

=
0

ice

20
3

1 / d = 10m 2 0 1 d

m 2 5/2,

0.5

where the last line follows from the fact that m . Note that the integral need not
be carried out to find the scaling with . This implies that G,ice 5/4 and the wanted
exponent is = 5/4.
According to the assumption of constant accumulation c the total mass accumulation
rate from an area of width between the ice divide at = 0 and some point at > 0
must equal the total mass flux through the corresponding vertical cross section at .
3.3
0.6
That is: = m (), from which the velocity is isolated:

() =
m
From the given relation of incompressibility it follows that
d
d

=
=
d
d
m
3.4
Solving this differential equation with the initial condition (0) = 0, shows that:

() =
m

0.6

Solving the two differential equations


d

=
and
=
d
m
d m
with the initial conditions that (0) = m , and (0) = gives
() = m e/m
and
() = e/m
3.5 This shows that = m /, meaning that flow lines are hyperbolas in the -plane. 0.9
Rather than solving the differential equations, one can also use them to show that
d
d
d

() =
+
=

=0
d
d
d m
m
which again implies that = const. Fixing the constant by the initial conditions, again
leads to the result that = m /.
3.6

At the ice divide, = 0, the flow will be completely vertical, and the -dependence of
1.0

found in 3.5 can be inverted to find (). One finds that () = m ln m .

Page 2 of 6

The Greenlandic Ice Sheet

T3

The present interglacial period extends to a depth of 1492 m, corresponding to 11,700


year. Using the formula for ()from problem 3.6, one finds the following accumulation
rate for the interglacial:
m
m
ig =
ln
= 0.1749 m/year.
11,700 years
m 1492 m

3.7a

The beginning of the ice age 120,000 years ago is identified as the drop in 18O in
figure 3.2b at a depth of 3040 m. Using the vertical flow velocity found in problem 3.4,
d

on has = d, which can be integrated down to a depth of 3040 m, using a


m

stepwise constant accumulation rate:


m ln

m 3040 m
m
1
= m
d
m 3040 m

120,000 year

11,700 year

ia d +

0.8

11,700 year

ig d

= ia (120,000 year-11,700 year)+ig 11,700 year

3.7b

Isolating form this equation leads to ia = 0.1232, i.e. far less precipitation than now.

Reading off from figure 3.2b: 18O changes from 43,5 to 34,5 . Reading off
from figure 3.2a, then changes from 40 to 28 . This gives 12 .

0.2

From the area G one finds that = G /10 = 4.14 105 m. Inserting numbers in
the volume formula found in 3.2c, one finds that:
G,ice =

3.8

20 5/2 2b

= 3.45 1015 m3
3
ice

This ice volume must be converted to liquid water volume, by equating the total masses,

i.e. G,wa = G,ice ice = 3.17 1015 m3 , which is finally converted to a sea level rise,
as G,rise =

G,wa
o

wa

= 8.79 m.

Page 3 of 6

0.6

The Greenlandic Ice Sheet

Figure 3.S1 Geometry of the ice ball (white circle) with a test mass (small gray circle).

The total mass of the ice is

ice = G,ice ice = 3.17 1018 kg = 5.31 107 E

The total gravitational potential felt by a test mass at a certain height above the surface
3. of the Earth, and at a polar angle (cf. figure 3.S1), with respect to a rotated polar axis 1.
9 going straight through the ice sphere is found by adding that from the Earth with that from 6
the ice:
E ice
1
/
tot =

=
+

E +

1 + /
/
where = /2 . Since /E 1 one may use the approximation given in the
problem, (1 + x)1 1 , || 1, to approximate this by
/
tot 1
+
.

/
Isolating now shows that = 0 +

/
/

, where 0 = + tot /(). Using

again that /E 1, trigonometry shows that 2E |sin(/2)|, and one has:


() 0

ice /E
1.69 m

.
|sin(/2)|
2|sin(/2)|

To find the magnitude of the effect in Copenhagen, the distance of 3500 km along the
surface is used to find the angle CPH = (3.5 106 m)/ 0.549, corresponding to
CPH 0 6.25 m. Directly opposite to Greenland corresponds to = , which gives
OPP 0 1.69 m. The difference is then CPH OPP 4.56 m, where 0 has dropped
out.

Page 4 of 6

T3

The Greenlandic Ice Sheet

Figure 3.S2 Same figure as above, but with the relevant forces depicted and showed again
outside figure for clarity. The blue dotted line indicates the Earth surface. The blue dashed line
indicates the local sea level, growing towards Greenland and decreasing towards the south pole.

Approach with forces:


This problem can also be solved using forces. The basic equations for mechanical
equilibrium of the test particle is then a simple matter of balancing the two gravitational
forces, and , with the reaction force from the Earth, . Given the angles indicated in
Figure 3.S2, the force balance along locally vertical and horizontal directions, respectively,
read
+ cos() = cos()
and
sin() = sin()
which can be divided to obtain (using that = /2 /2):

sin()
+ cos()

1
= cos(/2)

1 + ( / )sin(/2)

cos(/2)

/
=
cos(/2)
(/ )2
/
=
cos(/2)
4 sin2 (/2)

tan() =

where we have plugged in the gravitational forces and the relevant distances. We have also

Page 5 of 6

T3

The Greenlandic Ice Sheet

T3

approximated the fraction, using that / = 5.31 107 1, which is only valid
not too close to Greenland, i.e. for a certain size of . Since the local sea surface will be
perpendicular to the reaction force, it is seen from figure 3.S2 that

whereby

tan() =

d d d
1 d
=
=
d d d d

d
/
=
cos(/2)
d
4 sin2 (/2)

The difference in sea levels in Copenhagen and opposite to Greenland can now be obtained
by integrating this expression. That is
cos(/2)

d

4 sin2 (/2)
sin(/2) 2
=

q d
2 1

1
=

1
2 sin( /2)

CPH OPP =

where we have made the substitution = sin(/2). Plugging in the numbers found above,
we obtain again CPH OPP 4.56. Note that this solution strategy necessarily involves
consideration of tangential force components alongside with the radial components.
Total

9.0

Page 6 of 6

Speed of light
Notice: All measurements and calculated values must be presented with SI units with an
appropriate number of significant digits. Uncertainties required only when explicitly asked for.

1.0 Introduction
Experiments with a laser distance meter (LDM)

Figure 1.1 Equipment for the first experiments 1.1 and 1.2.
A: Laser distance meter
B: Fiber optic cable (approximately 1 m)
C: Self-adhesive black felt pads with hole
D: Tape measure
E: Tape
F: Scissors

G: Lid from the black box


A laser distance meter (LDM, see Fig. 1.2 and Fig. 1.3) consists of an emitter and a receiver. The
emitter is a diode laser that emits a modulated laser beam, i.e. a laser beam for which the amplitude
varies at a very high frequency. When the laser beam hits an object, light is reflected in all
directions from the laser dot. Some of this light returns to the instruments receiver which is situated

Page 1 of 9

E1

Speed of light
immediately next to the emitter. The instruments telescope optics is focused on the laser dot and
receives the light returned from the laser dot. The electronics of the instrument measures the time
difference in the modulation of the received light signal relative to the emitted light signal. The
delay in the modulation is exactly the time it takes for the light to travel from emitter to receiver.
The measured time is then converted to a value
1

= +
2

This value is shown in the instruments display. Here, = 2.998 108 ms1 is the speed of light.
The constant depends on the instrument setting; on the instrument you can switch between
measuring the distance either from the rear end or from the front end of the instrument. When the
laser distance meter is turned on, the default setting is to measure from the rear. This setting shall
be maintained during all measurements.
Due to parallax, the LDM cannot measure any distance shorter than 5 cm. The maximum distance
that can be measured is around 25 m. The shape of the instrument is such that the rear side is
perpendicular to the laser beam as well as the front side. When the instrument is lying on the table
the polarization is vertical (perpendicular to the display)
The diode laser is of class 2 with power < 1 mW and wavelength 635 nm. Manifacturer
uncertainty for measurements is +/- 2 mm.
Warning: The instruments diode laser can damage your eyes. Do not look into the laser beam
and do not shine it into other peoples eyes!
Settings for LDM
The above calculation of the distance of course assumes that the light has been travelling at
speed . At the level of accuracy in this experiment, there is no need to distinguish between the
speed of light in vacuum and in air, since the refractive index for dry, atmospheric air at normal
pressure and temperature is 1.000 29 1.000.

Page 2 of 9

E1

Speed of light
Figure 1.2 The unlabeled six buttons are irrelevant (they are used to calculate area and volume). The
relevant buttons are:
A: On/off
B: Switch between measurement from the rear and the front of the instrument.
C: Indicator for measurement from the rear/front
D: Turn on laser/start measurement
E: Continuous measurement
F: Indicator for continuous measurement

Figure 1.3 The laser distance meter seen from the front end:
A: Receiver: Lens for the telescope focused on the laser dot
B: Emitter: Do not look into the laser beam!

1.1 Measurement with the laser distance meter


The instrument will perform a measurement when you press the button D, see Fig. 1.2.

1.1

Use the LDM to measure the distance from the top of the table to the floor. Write
0.4
down the uncertainty . Show with a sketch how you perform this measurement.

Page 3 of 9

E1

Speed of light
1.2 Experiment with the fiber optic cable

Figure 1.4 Diagram of a fiber optic cable.

You have been given a fiber optic cable of length approximately 1 m and diameter approximately
2 mm. The cable consists of two optical materials. The core (diameter approximately 1 mm) is
made from a plastic with a high refractive index. The core is surrounded by a cladding made from a
plastic with a slightly lower refractive index, and this is covered by a protective jacket of black
plastic. Core and cladding serve as a wave guide for light shone into the cable, since the boundary
between core and cladding will cause total reflection and thereby prevent the light from leaving
the core as long as the angle of incidence is larger than the critical angle for total reflection. The
light will therefore follow the core fiber, even if the cable bends, as long as it is not bent too much.
The LDM should now be set for continuous measurement (E, see Fig. 1.2), so that the display
indication updates approximately once per second. The LDM will automatically go into sleep
mode after a few minutes. It can be reactivated by pushing the red start button.
Carefully and gently cover the lens of the receiver with one small, black felt pad (the other is a
backup) with a hole of diameter 2 mm (see figure 1.3A). The adhesive side of the pad should be
pressed softly against the lens. Insert a fiber optic cable of length in the hole in the pad so that it
touches the lens,
see Fig. 1.5.

Page 4 of 9

E1

Speed of light

Figure 1.5 (a) Felt pad and fiber optic cable. (b) Attaching the fiber optic cable.

The other end of the cable should be held against the emitter, so that it touches the glass in the
middle of the laser beam. Now read off the -value from the display. The supplied scissors should
be used to cut the fiber optic cable into different lengths .
Think very carefully before cutting the fiber optic cable, as you cannot get any more cable!!

Notice also that the LDM display might show a thermometer icon after a while in the continuous
mode due to excessive heating of the electronics. If this happens, turn off the LDM for a while to
cool off the instrument.

1.2a

Measure corresponding values of and . Set up a table with your measurements. Draw
1.8
a graph showing as a function of .

Use the graph to find the refractive index co for the material from which the core of
1.2b the fiber optic cable is made. Calculate the speed of light co in the core of the fiber 1.2
optic cable.

Page 5 of 9

E1

Speed of light
1.3 Laser distance meter at an angle from the vertical
In this part of the experiment you will need the equipment shown in Fig. 1.6.

Figure 1.6 Equipment for experiment 1.3 shown in the figure:


A: Optical vessel with water and measuring tape
B: Magnet to secure the angle iron on top of the black box. (You find magnet placed on the angle iron).
C: Angle iron with self-adhesive foam pads
D: Self-adhesive foam pads

Remove the black felt pad from the lens. The LDM should now be placed in the following set-up:
Place two self-adhesive foam pads on the angle iron, see A on Fig. 1.7.

Page 6 of 9

E1

Speed of light

Figure 1.7 How to place the two self-adhesive foam pads on the angle iron.

The LDM should be carefully placed on the angle iron as shown in Fig. 1.8.

Figure 1.8 How to place the laser distance meter on the angle iron.

The angle iron with the LDM should be mounted on the black box as shown in Fig. 1.9. Secure the
angle iron to the box with a magnet placed below inside the box. (The tiny magnet is found on the
angle iron). It is important to mount the LDM exactly as in the photo, since the side of the box
facing upwards slants by approximately 4 degrees. The laser beam should now be pointing
unobstructedly downwards at an angle.

Page 7 of 9

E1

Speed of light

Figure 1.9 The experimental set-up. (The black box only serves as a support. The equipment behind the
bottle is not used, though).
A: Important: The bottom of the black box must face forward as shown. The side that faces upwards
is slanting approximately 4 degrees with respect to the horizontal plane. Make sure that the angle is
the same all the time

When the LDM is turned on and mounted as explained above, the laser beam will form an angle 1
with respect to the vertical direction. This angle, which must be the same throughout this
experiment, must now be determined. The optical vessel is not needed here, so put it aside so far.
Measure with the LDM the distance 1 to the laser dot where the laser beam hits the
table top. Then move the box with the LDM horizontally until the laser beam hits the
1.3a
0.2
floor. Measure the distance 2 to the laser dot where the laser beam hits the floor. State
the uncertainties.
1.3b

Calculate the angle 1 using only these measurements 1 , 2 and (from problem 1.1).
0.4
Determine the uncertainty 1 .

Page 8 of 9

E1

Speed of light

E1

1.4 Experiment with the optical vessel


Place the optical vessel so that the laser beam hits the bottom of the vessel approximately in the
middle, see Fig. 1.10. Pour some water into the vessel. The depth of the water is . Read off on
the display of the LDM.
LDM

1 1
2

Figure 1.10 Diagram of laser beams in the optical vessel with water of depth .

1.4a

Measure corresponding values of and . Set up a table with your measurements. Draw
1.6
a graph of as a function of .

1.4b Use equations to explain theoretically what the graph is expected to look like.

1.2

1.4c Use the graph to determine the refractive index w for water.

1.2

Page 9 of 9

Speed of light (solution)

E1

In this document decimal comma is used instead of decimal point in graphs and tables

1.1

Use the LDM to measure the distance from the top of the table to the floor. Write
0.4
down the uncertainty
. Show with a sketch how you perform this measurement.

. See the sketch in the figure corresponding to 1.3b. It must appear how the
height is measured with the LDM in the rear mode.

1.2a

Measure corresponding values of and . Set up a table with your measurements. Draw
1.8
a graph showing as a function of .

Here, a 2 m cable is used, but 1 m is sufficient. There should be about 8 lengths evenly distributed
in the interval from 0 m to 1 m.
y
m
0,177
0,232
0,396
0,517
0,570
0,748
0,885
0,950
1,459
1,642

Display y as function of cable length x


y = 0,7710x + 0,1014
R = 0,9996
1,8
1,6
1,4
Display y [m]

x
m
0,103
0,176
0,348
0,546
0,617
0,839
1,025
1,107
1,750
2,000

1,2
1,0
0,8
0,6
0,4
0,2
0,0
0,0

0,5

1,0
cable length x [m]

Page 1 of 5

1,5

2,0

Speed of light (solution)

E1

Use the graph to find the refractive index


for the material from which the core of
1.2b the fiber optic cable is made. Calculate the speed of light
in the core of the fiber 1.2
optic cable.
The refractive index is twice the gradient of the linear graph,

The reason for that is that the travel time for a light pulse

The display will therefore show

The speed of light in the core of the cable is

Measure with the LDM the distance


to the laser dot where the laser beam hits the
table top. Then move the box with the LDM horizontally until the laser beam hits the
1.3a
0.2
floor. Measure the distance
to the laser dot where the laser beam hits the floor. State
the uncertainties.
,

1.3b

Calculate the angle using only these measurements


Determine the uncertainty
.

(
(
(see the figure)

and

(from problem 1.1).

0.4

Page 2 of 5

Speed of light (solution)

E1

Measuring the horizontal part of some triangle is very inaccurate because of the size of the laser dot.
No marks will be awarded for that. Using
as the uncertainty of ,
and , the
uncertainty of
can be calculated as follows:
(

Using simple derivatives yields

Otherwise, using min/max method


(

using (

Alternatively, calculate

(
(

( )
Also, accept

1.4a

and

Measure corresponding values of


a graph of as a function of .

x [mm]

y [mm]

450

17

454

27

457

32

459

39

461

51

466

58

467

66

471

76

473

82

476

90

478

96

480

)
(

and then

and . Set up a table with your measurements. Draw

Page 3 of 5

1.6

Speed of light (solution)

E1

1.4b Use equations to explain theoretically what the graph is expected to look like.
The time it takes the light to reach the water surface is
(

1.2

From the water surface to the bottom the light uses the time

Total travel time forth and back


(

Hence, the display will show (we simply write

)
(

which is a linear function of . Then, using a trigonometric identity and Snells law,

From this the gradient

is found to be

1.4c Use the graph to determine the refractive index


Knowing the gradient

1.2

for water.

from the graph, the index of refraction

is found by solving this equation.

Introducing a practical parameter,

the above equation becomes

with the solution

From the graph is found

, which leads to

and hence
.

All solutions with

are omitted.
Page 4 of 5

Speed of light (solution)

Another and more elegant way of finding

E1

is to use Snells law in the equation

This yields

From here
the angle

can be calculated leading to


, and measurement of

. This method also only uses the graph and

is not involved).

The table value for pure water at normal conditions is

at the wavelength

The following approximations can be used: For small angles

) )

For very small angles, we get


It is much simpler, but not recommendable, to do the experiment with very small
Reflections in the water surface will ruin the signal from the bottom.

Page 5 of 5

Solar cells

2.0 Introduction
Equipment used for this experiment is displayed in Fig. 2.1.

Figure 2.1 Equipment used for experiment E2.

List of equipment (see Fig. 2.1):


A: Solar cell
B: Solar cell
C: Box with slots for the mounting of light source, solar cells, etc.
D: LED-light source in holder
E: Power supply for light source D
F: Variable resistor
G: Holder for mounting single solar cell in the box C
H: Circular aperture for use in the box C
I: Holder for mounting two solar cells in the box C

Page 1 of 9

E2

Solar cells
J: Shielding plate for use in the box C
K: Digital multimeter
L: Digital multimeter
M: Wires with mini crocodile clips
N: Optical vessel (large cuvette)
O: Measuring tape
P: Scissors
Q: Tape
R: Water for filling the optical vessel N
S: Paper napkin for drying off excess water
T: Plastic cup for water from the optical vessel N (not shown in Fig. 2.1)
U: Plastic pipette (not shown in Fig. 2.1)
V: Lid for the box C (not shown in Fig. 2.1)

Data sheet: table of fundamental constants


Speed of light in vacuum
Elementary charge
Boltzmanns constant

A solar cell transforms part of the electromagnetic energy in the incident light to electric energy by
separating charges inside the solar cell. In this way an electric current can be generated. Experiment
E2 intents to examine solar cells with the use of the supplied equipment. This equipment consists of
a box with holders for light source and solar cells along with various plates and a lid. The variable
resistor should be mounted in the box, see Fig. 2.2. One of the three terminals on the resistor has
been removed, since only the two remaining terminals are to be used. Also supplied are wires with
mini crocodile clips and two solar cells (labeled with a serial number and the letter A or B) with
terminals on the back. The two solar cells are similar but can be slightly different. The two
multimeters have been equipped with terminals for designated use as ammeter and voltmeter,
respectively, see Fig. 2.3. Finally, the experiment will make use of an optical vessel together with
some drinking water from the bottle.

Page 2 of 9

E2

Solar cells

Figure 2.2 (a) Box with light source and resistor for mounting. (b) The resistor mounted in the box. Notice
that the small pin on the resistor fits in the hole to the right of the shaft.

Figure 2.3 Multimeters equipped with terminals for use as ammeter (left) and voltmeter (right),
respectively. The instrument is turned on by pressing POWER in the top left corner. The instrument turns
off automatically after a certain idle time. It can measure direct current and voltage
as well as alternating
current and voltage
. The internal resistance in the voltmeter is 10 M regardless of the measuring range.
The potential difference over the ammeter is 200 mV at full reading, regardless of the measuring range. In
case of overflow the display will show l, and you need to select a higher measuring range. The HOLD
button (top right corner) should not be pushed, except if you want to freeze a measurement.

Page 3 of 9

E2

Solar cells
WARNING: Do not use the multimeter as an ohmmeter on the solar cells since the measuring
current can damage them. When changing the measuring range on the multimeters, please turn
the dial with caution. It can be unstable and may break. Check whether there is a number under
the decimal point when measuring if the dial is not fully in place, the multimeter will not
measure, even if there are digits in the display.
Notice: Do not change the voltage on the power supply. It must be 12 V throughout the
experiment. (The power supply for the light source should be connected to the outlet (230 V ~) at
your table.)
Notice: Uncertainty considerations are only expected when explicitly mentioned.
Notice: All measured and calculated values must be given in SI units.
Notice: For all measurements of currents and voltages in this experiment, the LED-light
source is supposed to be on.

Page 4 of 9

E2

Solar cells

E2

2.1 The dependence of the solar cell current on the distance to the light source
For this question you will measure the current, , generated by the solar cell when in a circuit with
the ammeter, and determine how it depends on the distance, , to the light source. The light is
produced inside the individual light diodes and is therefore to be measured as shown in Fig. 2.4.

Figure 2.4 Top view of setup for question 2.1. Note the aperture a immediately in front of the solar cell A.
The distance is measured from inside the light diode to the surface of the solar cell.

Do not change the measuring range on the ammeter in this experiment: the internal resistance of the
ammeter depends on the measuring range and affects the current that can be drawn from the solar
cell. State the serial numbers of the light source and of solar cell A on your answer sheet. Mount the
light source in the U-shaped holder (the light source has a tight fit in the holder, so be patient when
mounting it. Mount solar cell A in the single holder and place it together with the circular aperture
immediately in front of the solar cell. The current as a function of the distance to the light source
can, when is not too small, be approximated by

where

and

are constants.

2.1a Measure I as a function of r, and set up a table of your measurements.

1.0

2.1b Determine the values of Ia and a by the use of a suitable graphical method.

1.0

Page 5 of 9

Solar cells

E2

2.2 Characteristic of the solar cell


Remove the circular aperture. Mount the variable resistor in the box as shown on Fig. 2.2. Place the
light source in slot number 0, furthest away from the resistor. Mount solar cell A in the single
holder without the circular aperture in slot number 10. Build a circuit as shown in Fig. 2.5, so that
you can measure the characteristic of the solar cell, i.e. the terminal voltage U of the solar cell as a
function of the current I in the circuit consisting of solar cell, resistor and ammeter.

Figure 2.5 Electrical diagram for measuring the characteristic in question 2.2.

2.2a Make a table of corresponding measurements of U and I.

0.6

2.2b Graph voltage as function of current

0.8

2.3 Theoretical characteristic for the solar cell


For the solar cells in this experiment, the current as function of the voltage is given by the equation
(

where the parameters


, and are constant at a given illumination. We take the temperature to
be
. The fundamental constants and
are the elementary charge and Boltzmanns
constant, respectively.
2.3a Use the graph from question 2.2b to determine

0.4

The parameter can be assumed to lie in the interval from 1 to 4. For some values of the potential
difference , the formula can be approximated by
(
2.3b

Estimate the range of values of


for which the mentioned approximation is good.
1.2
Determine graphically the values of and for your solar cell.

Page 6 of 9

Solar cells

E2

2.4 Maximum power for a solar cell


2.4a

The maximum power that the solar cell can deliver to the external circuit is denoted
. Determine
for your solar cell through a few, suitable measurements. (You 0.5
may use some of your previous measurements from question 2.2).

Estimate the optimal load resistance


, i.e. the total external resistance when the
2.4b solar cell delivers its maximum power to
. State your result with uncertainty and 0.5
illustrate your method with suitable calculations.

2.5 Comparing the solar cells


Mount both solar cells (A and B) in the double holder in slot number 15, see Fig. 2.6.

Figure 2.6 Top view of light source and solar cells in question 2.5.

Measure, for the given illumination:


- The maximum potential difference that can be measured over solar cell A.
2.5a
- The maximum current that can be measured through solar cell A.
Do the same for solar cell B.
2.5b

0.5

Draw electrical diagrams for your circuits showing the wiring of the solar cells and the
0.3
meters.

Page 7 of 9

Solar cells
2.6 Couplings of the solar cells
The two solar cells can be connected in series in two different ways as shown in Fig. 2.7. There are
also two different ways to connect them in parallel (not shown in the figure).

Figure 2.7 Two ways to connect the solar cells in series for question 2.6. The two ways to connect them in
parallel are not shown.

Determine which of the four arrangements of the two solar cells yields the highest
possible power in the external circuit when one of the solar cells is shielded with the
shielding plate (J in Fig. 2.1). Hint: You can estimate the maximum power quite well by
2.6
1.0
calculating it from the maximum voltage and maximum current measured from each
configuration.
Draw the corresponding electrical diagram.

2.7 The effect of the optical vessel (large cuvette) on the solar cell current
Mount the light source in the box and place solar cell A in the single holder with the circular
aperture immediately in front, so that there is approximately 50 mm between the solar cell and the
light source. Place the empty optical vessel immediately in front of the circular aperture as shown in
Fig. 2.8.

Page 8 of 9

E2

Solar cells

E2

Figure 2.8 Experimental set-up for question 2.7.

2.7a

Measure the current I, now as a function of the height, h, of water in the vessel, see Fig.
1.0
2.8. Make a table of the measurements and draw a graph.

2.7b Explain with only sketches and symbols why the graph looks the way it does.

1.0

Mount the light source in the box and place solar cell A in the single holder so that the distance
between the solar cell and the light source is maximal. Place the circular aperture immediately in
front of the solar cell.

For this set-up do the following:


- Measure the distance between the light source and the solar cell and the current .
- Place the empty vessel immediately in front of the circular aperture and measure the
2.7c
0.6
current .
- Fill up the vessel with water, almost to the top, and measure the current .
Use your measurements from 2.7c to find a value for the refractive index
for water.
2.7d Illustrate your method with suitable sketches and equations. You may include additional 1.6
measurements.

Page 9 of 9

Solar cells (solution)

E2

2.1 The dependence of the solar cell current on the distance to the light source
( )

2.1a Measure I as a function of r, and set up a table of your measurements.

1.0

2.1b Determine the values of Ia and a by the use of a suitable graphical method.

1.0

slot #
3
4
5
6
7
8
9
10
11
12
13
14
15
16
17
18
19
20

r
mm
9.0
14.5
20.0
25.5
31.0
36.5
42.0
47.5
53.0
58.5
64.0
69.5
75.0
80.5
86.0
91.5
97.0
102.5

I
mA
5.440
5.290
5.010
4.540
3.840
3.230
2.730
2.305
1.985
1.730
1.485
1.305
1.140
1.045
0.930
0.840
0.755
0.690

1/I
1/mA
0.184
0.189
0.200
0.220
0.260
0.310
0.366
0.434
0.504
0.578
0.673
0.766
0.877
0.957
1.075
1.190
1.325
1.449

r^2
mm^2
81
210
400
650
961
1332
1764
2256
2809
3422
4096
4830
5625
6480
7396
8372
9409
10506

)
(

Page 1 of 12

Solar cells (solution)

E2

2.2 Characteristic of the solar cell


2.2a Make a table of corresponding measurements of U and I.

0.6

2.2b Graph voltage as a function of current

0.8

I
mA
0.496
1.451
5.05
8.88
14.05
31.1
25.3
21.6
30.6
31.9
32.6
32.6
33.1
33.4
33.3
33.4
33.4
33.5
33.5
1.05
27.8
15.9
22.3
26.8
29.2

U
V
0.532
0.531
0.526
0.52
0.509
0.395
0.471
0.488
0.41
0.364
0.299
0.313
0.239
0.085
0.138
0.096
0.058
0.046
0.045
0.529
0.454
0.503
0.483
0.458
0.435

Page 2 of 12

Solar cells (solution)

E2

2.3 Theoretical characteristic for the solar cell


2.3a Use the graph from question 2.2b to determine
2.3b

0.4

Estimate the range of values of U for which the mentioned approximation is good.
1.2
Determine graphically the values of and for your solar cell.

for

(
for

)
(

where

)
(

Page 3 of 12

( )
)

Solar cells (solution)

E2

2.4 Maximum power for a solar cell


2.4a

The maximum power that the solar cell can deliver to the external circuit is denoted
. Determine
for your solar cell through a few, suitable measurements. (You 0.5
may use some of your previous measurements from question 2.2)

Estimate the optimal load resistance


, i.e. the total external resistance when the
2.4b solar cell delivers its maximum power to
. State your result with uncertainty and 0.5
illustrate your method with suitable calculations.

I
mA
26.8
27.8
29.2
30.6
31.1

Ropt

Pmax 12.71mW

(15.3 0.3)
2
I opt
(28.8mA)2

Page 4 of 12

U
V
0.458
0.454
0.435
0.410
0.395

P
mW
12.2744
12.6212
12.7020
12.5460
12.2845

1
2
3
4
5

Solar cells (solution)

E2

2.5 Comparing the solar cells


Measure, for the given illumination:
- The maximum potential difference that can be measured over solar cell A.
2.5a
- The maximum current that can be measured through solar cell A.
Do the same for solar cell B.
2.5b

0.5

Draw electrical diagrams for your circuits showing the wiring of the solar cells and the
0.3
meters.

2.5a. UA=0.512 V

IA=16.465 mA

UB=0.480 V

2.5b.

Page 5 of 12

IB = 16.325 mA

Solar cells (solution)


2.6 Couplings of the solar cells
Determine which of the four arrangements of the two solar cells yields the highest
possible power in the external circuit when one of the solar cells is shielded with the
2.6 shielding plate (J in Fig. 2.1).
1.0
Draw the corresponding electrical diagram.
Two approaches:
Approach 1: use a constant setting of the variable resistor to simulate a constant external load.
Approach 2: use the hint given in the question and measure values of maximal U and maximal I
independently (no variable resistor involved).
In the following only measurements for approach 1 are presented.
a.
Unshielded (adjusting R for reasonable P)
13.10 mA; 0.794 V; 10.4 mW
A shielded: 0.37 mA; 0.022 V
B shielded: 0.83 mA; 0.049 V
b.
R like in a.
A shielded: 1.47 mA; 0.088 V
B shielded: -2.82 mA; -0.170 V

c.
R like in a.
A shielded: 6.89 mA; 0.415 V
B shielded: 6.905 mA; 0.4165 V

Page 6 of 12

E2

Solar cells (solution)


d.
R like in a.

A shielded: 7.14 mA; 0.436 V


B shielded: -7.76 mA; -0.474 V
Conclusion: Best power: Set-up d with B shielded. (Solar cell A slightly better than B).

(2.7 on next page)

Page 7 of 12

E2

Solar cells (solution)

E2

2.7 The effect of the optical vessel (large cuvette) on the solar cell current
2.7a

Measure the current I, now as a function of the height, h, of water in the vessel, see Fig.
1.0
2.8. Make a table of the measurements and draw a graph.

2.7b Explain with only sketches and symbols why the graph looks the way it does.

1.0

For this set-up do the following:


- Measure the distance between the light source and the solar cell, and the current .
2.7c - Place the empty vessel immediately in front of the circular aperture and measure the 0.6
current .
- Fill up the vessel with water, almost to the top, and measure the current .
Use your measurements from 2.7c to find a value for the refractive index
for water.
2.7d Illustrate your method with suitable sketches and equations. You may include additional 1.6
measurements.
2.7a
h
mm
2
22
28
34
38
42
45
46
48
49
50
52
53
57
59
63
67
69
72

I
mA
2.54
2.55
2.56
2.57
2.42
2.21
2.13
2.08
2.15
2.54
2.97
3.36
3.61
3.96
3.99
3.89
3.6
3.49
3.47

C
A

D
B

aperture

Page 8 of 12

Solar cells (solution)

E2

2.7b Exemple drawings for position A, B, C and D on previous graph:

mA

mA

Page 9 of 12

Solar cells (solution)

E2

mA

mA

Page 10 of 12

Solar cells (solution)


2.7c NOTE: The exemplar measurements are from a different lamp than in 2.1. For a
solution to 2.7d using the distance graph it is necessary to refer to the graph below.
;
;
;

Page 11 of 12

E2

Solar cells (solution)


2.7d

h (b r ) tan 1 b tan 2

tan 1 sin 1
b

n , da 2 1 1.
b r tan 2 sin 2

NOTE: Better results may be obtained. The uncertainty is rather large in this method because of the
subtraction of two large numbers for
A different method is to determine the shift by actually moving the set-up and perhaps making an
interpolation in directly measured data.

Page 12 of 12

E2

Theoretical competition. Tuesday, 15 July 2014

1/1

Problem 1 (9 points)
This problem consists of three independent parts.

Part A (3 points)
A small puck of mass is carefully placed onto the inner surface of the
thin hollow thin cylinder of mass and of radius . Initially, the cylinder
rests on the horizontal plane and the puck is located at the height above
the plane as shown in the figure on the left. Find the interaction force
between the puck and the cylinder at the moment when the puck passes
the lowest point of its trajectory. Assume that the friction between the
puck and the inner surface of the cylinder is absent, and the cylinder
moves on the plane without slipping. The free fall acceleration is .

Part B (3 points)
A bubble of radius = 5.00 cm, containing a diatomic ideal gas, has the soap film of thickness =
N
10.0 m and is placed in vacuum. The soap film has the surface tension = 4.00 102 m and the density
g

= 1.10 3 . 1) Find formula for the molar heat capacity of the gas in the bubble for such a process when
cm
the gas is heated so slowly that the bubble remains in a mechanical equilibrium and evaluate it; 2) Find
formula for the frequency of the small radial oscillations of the bubble and evaluate it under the
assumption that the heat capacity of the soap film is much greater than the heat capacity of the gas in the
bubble. Assume that the thermal equilibrium inside the bubble is reached much faster than the period of
oscillations.
Hint: Laplace showed that there is pressure difference between inside and outside of a curved
2
surface, caused by surface tension of the interface between liquid and gas, so that = .

Part C (3 points)
Initially, a switch is unshorted in the circuit shown in the figure on the right, a
capacitor of capacitance 2 carries the electric charge 0 , a capacitor of
capacitance is uncharged, and there are no electric currents in both coils of
inductance and 2, respectively. The capacitor starts to discharge and at the
moment when the current in the coils reaches its maximum value, the switch is
instantly shorted. Find the maximum current max through the switch thereafter.

Theoretical competition. Tuesday, 15 July 2014

1/4

Problem 1
Solution
Part A
Consider the forces acting on the puck and the cylinder and
depicted in the figure on the right. The puck is subject to the
gravity force and the reaction force from the cylinder . The
cylinder is subject to the gravity force , the reaction force from
the plane 1 , the friction force and the pressure force from the
puck = . The idea is to write the horizontal projections of
the equations of motion. It is written for the puck as follows
= sin ,
(A.1)
where is the horizontal projection of the puck acceleration.
For the cylinder the equation of motion with the
acceleration is found as
= sin .
(A.2)
Since the cylinder moves along the plane without sliding its
angular acceleration is obtained as
= /
(A.3)
Then the equation of rotational motion around the center of mass of the cylinder takes the form
= ,
(A.4)
where the inertia moment of the hollow cylinder is given by
= 2 .
(A.5)
Solving (A.2)-(A.5) yields
2 = sin .
(A.6)
From equations (A.1) and (A.6) it is easily concluded that
= 2.
(A.7)
Since the initial velocities of the puck and of the cylinder are both equal to zero, then, it follows from
(A.7) after integrating that
= 2.
(A.8)
It is obvious that the conservation law for the system is written as
2

= 2 + 2 + 2 ,
(A.9)
where the angular velocity of the cylinder is found to be

= ,
(A.10)
since it does not slide over the plane.
Solving (A.8)-(A.10) results in velocities at the lowest point of the puck trajectory written as
=2

(A.12)

(2+ )

(2+ )

(A.13)

In the reference frame sliding progressively along with the cylinder axis, the puck moves in a circle
of radius and, at the lowest point of its trajectory, have the velocity
= +
(A.14)
and the acceleration
2

rel = rel
.
(A.15)

At the lowest point of the puck trajectory the acceleration of the cylinder axis is equal to zero,
therefore, the puck acceleration in the laboratory reference frame is also given by (A.15).
2

= .
then the interaction force between the puck and the cylinder is finally found as

= 3 1 + 3 .

(A.16)
(A.17)

Theoretical competition. Tuesday, 15 July 2014

2/4

Part B
1) According to the first law of thermodynamics, the amount of heat transmitted to the gas in the
bubble is found as
= + ,
(B.1)
where the molar heat capacity at arbitrary process is as follows
1

= = + .
(B.2)
Here stands for the molar heat capacity of the gas at constant volume, designates its pressure, is the
total amount of moles of gas in the bubble, and denote the volume and temperature of the gas,
respectively.
Evaluate the derivative standing on the right hand side of (B.2). According to the Laplace formula,
the gas pressure inside the bubble is defined by
4
= ,
(B.3)
thus, the equation of any equilibrium process with the gas in the bubble is a polytrope of the form
3 = const.
(B.4)
The equation of state of an ideal gas has the form
= ,
(B.5)
and hence equation (B.4) can be rewritten as
3 2 = const.
(B.6)
Differentiating (B.6) the derivative with respect to temperature sought is found as

3
= 2 .
(B.7)

Taking into account that the molar heat capacity of a diatomic gas at constant volume is
5
= 2 ,
(B.8)
and using (B.5) it is finally obtained that
3
J
= + 2 = 4 = 33.2 mole K .
(B.9)
2) Since the heat capacity of the gas is much smaller than the heat capacity of the soap film, and
there is heat exchange between them, the gas can be considered as isothermal since the soap film plays the
role of thermostat. Consider the fragment of soap film, limited by the angle as shown in the figure. It's
area is found as
= ()2 .
(B.10)
and the corresponding mass is obtained as
= .
(B.11)
Let be an increase in the radius of the bubble, then the
Newton second law for the fragment of the soap film mentioned
above takes the form
= ,
(B.12)
where denotes the projection of the resultant surface tension
force acting in the radial direction, stands for the gas pressure
beneath the surface of the soap film and

= 1 + 2 .
is easily found as
= = 2 2[ + ] . (B.13)
Since the gaseous process can be considered isothermal, it is
written that
= .
(B.14)
Assuming that the volume increase is quite small, (B.14) yields
1
1
3
= 3 3 1 .
(B.15)
1+

1+

Thus, from (B.10) - (B.16) and (B.3) the equation of small oscillations of the soap film is derived as
8
= 2
(B.16)

Theoretical competition. Tuesday, 15 July 2014

3/4

with the frequency


8

= 108 s 1 .

(B.17)

Part C
The problem can be solved in different ways. Herein several possible solutions are considered.
Method 1. Direct approach
At the moment when the current in the coils is a maximum, the total voltage across the coils is equal
to zero, so the capacitor voltages must be equal in magnitude and opposite in polarity. Let be a voltage on
the capacitors at the time moment just mentioned and 0 be that maximum current. According to the law of
charge conservation
0 = 2 + ,
(C1.1)
thus,

= 30 .
(C1.2)
Then, from the energy conservation law
02

= 20 +
22
the maximum current is found as
0
0 = 3 2
.

202
2

2
2

2 2
2

(C1.3)
(C1.4)

After the key is shortened there will be independent oscillations in both circuits with the frequency
1
= 2 ,
(C1.5)
and their amplitudes are obtained from the corresponding energy conservation laws written as
2 2
2
2

02

2
202

12

2
2 22

(C1.6)

+ 2 = 2 .
(C1.7)
Hence, the corresponding amplitudes are found as
1 = 50 ,
(C1.8)
2 = 20 .
(C1.9)
Choose the positive directions of the currents in the circuits as shown in the
figure on the right. Then, the current flowing through the key is written as follows
= 1 2 .
(C1.10)
The currents depend on time as
1 = cos + sin ,
(C1.11)
2 = cos + sin ,
(C1.12)
The constants , , , can be determined from the initial values of the currents and their
amplitudes by putting down the following set of equations
1 0 = = 0 ,
(C1.13)
2
2
2
+ = 1 ,
(C1.14)
2 0 = = 0 ,
(C1.15)
2
2
2
+ = 2 .
(C1.16)
Solving (C1.13)-(C1.16) it is found that
= 20 ,
(C1.17)
= 0 ,
(C1.18)
The sign in is chosen negative, since at the time moment of the key shortening the current in the coil 2
decreases.
Thus, the dependence of the currents on time takes the following form
1 = 0 (cos + 2 sin ),
(C1.19)
2 = 0 (cos sin ).
(C1.20)
In accordance with (C.10), the current in the key is dependent on time according to
= 1 2 = 30 sin .
(C1.21)
Hence, the amplitude of the current in the key is obtained as
0
max = 30 = 0 = 2
.
(C1.22)
2

Theoretical competition. Tuesday, 15 July 2014


Method 2. Vector diagram
Instead of determining the coefficients , , , the vector diagram shown
in the figure on the right can be used. The segment represents the current sought
and its projection on the current axis is zero at the time of the key shortening. The
current 1 in the coil of inductance grows at the same time moment because the
capacitor 2 continues to discharge, thus, this current is depicted in the figure by
the segment . The current 2 in the coil of inductance 2 decreases at the time of
the key shortening since it continues to charge the capacitor 2, that is why this
current is depicted in the figure by the segment .
It is known for above that = 0 , = 50 , = 20 . Hence, it is
found from the Pythagorean theorem that
= 2 2 = 20 ,
(C2.1)
2
2
= = 0 ,
(C2.2)
Thus, the current sought is found as
0
max = = + = 30 = 0 = 2
.

4/4

(C2.3)

Method 3. Heuristic approach


It is clear that the current through the key performs harmonic oscillations with the frequency
1
= 2 .
(C3.1)
and it is equal to zero at the time of the key shortening, i.e.
= max sin .
(C3.2)
Since the current is equal to zero at the time of the key shortening, then the current amplitude is equal
to the current derivative at this time moment divided by the oscillation frequency. Let us find that current
derivative. Let the capacitor of capacitance 2 have the charge 1 . Then the charge on the capacitor of
capacitance is found from the charge conservation law as
2 = 0 1 .
(C3.3)
After shortening the key the rate of current change in the coil of inductance is obtained as
1
1 = 2
,
(C3.4)
whereas in the coil of inductance 2 it is equal to

2 = 02 1 .
(C3.5)
Since the voltage polarity on the capacitors are opposite, then the current derivative with respect to
time finally takes the form
0
= 1 2 = 2
= 2 0 .
(C3.6)
Note that this derivative is independent of the time of the key shortening!
Hence, the maximum current is found as

0
max = = 0 = 2
,
and it is independent of the time of the key shortening!

(C3.7)

Theoretical competition. Tuesday, 15 July 2014

1/3

Problem 2. Van der Waals equation of state (11 points)


In a well-known model of an ideal gas, whose equation of state obeys the Clapeyron-Mendeleev law,
the following important physical effects are neglected. First, molecules of a real gas have a finite size and,
secondly, they interact with one another. In all parts of this problem one mole of water is considered.

Part . Non-ideal gas equation of state (2 points)


Taking into account the finite size of the molecules, the gaseous equation of state takes the form
= ,
(1)
where , , stands for the gas pressure, its volume per mole and temperature, respectively, denotes the
universal gas constant, and is a specific constant extracting some volume.
A1

Estimate and express it in terms of the diameter of the molecules . (0.3 points)

With account of intermolecular attraction forces, van der Waals proposed the following equation of
state that neatly describes both the gaseous and liquid states of matter

+ 2 = .
(2)

where is another specific constant.


At temperatures below a certain critical value the isotherm of equation (2) is well represented
by a non-monotonic curve 1 shown in Figure 1 which is then called van der Waals isotherm. In the same
figure curve 2 shows the isotherm of an ideal gas at the same temperature. A real isotherm differs from the
van der Waals isotherm by a straight segment drawn at some constant pressure . This straight
segment is located between the volumes and , and corresponds to the equilibrium of the liquid phase
(indicated by ) and the gaseous phase (referred to by ). From the second law of thermodynamics
J. Maxwell showed that the pressure must be chosen such that the areas I and II shown in Figure 1 must
be equal.

Figure 1. Van der Waals isotherm of gas/liquid Figure 2. Several isotherms for van der Waals
(curve 1) and the isotherm of an ideal gas (curve 2).
equation of state.
With increasing temperature the straight segment on the isotherm shrinks to a single point when
the temperature and the pressure reaches some values and = , respectively. The parameters and
are called critical and can be measured experimentally with high degree of accuracy.
A2
A3
A4

Express the van der Waals constants and in terms of and . (1.3 points)
For water = 647 K and = 2.2 107 Pa. Calculate and for water. (0.2 points)
Estimate the diameter of water molecules . (0.2 points)

Theoretical competition. Tuesday, 15 July 2014

2/3

Part B. Properties of gas and liquid (6 points)


This part of the problem deals with the properties of water in the gaseous and liquid states at
temperature = 100 C. The saturated vapor pressure at this temperature is known to be = 0 = 1.0
kg
105 Pa, and the molar mass of water is = 1.8 102 mole.
Gaseous state
It is reasonable to assume that the inequality is valid for the description of water properties in
a gaseous state.
B1

Derive the formula for the volume and express it in terms of , , 0 , and . (0.8 points)
Almost the same volume 0 can be approximately evaluated using the ideal gas law.

B2

Evaluate in percentage the relative decrease in the gas volume due to intermolecular forces,


= 0 . (0.3 points)

If the system volume is reduced below , the gas starts to condense. However, thoroughly purified
gas can remain in a mechanically metastable state (called supercooled vapor) until its volume reaches a
certain value min.
The condition of mechanical stability of supercooled gas at constant temperature is written as:
0.

<

Find and evaluate how many times the volume of water vapor can be reduced and still remains in a
metastable state. In other words, what is /min ? (0.7 points)
Liquid state

For the van der Waals description of water in a liquid state it is reasonable to assume that the
following inequality holds / 2 .
B4

Express the volume of liquid water in terms of , , , and . (1.0 points)

Assuming that , find the following characteristics of water. Do not be surprised if some of
the data evaluated do not coincide with the well-known tabulated values!
B5
B6
B7
B8

Express the liquid water density in some of the terms of , , , and evaluate it. (0.5 points)
1

Express the volume thermal expansion coefficient =

in terms of , , , and evaluate it.

(0.6 points)
Express the specific heat of water vaporization in terms of , , , and evaluate it. (1.1 points)
Considering the monomolecular layer of water, estimate the surface tension of water. (1.2 points)

Part . Liquid-gas system (3 points)


From Maxwells rule (equalities of areas, by applying trivial integration) and the van der Waals
equation of state together with the approximations made in Part B, it can be shown that the saturated vapor
pressure depends on temperature as follows

ln = + ,
(3)
where and are some constants, that can be expressed in terms of and as = ln

1 ; =

Theoretical competition. Tuesday, 15 July 2014

3/3

W. Thomson showed that the pressure of saturated vapor depends on the


curvature of the liquid surface. Consider a liquid that does not wet the material
of a capillary (contact angle 180). When the capillary is immersed into the
liquid, the liquid in the capillary drops to a certain level because of the surface
tension (see Figure 3).
1 Find a small change in pressure of the saturated vapor over the
curved surface of liquid and express it in terms of the vapor density ,
the liquid density , the surface tension and the radius of surface
curvature . (1.3 points)
Figure 3. Capillary
Metastable states, considered in part B3, are widely used in real
experimental setups, such as the cloud chamber designed for registration of immersed in a liquid that
does not wet its material
elementary particles. They also occur in natural phenomena such as the
formation of morning dew. Supercooled vapor is subject to condensation by forming liquid droplets. Very
small droplets evaporate quickly but large enough ones can still grow.
C2

Suppose that at the evening temperature of = 20 C the water vapor in the air was saturated, but
in the morning the ambient temperature has fallen by a small amount of = 5.0 C. Assuming that
the vapor pressure has remained unchanged, estimate the minimum radius of droplets that can grow.
Use the tabulated value of water surface tension = 7.3 102 . (1.7 points)

Theoretical competition. Tuesday, 15 July 2014

1/3

Problem 2.Van der Waals equation of state


Solution
Part . Non-ideal gas equation of state
A1.If = is substituted into the equation of state, then the gas pressure turns infinite. It is obvious that this
is the moment when all the molecules are tightly packed. Therefore, the parameter is approximately equal
to the volume of all molecules, i.e.
= 3
(A1.1)
A2.In the most general case thevan der Waals equation of state can be rewritten as
3 + 2 + = 0
(A2.1).
Since at the critical values of the gas parameters the straight line disappears, then, the solution of
(A2.1) must have one real triple root, i.e. it can be rewritten as follows
( )3 = 0
(A2.2).
Comparing the coefficients of expression (A2.1) and (A2.2), the following set of equations is
obtained
3 = +
3 2 =
(A2.3).
3
=
Solution to the set (A2.3) is the following formulas for the van der Waals coefficients
=

27 2 2

(A2.4),

64

= 8

(A2.5).

Alternative solution
The critical parameters are achieved in the presence of an inflection point in the isotherm, at which
the first and second derivatives are both zero. Therefore, they are defined by thefollowingconditions

=0
(A2.6),

and
2

=0

(A2.7).

Thus, the following set of equations is obtained

2
2 + 3 = 0

2
3

6
4

=0

(A2.8),

which has the same solution (A2.4) and (A2.5).


A3.Numericalcalculationsforwaterproduce the following result
= 0.56

m6 Pa

(A3.1).

mole2
m3
3.1 105 mole

=
A4.From equations (A1.4) and (A3.2) it is found that
=

= 3.7 1010 m 4 1010 m

(A3.2).
(A4.1).

Part B. Properties of gas and liquid


B1.Usingtheinequality , the van der Waals equation of state can be written as
0 +

(B1.1),

which has the following solutions

= 2

1 1

40
2 2

(B1.2).

Theoretical competition. Tuesday, 15 July 2014

2/3

Smaller root in (B1.2) gives the volume in an unstable state on the rising branch of thevan der Waals
isotherm. The volume of gas is given by the larger root, since at = 0an expression for the volume of an
ideal gasshould be obtained, i.e.

= 2

1+ 1

40

(B1.3).

2 2

For given values of the parameters the value 02 = 5.8 103 . It can therefore be assumed

that 02 1, then (B1.3)takes the form

2 2

(B1.4).

B2. For an ideal gas

0 =

(B2.1),

hence,

0
0

=2 1 1

40
2 2

0
2 2

= 0.58%.

B3.Mechanical stability of a thermodynamic system is inpower provided that

< 0.

(B2.2)
(B3.1)

The minimum volume, in which the mattercan still exist in the gaseous state, corresponds to a point
in which

= 0
(B3.2).

Using the van der Waals equation of state (B3.2) is written as

2
=
2+ 3 = 0

()

(B3.3).

From (B3.2) and (B3.3), and with the help of , it is found that
2
=
Thus,

2 2

(B3.4).

= 2 = 86

(B3.5).

B4. Usingtheinequality / , the van der Waals equation of state is written as

2 = ,

(B4.1)

whose solution is

= 2 1 1

(B4.2).

In this case, the smaller root shouldbe taken, since at 0the liquid volume = must be obtained
according to (B4.1), i.e.

= 2 1 1

1+

(B4.3).

B5. Since (B4.3) givesthevolumeoftheonemoleofwaterits mass density is easily found as

2 kg
= =
= 5.8 10 m3

(B5.1).

B6. Inaccordancewith (B4.3) the volume thermal expansion coefficient is derived as


1

= = + = 4.6 104 1

(B6.1).

1+

B7.The heat, required to convert the liquid to gas, is used to overcome the intermolecular forces that create
negative pressure / 2 , therefore,

1
1
= 2 =
(B7.1),

and using , (B7.1) yields

= =

1+

= 1.0 106 kg

(B7.2).

Theoretical competition. Tuesday, 15 July 2014

3/3

B8.Consider some water of volume. To make a monolayer of thickness out of it, the following work
must be done
= 2
(B8.1).
Fabrication of the monomolecular layer may be interpreted as the evaporation of an equivalent
volume of water which requires the following amount of heat
=
(B8.2),
where the mass is given by
=
(B8.3).
Using (A4.1a), (B5.1)and(B7.2), one finally gets

N
= 2 2 = 0.12 102 m
(B8.4).

Part . Liquid-gas systems


C1.At equilibrium, the pressure in the liquid and gas should be equalat all depths. The pressurein the fluid
at the depth is related to the pressure of saturated vapor above the flat surface by
= 0 +
(C1.1).
The surface tension creates additional pressure defined by the Laplace formula as
2
=
(C1.2).
The same pressureinthefluidatthedepth depends on the vapor pressure over the curved liquid
surface and its radiusofcurvature as
2
= +
(C1.3).
Furthermore, the vapor pressure at different heights are related by
= 0 +
(C1.4).
Solving (C1.1)-(C1.4), it is found that
2
= ( )
(C1.5).

Hence,the pressure difference sought is obtained as


2
2
= 0 = =
.

(C1.6).

Note that the vapor pressure over the convex surface of the liquid is larger than the pressure above
the flat surface.
C2.Let be vapor pressure at a temperature , and be vapor pressure at a temperature .
In accordance with equation (3) from problem statement, whentheambient temperature falls by an amount of
the saturated vapor pressure changes by an amount

= 2
(C2.1).

In accordance with the Thomson formula obtained in part C1, the pressure of saturated vapor above
the droplet increases by the amountof . While a droplet is small in size, the vapor above its surface
remains unsaturated. Whena droplet hasgrownuptoacertainminimumsize, thevaporaboveitssurface turns
saturated.
Since the pressure remains unchanged, the following condition must hold
+ =
(C2.2).
Assuming the vapor is almost ideal gas, its density can be found as

=
(C2.3).

From equations (C2.1)-(C2.3), (B5.1) and (C1.6) one finds


2

=
2

(C2.4).

Thus, it is finally obtained that


=

2 2

= 1.5 108 m

(C2.5).

Theoretical competition. Tuesday, 15 July 2014

1/2

Problem3. Simplest model of gas discharge (10 points)


An electric current flowing through a gas is called a gas discharge. There are many types of gas
discharges including glow discharge in lighting lamps, arc discharge inwelding and the well known spark
discharge that occurs between the clouds and the earth in the form of lightning.

Part . Non-self-sustained gas discharge (4.8points)


In this part of the problem the so-called non-self-sustained gas discharge is studied. To maintain it a
permanent operationan external ionizer is needed, which creates ext pairs of singly ionized ions and free
electrons per unit volume and per unit time uniformly in the volume.
When an external ionizer is switched on, the number of electrons and ions starts to grow. Unlimited
increase in the number densities of electrons and ions in the gas is prevented by the recombination process in
which a free electron recombineswith an ion to form a neutral atom. The number of recombining
eventsrec that occurs in the gas per unit volume and per unit time is given by
rec = ,
where is a constant called the recombination coefficient, and , denotethe electron and ion number
densities, respectively.
Suppose that at time = 0the external ionizer is switched on and the initial number densities of
electrons and ions in the gas are both equal to zero. Then, the electron number density () depends on
timeas follows:
= 0 + tanh ,
where 0 , and are some constants, and tanh stands for the hyperbolic tangent.
A1

Find0 , , and express them in terms ofext and.(1.8points)

Assume that there are two external ionizers available. When the first one is switched on, the electron
number density in the gas reaches its equilibrium value of be 1 = 12 1010 cm3 . When the second
external ionizer is switched on, the electron number density reaches its equilibrium value of 2 = 16
1010 cm3 .
A2

Find the electron number density at equilibriumwhen both external ionizers are switched on
simultaneously.(0.6 points)

Attention!In what follows it is assumed that the external ionizer is switched on for quite long period
of time such that all processes have become stationary and do not depend on time. Completely neglect the
electric field due the charge carriers.
Assume that the gas fills in the tube between the two parallel conductive plates of area separated by
the distance from each other. The voltage is applied across the plates to create an electric field
between them. Assume that the number densities of both kinds of charge carriers remain almost constant
along the tube.
Assumethat both the electrons (denoted by thesubscript) and the ions (denoted by the subscript)
acquire the same ordered speeddue to the electric field strength found as
= ,
where isa constant called charge mobility.
A3
A4

Express the electric currentin the tube in terms of , , , , ext , and whichis the elementary
charge.(1.7 points)
Find the resistivitygas of the gas at sufficiently small values of the voltage applied and express it in
terms of, , ext , and .(0.7 points)

Theoretical competition. Tuesday, 15 July 2014

2/2

PartB. Self-sustained gas discharge (5.2 points)


In this part of the problemthe ignition of the self-sustained gas discharge is consideredto show how
the electric current in the tube becomes self-maintaining.
Attention!In the sequel assume that the external ionizer continues to operatewith the same ext rate,
neglect the electric field due to the charge carriers such that the electric field is uniform along the tube, and
the recombination can be completely ignored.
For the self-sustained gas discharge there are two
important processes not considered above. The first process
isasecondary electron emission, and the second one isa
formation of electron avalanche. The secondary electron
emission occurs when ions hit on the negative electrode,
called a cathode, and the electrons are knocked out of it to
move towards the positive electrode, called an anode. The
ratio of the number of the knocked electrons per unit
timeto the number of ions hitting the cathode per unit
time is called the coefficient of the secondary electron
emission, = / .The formation of the electron
avalanche isexplained as follows. The electric field accelerates free electrons which acquire enough kinetic
energy to ionize the atoms in the gas by hitting them. As a result the number of free electrons moving
towards the anodesignificantly increases. This process is described by the Townsend coefficient , which
characterizes an increase in the number of electrons due to moving electrons that have passed the
distance, i.e.

= .

The total current in any cross section of the gas tube consists of the ion ()and the electron ()
currents which, in the steady state, depend on the coordinate , shown in the figureabove. The electron
current () varies along the -axis according to the formula
= 1 1 + 2 ,
where 1 , 2 , 1 are some constants.
B1

Find1 , 2 and express them in terms ofext , , , , .(2 points)

The ion current () varies along the -axis according to the formula
= 2 + 1 2 ,
where 1 , 2 , 2 are some constants.
B2
B3
B4
B5

Find1 , 2 and express them in terms ofext , , , , , 1 .(0.6 points)


Write down the condition for ()at = .(0.3 points)
Write down the condition for ()and ()at = 0.(0.6 points)
Find the total currentand express it in terms ofext , , , , , .Assume that it remains finite
(1.2 points)

Let the Townsend coefficient be constant. When the length of the tube turns out greater than some
critical value, i.e. > cr , the external ionizer can be turned off and the discharge becomes self-sustained.
B6

Find and express it in terms ofext , , , , , .(0.5 points)

Theoretical competition. Tuesday, 15 July 2014

1/3

Problem 3. Simplest model of gas discharge


Solution
Part . Non-self-sustained gas discharge
A1.Let us derive an equation describing the change of the electron number density with time. It is
determined by the two processes; the generation of ion pairs by external ionizer and the recombination of
electrons with ions. At ionization process electrons and ions are generated in pairs, and at recombination
processthey disappear in pairs as well.Thus, their concentrations are alwaysequal at any given time, i.e.
= = ()
(A1.1).
Then the equation describing the numberdensityevolution of electrons and ions in time can be written
as
()
= ()2
(A1.2).

It is easy to show that at 0 the function tanh 0, therefore, by virtue of the initial condition
0 = 0,one finds
0 = 0
(A1.3).
Substituting = tanh in (A1.2) and separating it in the independent functions (hyperbolic,
or 1 and ), one gets

(A1.4),

=
(A1.5).
A2.According to equation (A1.4) the number density of electronsat steady-state is expressed in terms of the
external ionizer activity as
1 =
2 =
=

(A2.1),

(A2.2),

1 + 2

(A2.3).

Thus,the following analogue of the Pythagorean theorem is obtained as


2
2
= 1
+ 2
= 20.0 1010 cm3 .
(A2.4)
A3.In the steady state, the balance equations of electrons and ions in the tube volume take the form

= +
(A3.1),

= +
(A3.2).
It follows from equations (A3.1) and (A3.2) that the ion and electron currents are equal, i.e.
=
(A3.3).
At the same time the total current in each tube section is the sum of the electron and ion currents
= +
(A3.4).
By definition ofthe current density the following relations hold

= 2 = =
(A3.5),

= 2 = =
(A3.6).
Substituting (A3.5) and (A3.6) into (A3.1) and (A3.2), the following quadratic equation for the
current is derived
=

+ 2

The electric field strength in the gas is equal to

=
and solution to the quadratic equation (A3.7) takes the form
=

2 2
3

1 1 +

4 4
22

(A3.7).
(A3.8).
(A3.9).

Theoretical competition. Tuesday, 15 July 2014

2/3

It is obvious that only positive root does make sense, i.e.


=

2 2

1+

4 4
22

(A3.10).

A4.At low voltages (A3.10) simplifies and gives the following expression
= 2

(A4.1)

which is actually the Ohm law.


Using the well-known relation

=
together with

=
one gets
1

= 2

(A4.2)
(A4.3),

(A4.4).

Part B. Self-sustained gas discharge


B1.Consider a gas layer located between and + .The rate of change in the electron number inside the
layer due to the electric current is givenfor a small time interval by
+ ()

=
=
.
(B1.1).

This change is due to the effect of the external ionization and the electron avalanche formation.
The external ionizer creates the following number of electrons in the volume
=
(B1.2).
whereas the electron avalanche produces the number of electrons found as
()
= = =
(B1.3).
The balance equationfor the number of electrons is written as
= +
(B1.4),
whichresults in the following differential equation for the electron current
()
= + ()
(B1.5).

1
On substituting = 1
+ 2 ,one derives
1 =
(B1.6),

2 =
(B1.7).

B2.Given the fact that the ions flow in the direction opposite to the electron motion,the balance equationfor
the number of ionsis written as
= +
(B2.1),
where

(+ )

=
=

(B2.2).

=
(B2.3).
()

=
(B2.4).
Hence, the following differential equation for the ion current is obtained
()

= + ().
(B2.5)
Onsubstituting the previouslyfound electron current together with the ion current, = 2 +
2
1 ,yields
1 = 1
(B2.6),
2 =
(B2.7).
B3.Sincetheionsstartstomovefrom the anode located at = , the following condition holds
= 0
(B3.1).
B4.By definition of secondary electron emission coefficient the following condition should be imposed
0 = 0
(B4.1).

Theoretical competition. Tuesday, 15 July 2014


B5.Total current in each tube section is the sum of the electron and ion currents:

= + = 2

Aftersubstituting the boundary conditions (B3.1) and (B4.1):


2 1 = 0
and

1
= (2 1 )

Solving (B5.2) and (B5.3) one can obtain:



1
2 =

(1+)

3/3
(B5.1).
(B5.2)
(B5.3).
(B5.4).

So the total current:


=

(1+)

(B5.5).

B6.When the discharge gap length is increased, the denominator in formula (B5.1) decreases. At that
moment, when it turns zero, the electric current in the gas becomes self-sustaining and external ionizer can
be turned off. Thus,
1
1
= ln 1 +
(B6.1).

Experimental competition. Thursday, 17 July 2014

1/9

Experiment. To see invisible! (20 points)


Introduction
Many substances exhibit optical anisotropy resulting in that the refractive index turns out dependent
on the direction of light propagation and its polarization. Optical anisotropy can occur even in isotropic
media in the presence of mechanical stresses, nonuniform heating or application of external electric fields.
The direction in which the light propagates without the birefringentis called the optic axis of a crystal.
Consider a traditional optic scheme of experiments for studying the optical anisotropy (see Fig. 1),
which is to be used in this experimental problem.

Fig. 1. Optic scheme of an experiment for studying the optical anisotropy.


Let a light beamfall onto polarizer 1 whose transmission plane intersects its ownplane along the
straight line 1 1 . After passing polarizer 1 the light beam becomes linearly polarized and its electric field
strength vector 0 oscillatesexactly in the transmition plane of polarizer 1. Then, the light beam falls onto
the anisotropic plate P oriented such that its optical axislies in the plate plane to makethe angle 45 with
the transmission plane of polarizer 1. Two kinds of light waves are then generated in the plate P: ordinary
, polarized perpendicular to the optical axis of the plate, and extraordinary , polarized along the optical
axis of the plate. Refractive indices for these two waves are different and their difference is denoted as
= . This results in the appearanceof the phase difference = 2/( being the plate
thickness, being the wavelength of the incident light in vacuum) between the two waves on leaving the
plate. Therefore, the polarization of the outgoing light beam changes to be elliptically polarized. The light
beam then falls onto polarizer 2, whosetransmission plane 2 2 is perpendicular to the transmission plane of
polarizer 1.
A simple derivation shows that the intensity of the light beam transmitted through the plate P and
polarizer 2 is determined as

2 = 0 sin2 2 ,
(1)
where0 stands for the light intensity falling onto the plate, denotes the light transmittance coefficient of the
plate P and polarizer 2, and designates the phase difference between the ordinary and extraordinary
waves after passing the plate P.

In this experiment do not evaluate errors unlessasked to do so!


The description of the equipment in Appendix A

Experimental competition. Thursday, 17 July 2014

2/9

Part 1.Qualitative observations!(3.5 points)


Part 1.1.Polarizers(0.8 points)
1.1

Find the orientation(i.e. which of the diagonals) of the transmission plane of polarizer 1 and
polarizer 2. Show these planes in the figure in the Answer Sheet. (0.8 points)

Part 1.2.Rulers(1.0 points)


In this part of the experiment uselight emitting diodes(LED) as a light source.
Fix LED on a stand and connect it to its power supply. Set up both polarizers by their face sides
(indicated bynumbers 1 and 2) pointing towards the light source. Make sure that the polarizers are crossed,
i.e. the light beamcannot pass through them. Block the first polarizer by a sheet of white paperplacing it on
the face side as shown in Fig.1B in Appendix B.
Place the plastic ruler1between the polarizers. You can move the ruler with your hands.
1.2.1 Find the possible directions of the optical axis in thecenter of the plastic ruler. Show these
directions in the figure on the Answer Sheet. (0.4 points)
1.2.2 Determine approximately at what distance along the ruler 1 and along the two rulersstacked
together, the phase difference for the blue light changes to 2. (0.6 points)

Part 1.3.Strip(0.8 points)


1.3.1

Find the possible directions of the optical axis of the strip. Show qualitatively them in the figure in
the Answer Sheet. (0.4 points)

Using the clamps fix a long flexible plastic strip on the screen so that the strip edges coincide with the
sreenedges. The stripshould be curved (see Fig. 3B). Place the screen with the strip between the polarizers.
Shifting the screen, observe the color change of the strip. Measure x coordinates of strip points on the screen
scale, use the left edge of the screenholder as an origin as shown in Figure 3B.
Hereinafter coordinates are measured by the scale in the screen. As a pointer, use the left edge of the
holder, which indicated in Fig. 3B by the arrow!
1.3.2

Measure the coordinates of the middle points of two dark bands, the left and the right ,
visible on the strip. (0.4 points)

Part 1.4.Liquid crystal cell(0.9 points)


Liquid crystal (LC) is a state of matter that is
intermediate between the crystalline solid and the amorphous
liquid. The orientation of its molecules can be easily aligned
and controlled by applying an electrical field. The LC cell
exhibits the optical anisotropy phenomenon with two principal
refractive indices. The magnitude of this effect depends on the
applied AC voltage. The Liquid Crystal Cell (LCC) is
composed of two glass plates 1 whose inner surface is coated
with a transparent conductive layer 2. Between the plates there
is a thin (approximately 10 microns) layer of the solution 3
which is in a liquid crystal state. Leads are soldered to the
plates for connecting to the AC power supply.
Place the liquid crystal cell (LCC) between the polarizers. Plug it into its power supply. Varying the
voltage across the LCC observe the changing colors of the light transmitted through it.
1.4.1 Find the possible directions of the optical axis of the LCC at zero and maximum voltageapplied
across it. Show these directions in the figure on the Answer Sheet. Z-axis is directed vertically.(0.6
points)

Experimental competition. Thursday, 17 July 2014

3/9

1.4.2 Measure the voltage across the cell at which abrupt,with respect to voltage, reorientation of
liquid crystal molecules by 90 occur.Make sure that multimeter is on AC voltage mode.(0.3
points).

Part 2.Measure!(16.5 points)


Disconnect the LED from the power supply and remove it. Remove the sheet of white paper.
In this part of the work use laser as a light source, make sure you connect it to its power supply!
Fixthe laser, polarizer 1, the screen with the slit and a photodetector (a photodiode) in the holders.
Adjust the setting so that the laser beam passes through the polarizer andthe slit of the screen to fall strictly
onto the photodiode. Using the screw 5c for the beam width adjustment, make sure that the spot size on the
photodetectoris about 5-6 mm.
The laser emits a linearly polarized light. With the ring 5a designed for the laser reorientation,make
sure that the laser beam almost completely passes through the first polarizer and the major axis of the
elliptically-shaped spot is vertical. In the following,the orientations of the laser and the photodetectormust be
fixed with the screws 5d and15c. Set up polarizer 2. Make sure that the polarizers are crossed. Fig. 4B shows
the whole setup with the screen mounted.

Part2.1.Investigating a photodiode(3.2 points)


For measuringthe light intensity,of use is the photodiode
EMF which is a rather complicated function of the incident light
intensity itself. Therefore, for measuring the light intensity the
circuit shown in Fig. 2 is used. The DC voltage measured by a
multimeter depends on the incident light intensity and the
resistance of a resistor. The main objective is to choose such an
optimum value of the resistance thatthe voltage across the
resistor is to be proportional to the intensity of the light
incident on the photodiode.

Fig. 2. Circuit for measuring the


photodiode emf.

For measurements in this part, remove the second polarizer and the screen from the optical bench.
The filters that attenuate the beam intensity should be necessarilyfixed with the clamps on the back side
of the polarizer as shown in Fig. 5B.
The maximum values of the measured voltages must be at least 300 mV.
Using a multimeter, you can measure the resistance of the resistor and the voltage across it (of
course, you have to properlyadjust the register of the multimeter). Add, in an appropriate place,the switch
provided so that you could measure both the resistance and the voltage with the single multimeter, i.e.
without disconnecting the circuit by just shorting/unshorting the switch and the multimeter register
adjustment.
2.1.1 Draw a circuit with the switch installed for measuring of the voltage across the resistor and its
resistance. (0.2 points)
2.1.2 Measure the voltage across the resistor as a function of its resistance for two values of the incident
light intensity: maximum (with the number of filters = 0) and the minimum (with the number of
filters = 5). Plot the corresponding graphs in the same figure. Specify the range of the resistance
for which the difference between the voltages is maximal. (1.0 point)
2.1.3 Measure the voltage across the resistor as a function of the number of filters = 0,1,2,3,4,5,
attenuating the intensity of the incident light on the photodiode. Measurements must be carried out
at three fixed values of the resistance, approximately equal to = 30 , = 20 and
= 10 .Plot the corresponding graphs in the same figurechoosing the scale such that it
would be possible to verify whether the voltage across the resistor depends linearly on the intensity
of the incident light registered by the photodiode. From the three above mentioned values of the
resistance choose an optimum one at which further measurements of the light intensity should
be made. (1.0 point)
2.1.4 Using this data obtained, calculate the transmittance of the filter = / and evaluate its

Experimental competition. Thursday, 17 July 2014

4/9

error,with being the intensity of the transmitted light, and being the intensity of the incident
light. You can make additional measurements if necessary. (1.0 point)
All subsequent measurements must be made at the optimal value of the resistor chosen!
It is assumed in what follows that the light intensityin relative units is equal to the voltage across the
resistor in mV.

Part 2.2 Light transmission through plastic rulers(5.4 points)


Place the plastic ruler between the polarizers. You can move the ruler with your hands. Then, secure
it with the clamps on the screen with a slit (see Fig. 2B). The lower edge of the ruler should coincide with a
line drawn on the screen, and its scale should be at the top. Make sure that both of the rulers provided
demonstrate the birefringence phenomenon. Observe a picture emerging when you put one ruler on the top
of the other so that light is to pass through both of them.
In this part use an optical scheme shown in Fig. 4B. Make sure that the rulersare fixed on the screen
at the position described in Part 1.2.
2.2.1 Measure the intensity of the transmitted light (in mV) as a functionof the coordinate of the point
of the light incidence on the ruler in the range from 0 to 10 cm.. Measurements must be carried out
for each ruler provided and for the two rulers stacked together. In each case, measure the
maximum value of the voltage. Plot the corresponding graphs in the same figure. (2.0 points)
2.2.2 For each of the two rules calculate the values of the phase shift between the ordinary and
extraordinary waves in the range of from 0 to 7 cm.Plotthe correspondinggraphs().Put down
the formula you have used for calculations. (1.2 points)
Note that the phase difference cannot unambiguously be determined from formula
(1),additional physical assumptions should be applied to determine it correctly.
2.2.3 Assuming that () is linear for each ruler
1 = 1 + 1 ,
2 = 2 + 2 ,
calculate the numerical values of theabove coefficients for rulers 1 and 2. (1.0 point)
2.2.4 Using those data obtained in parts 2.2.1-2.2.3, calculate the theoretical values of the intensity of
light passing through the two rulersstacked together. Put down the formula that you have used for
calculations. Plot the theoretical dependence in the same figure from part 2.2.1. (1.2 point)

Part 2.3 Liquid crystal cell(4.5 points)


Light transmission through LCC
Place the LCC between the polarizers as shown in Fig. 6B.
The experimental dependence under investigation is strongly nonmonotonicwith domains of quite abrupt
changes. Take this into account when taking measurements.
To measure the AC voltage of the cellpower supply and the DC voltage of the photodetector, connect the
appropriate leads directly to the multimeter.
2.3.1
2.3.2
2.3.3

Measure the intensity of the transmitted light as a function of the voltage across the LCC. Plot
the corresponding graph. (2.0 points)
Calculate the phase difference between the ordinary and extraordinary waves 0 when the
power supply is disconnected with the LCC. (1.5 points)
In a sufficiently wide range of the voltage dropacross the LCC, the phase difference between the
ordinary and extraordinary waves depends on the voltage applied by the power law
= .
Using the data obtained, plot the graphwhich allows one to determine the applicability range of
the above formula and calculate the exponent. Specify that range of applicability and evaluate
the numerical value of the parameter. (1.0 point)

Experimental competition. Thursday, 17 July 2014

5/9

Part 2.4.Light transmission through a curved strip (3.4 points)


Secure the plastic strip on the screen as described in part 1.3.
2.4.1 Measure the intensity of light, transmitted through the optical system,as a function of the
coordinatexof thepoint of light penetration into the strip in the range of 20 from its center.
Plot the corresponding graph. (1.2 point)
2.4.2 Calculate the phase shift between the ordinary and extraordinary waves0 , passing through
anuncurved strip. It is known that 0 lies in the range of10to 12. (1.2 points)
Near the center of the strip its shape may be approximated a circular arc of radius R. The theoretical
dependence of the phase shift on the distance z from the center of the strip, zR, has the form:
2

=0 1 + 2 2 2 ,
where = 1.4, is the refractive index of the strip material
2.4.3 Using the data obtained in the previous parts, calculate the radius of strip curvature near its
center. The refractive index of the strip material is equal to = 1.4. (1.0 points)

Experimental competition. Thursday, 17 July 2014


Appendix A. Experimental equipment
Optical bench 1with holders:
1a support for the light source with the
screw;
1b support for the photodetector with the
screw;
1c, 1d stands for the polarizers;
1 stand for the screen and the liquid
crystal cell (LCC).
2a, 2b polarizers on the stands. There are
numbers 1 and 2 on their face sides.
The polarizers should be installed with
their face sides towards the light source!
The transmission planes of the polarizers
makes the angle 45with the horizontal.
3 the screen with the slit (3a) and the scale
(3b)

Light sources:
4 light emitting diode (LED):
4a leads for the power supply;
4b fixing screw;
5 laser:
5a ring for turning the laser with the
scale(scale is not used);
5b leads for the power supply;
5c the screw for the beam width
adjustment on the front side;
5d fixing screw;
6 power supply for light sources:
6a switch;
6b leads for light source.
Keep the source operating only while
making measurements!
Do not point the laser beam in youror
anyones eyes, it is very dangerous!
7 variable resistor:
7a, 7b, 7c terminals for connection to a
circuit
7d knob for changing resistance.
8 switch:
8a, 8b terminals for connection to a circuit;

6/9

Experimental competition. Thursday, 17 July 2014


9a liquid crystal cell (LCC) 9ain a holder
(9b), 9 lead for connecting to the power
supply;
10 the power supply for LCC:
10a connector for the LCC;
10b leads for measuring the output
voltage;
10c knob to adjust the output voltage;
10d power on/off.
Keep the source operating only while
making measurements!
11 multimeter;
Do not press the HOLD button
11a register to measure resistance
(200 kOhm);
11b register to measure DC voltage (2V);
11c register to measure AC voltage (20V);
11d, 11e connectors for test leads;
11f power on/off.
If the display multimeter is in a "sleep"
mode - double-press power on/off!
When measuring the resistance with the
multimeter, the element must be
disconnected with a power supply!

Optical elements to be investigated


12 plastic rulers:
12a No. 1 (with the scalefrom 0 to 14 cm)
12b No. 2 (with the scalefrom20 to34cm)
13 flexible strip;
14 set of identical filters;
The filters and the strip are provided in a
separate envelope!
The plastic rulers and the strip exhibit
birefringence, their optical axes lie in
theirown planes.
15 Photodetector (photodiode)
15a - input window;
15b - leads for measuring the output voltage;
15c fixing screw.

Connecting wires, clamps, paper napkin,


a piece of paper.

7/9

Experimental competition. Thursday, 17 July 2014

8/9

The liquid crystal cell is composed of two glass plates


1 whose inner surface is coated with a transparent conductive
layer 2. Between the plates there is a thin (approximately 10
microns) layer of the solution 3 which is in a liquid crystal
state. Leads are soldered to the plates for connecting to the
AC power supply. In the absence of voltage long molecules
of liquid crystal are oriented parallel to the plates. The
direction of molecular orientation coincides with the optical
axis of the crystal.
Try not to touch those parts of the optical elements
through which the light passes! If necessary, wipe them
with a paper napkin!

Appendix B. Photos of the experimental setups

Fig. 1B Setup for the observation of birefringence Fig. 2B Mounted ruler on the screen
in the ruler

Fig. 3B Fixing flexible plastic strip to the screen.

Fig. 4B Setup for measuring the transmittance of light


through plastic ruler

Experimental competition. Thursday, 17 July 2014

Fig. 5B Mounted filters on the polarizer

9/9

Figure 6B Setup for measuring the characteristics of


the LCC

Experimental competition. Thursday, 17 July 2014

1/13

Experimental problem. To see invisible!


Part 1. Qualitative observations!
Section 1.1. Polarizers
For determination of the transmission plane of the polarizer, one can
use a glaring effect from any shining surface. It is known that the reflected
light is polarized in the plane of the reflecting surface. The corresponding
transmission planes are shown in the figure on the right.

Section 1.2. Rulers


1.2.1. In case of the incident light being polarized
along the optical axis or perpendicular to it, there is
only one kind of waves generated in the medium.
This means that no change in light polarization is to
occur. Thus, it is possible to determine either the
direction of the optical axis or the direction which is
perpendicular to it. Those possible alternatives are shown in the figure above (either along the ruler, or
perpendicular to it).
1.2.2. One can see at what parts of the rulers similar colors are observed, mainly with blue hue.
The distance between those bands for the ruler No.1 is ~12 cm, while for the two rules stacked together it is
~ 8 cm.

Section 1.3. Strip


1.3.1 Possible directions of the optical axis of the strip can be
determined in a similar way. As shown in the figure on the right,
those directions make a small angle 10 with the sides of the
strip.
1.3.2 The coordinates of the dark bands are approximately found
as follows = 3,5, = 7,5.

Section 1.4. Liquid crystal cell


1.4.1 In case of zero voltage, the directions of the optical axis can
be determined in the same way: It is either horizontal or vertical.
At the maximum voltage applied the optical axis orients along the
electric field, which means it turns perpendicular to the cell plane.
1.4.2 The voltage at which suach a sharp transition in orientation of
molecules of the liquid crystal occurs is approximately equal to
= 2 V.

Experimental competition. Thursday, 17 July 2014

2/13

Part 2. Measure!
Section 2.1. Investigating a photodiode
2.1.1 In the figure below a position for a circuit switch is shown. During measurements of the resistance, the
circuit switch should be unshorted.

2.1.2 In table 1 the results are presented of the measurements of the voltage U as a function of the
resistance. Those data are plotted in the corresponding graph.
Table 1.
=0
=5
3
3
, 10 , 10 , 10 , 103
3

0,4
0,6
2,2
3,1
5,1
7,4
12,7
17,4
24,0
31,5
41,5
51,4
58,3
66,6
75,4
93,5

33
48
156
213
311
344
363
370
374
376
378
379
380
380
381
382

0,9
4,1
6,6
8,0
11,8
15,0
19,4
24,9
31,8
38,8
46,0
51,9
60,4
67,5
76,4
88,2
96,9
99,8

3
18
29
35
52
66
86
111
141
170
200
220
240
252
263
271
275
276

Note that the optimal resistance should be within the range 5-15 , which corresponds to the
largest variation in the voltage.

Experimental competition. Thursday, 17 July 2014

3/13

2.1.3 In table 2 the results are shown of the measurements for the voltage as a function of the number of
light filters at different values of resistance.
Table 2.
=

29,9 kOhm
,
ln

20,4 kOhm
,
ln

10,1 kOhm
,
ln

0
1
2
3
4

391
370
346
317
288

5,969
5,914
5,846
5,759
5,663

388
364
336
309
234

5,961
5,897
5,817
5,733
5,455

377
341
294
179
105

5,932
5,832
5,684
5,187
4,654

212

5,357

148

4,997

66

4,190

Intensity of the light that has passed through the filter decreases as a geometric progression when
increasing the number of filters :
= 0 .
(1)
In case if the measured voltage is proportional to the intensity of the incident light, it obeys a similar
law:
= 0 .
(2)
To verify equation (2), one needs to use a semi-logariphmic scale. In other words, it is necessary to
plot ln as a function of :
ln = ln 0 + ln .
(3)
That plot is shown in the following figure.

Experimental competition. Thursday, 17 July 2014

4/13

According to the graph above, by decreasing the resistance the dependence turns a linear function
and further measurements should be made at the lowest resistance among given values, i.. at =
10 .
2.1.4. According to equation (3) , the slope is = ln . Using the Method of Least Squares, we can obtain
its value = 0.53 0.03. Thus, the coefficient of transmission turns to be equal to = exp = 0.59
with an error, which can be calculated by applying the following formula = exp() = 0.02. Finally
we obtain
= 0.59 0.02.
Note that values for = 10 produce the following result: = 0.59 0.02.

Experimental competition. Thursday, 17 July 2014

5/13

2.2 Light transition through a plastic ruler


2.2.1 Results of measurements of the light intensity as a function of coordinates of transmission points
through ruler #1, #2 and both rulers, are shown in table 3 and in the graph below.
Table 3.
X,mm

1
U,mV

5
10
15
20
25
30
35
40
45
50
55
60
65
70
75
80
85
90
95
100

23
6
0
3
12
30
50
71
93
113
128
150
156
153
160
146
146
140
131
113

0,778
0,390
0,000
0,275
0,555
0,896
1,186
1,458
1,734
1,996
2,214
2,636
2,824
2,720
3,142
2,541
2,541
2,419
2,262
1,996

X,mm

2
U,mV

5
10
15
20
25
30
35
40
45
50
55
60
65
70
75
80
85
90
95
100

14
22
30
43
55
67
78
90
99
107
116
123
129
133
130
134
143
144
146
145

0,601
0,760
0,896
1,090
1,253
1,408
1,546
1,696
1,811
1,915
2,038
2,138
2,230
2,295
2,246
2,312
2,478
2,498
2,541
2,519

Both rulers
X,mm
U,mV
Ucalc
5
10
15
20
25
30
35
40
45
50
55
60
65
70
75
80
85
90
95
100

1
2
8
18
31
39
41
42
41
36
27
21
12
7
2
1
1
3
6
11

0,0
2,1
7,7
15,7
24,7
33,0
39,2
41,9
40,8
36,0
28,5
19,6
10,9
4,1
0,4
0,5
4,4
11,4
20,1
29,0

Experimental competition. Thursday, 17 July 2014

6/13

2.2.2 To calculate a phase shift, we use equation (1), mentioned in the problems formulation, which can be
represented as

= sin2 2 ,
(1)
where is the largest value of voltage. But we have to be sure that this value actually corresponds to the
maximum of function (1), not just another boundary point. According to measurements, (see the graph) for
each ruler the most suitable value for is = 160 .
The following equation

0 = sin2 2
(1)
has multiple roots and it is not easy to find actual values of the phase shift, even if its possible to calculate
certain value for Umax, roots of the equation mentioned above are shown in the figure below

Formally we can represent the roots in different forms, for example,


= 2(arcsin 0 + ),
= 2( arcsin 0 + ),
= 0,1,2 .
Choosing a correct root should depend on a function obtained experimentally.
Values for the phase shifts calculated by the equation
= 2 arcsin

(2)

(3)

are shown in table 3


It is clear that the function () has to be monotonous, that is why the signs of roots for the first
two points must be changed, which is a mathematically correct operation (just reflecting the graph). Note
that the phase shifts are calculated with uncertainty of 2.
2.2.3 Obtained functions are close to linear, using MLS we get
1 = 0.059 0.94,
2 = 0.028 + 0.52.

Experimental competition. Thursday, 17 July 2014

7/13

Graphs of those functions are shown below.

2.2.4 If two rulers are stacked together, then phase shifts simply add, and, theoretically, the intensity as
function of phase shifts can be written as
+
= sin2 1 2 2 .
(5)
Here is the largest value of voltage at the light transition through both rulers and can be obtained from
experimental data.

Results of the calculations are shown in table 3 and in the graph above. Consistency of theoretical
calculations and experimental data can clearly be seen.

Experimental competition. Thursday, 17 July 2014

8/13

Part 2.3 Liquid crystal cell


2.3.2 Light transmission through LCC
2.3.1 Results of the intensity measurements as a functions of voltage are shown in table 51. Graph of the
obtained function is drawn in the figure below.

Table 5.

0
0,86
0,91
0,93
0,94
1,02
1,07
1,09
1,11
1,18
1,23
1,27
1,29
1,31
1,32
1,36
1,39
1,42
1,46
1,5
1,55
1,63
1,68
1,71
1,78
1,83
1,93
2,01
2,11
2,24
2,34
2,51
2,65
2,72
2,85
2,92
3,05
3,16
3,22
3,34
3,45
3,59
3,66
3,75

207
207
211
226
237
294
297
294
285
201
110
51
26
10
5
2
12
28
66
102
156
232
261
275
289
294
295
294
287
273
258
229
202
191
169
159
141
128
121
109
98
86
81
74

1,961
1,961
1,990
2,102
2,190
2,858
2,941
2,858
2,691
1,918
1,301
0,850
0,598
0,367
0,259
0,163
0,403
0,621
0,976
1,245
1,611
2,149
2,404
2,556
2,756
2,858
2,883
2,858
2,722
2,532
2,375
2,125
1,925
1,848
1,698
1,631
1,511
1,424
1,376
1,294
1,217
1,130
1,093
1,039

10,606
10,606
10,577
10,464
10,377
9,709
9,625
9,709
8,974
8,201
7,584
7,133
6,881
6,650
6,542
6,447
5,880
5,662
5,307
5,038
4,672
4,134
3,879
3,727
3,527
3,425
3,401
2,858
2,722
2,532
2,375
2,125
1,925
1,848
1,698
1,631
1,511
1,424
1,376
1,294
1,217
1,130
1,093
1,039

ln

ln

-0,151
-0,094
-0,073
-0,062
0,020
0,068
0,086
0,104
0,166
0,207
0,239
0,255
0,270
0,278
0,307
0,329
0,351
0,378
0,405
0,438
0,489
0,519
0,536
0,577
0,604
0,658
0,698
0,747
0,806
0,850
0,920
0,975
1,001
1,047
1,072
1,115
1,151
1,169
1,206
1,238
1,278
1,297
1,322

2,361
2,361
2,359
2,348
2,340
2,273
2,264
2,273
2,194
2,104
2,026
1,965
1,929
1,895
1,878
1,864
1,772
1,734
1,669
1,617
1,542
1,419
1,356
1,316
1,260
1,231
1,224
1,050
1,001
0,929
0,865
0,754
0,655
0,614
0,529
0,489
0,413
0,353
0,319
0,258
0,196
0,122
0,089
0,039

We do not expect that participants can take the same number of measurements, 15-20 points are enough. It is principally
important to find the dip in the graph.

Experimental competition. Thursday, 17 July 2014


3,83
3,91
4,03
4,21
4,43
4,79
4,98
5,15
5,51
5,79
6,07
6,59
6,94

70
65
58
50
41
30
25
21
15
11
8
4
2

1,008
0,968
0,911
0,841
0,757
0,644
0,586
0,536
0,451
0,385
0,328
0,231
0,163

1,008
0,968
0,911
0,841
0,757
0,644
0,586
0,536
0,451
0,385
0,328
0,231
0,163

1,343
1,364
1,394
1,437
1,488
1,567
1,605
1,639
1,707
1,756
1,803
1,886
1,937

9/13

0,008
-0,032
-0,094
-0,173
-0,278
-0,441
-0,535
-0,625
-0,796
-0,954
-1,115
-1,463
-1,811

It is important to choose correct roots of equation (2) in order to adequately calculate phase shifts. In
this case it is rather obvious because at large values of the voltage difference tends to zero, 0.
Other solutions and corresponding equations are shown in the figure below.

Experimental competition. Thursday, 17 July 2014

Results of calculation of = arcsin

10/13

and correct values of phase shift are shown in the figure

below.

This figure is drawn for the sake of


understanding. (Its not required to draw it
for participants).
2.3.2 The value of the phase shift at zero
voltage is 0 10.6.
2.3.3 In order to check applicability of the
power
function
=
it
is
recommended to redraw the last graph logarithmically, as shown in the figure below.

Experimental competition. Thursday, 17 July 2014

11/13

It can be seen from the graph that in the range of 1 V to 5 V the function is almost linear, which
justifies the applicability of the power law. The power in that equation is equal to the slope of the graph, its
numerical value is 1.75.

Experimental competition. Thursday, 17 July 2014

12/13

Section 2.4 Light transmission through a curved strip


2.4.1 Results of the measurements of the light intensity as a function of coordinate z of the point of light
penetration into the strip are presented in table 6 and plotted below.
Table 6.
x, mm
U, mV
z, mm

40
41
42
43
44
45
46
47
48
49
50
51
52
53
54
55
56
57
58
59
60
61
62
63
64
65
66
67
68
69
70
71
72
73
74
75
76
77
78
79
80

16
8
3
1
1
1
1
1
1
4
11
23
41
61
78
90
99
102
100
98
96
95
93
96
99
104
107
107
98
81
65
44
24
10
3
1
1
1
1
1
2

0,794
0,554
0,336
0,194
0,194
0,194
0,194
0,194
0,194
0,389
0,653
0,964
1,335
1,711
2,046
2,322
2,588
2,706
2,624
2,553
2,489
2,459
2,401
2,489
2,588
2,805
3,142
3,142
2,553
2,111
1,787
1,392
0,987
0,621
0,336
0,194
0,194
0,194
0,194
0,194
0,274

5,489
5,729
5,947
6,090
6,090
6,090
6,090
6,090
6,090
5,894
5,630
5,319
4,948
4,572
4,237
3,962
3,696
2,706
2,624
2,553
2,489
2,459
2,401
2,489
2,588
2,805
3,142
3,142
3,730
4,173
4,496
4,891
5,296
5,662
5,947
6,090
6,090
6,090
6,090
6,090
6,009

-22
-21
-20
-19
-18
-17
-16
-15
-14
-13
-12
-11
-10
-9
-8
-7
-6
-5
-4
-3
-2
-1
0
1
2
3
4
5
6
7
8
9
10
11
12
13
14
15
16
17
18

Experimental competition. Thursday, 17 July 2014

13/13

2.4.2 The shape of the curve indicates that 0 lies at the ascending part of the relation between the
intensity and the phase shift, which can be calculated as
0 = 10 + 2 arcsin

33.9.

Graph of the phase shift is drawn


in the figure on right. Because we are
interested in the central part of the graph,
reflection parts are not shown. (This
graph is not required from participants)
2.4.3 The graph shows that the central part
is approximately parabolic function of z
= 2 + .
(10)
In order to determine the
coefficients of the function we can draw
graph of as a function of 2 (see figure
on the right). Using MLS, we can
determine the parameters
= 0.0104 1 ,
= 2.45.
It is necessary to add 10 to the obtained value of
b.
Comparing equations (9) and (10), we
conclude that the parameters can be represented by
the strip characteristics as:

= 2 2 0 2 , = 0 .
(11)
From those equations we get the radius of
curvature of the strip
1

(12)

Substitution of the obtained results leads us to = 29 . Note that the obtained result is quite
rough, due to uncertainties in measuring.

Q T-1
Page 1 of 2
Particles from the Sun1

(Total Marks: 10)

Photons from the surface of the Sun and neutrinos from its core can tell us about solar temperatures and also
confirm that the Sun shines because of nuclear reactions.
Throughout this problem, take the mass of the Sun to be
, its luminosity (radiation energy emitted per unit time),
distance,
.

, its radius,
, and the Earth-Sun

Note:

)
)

A Radiation from the sun :


A1

Assume that the Sun radiates like a perfect blackbody. Use this fact to calculate the temperature,
solar surface.

, of the

0.3

The spectrum of solar radiation can be approximated well by the Wien distribution law. Accordingly, the
solar energy incident on any surface on the Earth per unit time per unit frequency interval,
, is given by

where

is the frequency and

is the area of the surface normal to the direction of the incident radiation.

Now, consider a solar cell which consists of a thin disc of semiconducting material of area,
perpendicular to the direction of the Sun rays.

, placed

A2

Using the Wien approximation, express the total radiated solar power, , incident on the surface of the
solar cell, in terms of ,
,
, and the fundamental constants , , .

0.3

A3

Express the number of photons,


the solar cell in terms of ,
,

0.2

, per unit time per unit frequency interval incident on the surface of
, and the fundamental constants , , .

Th
du
r l f h
l r ll h
b d
f
r y, . We assume the following
model. Every photon of energy
excites an electron across the band gap. This electron contributes an
energy, , as the useful output energy, and any extra energy is dissipated as heat (not converted to useful
energy).
A4

Define
, ,

A5

Express the efficiency, , of this solar cell in terms of

A6

Make a qualitative sketch of


is the slope of
at

A7

Let
be the value of
for which
value of within an accuracy of

A8

The band gap of pure silicon is


value.

where
. Express the useful output power of the cell,
and the fundamental constants , , .

versus
and

. The values at
?

, in terms of

, ,

1.0
0.2

and

should be clearly shown. What

is maximum. Obtain the cubic equation that gives


. Hence calculate
.
. Calculate the efficiency,

. Estimate the

, of a silicon solar cell using this

1.0
1.0
0.2

Amol Dighe (TIFR), Anwesh Mazumdar (HBCSE-TIFR) and Vijay A. Singh (ex-National Coordinator, Science Olympiads)
were the principal authors of this problem. The contributions of the Academic Committee, Academic Development Group and the
International Board are gratefully acknowledged.

Q T-1
Page 2 of 2
In the late nineteenth century, Kelvin and Helmholtz (KH) proposed a hypothesis to explain how the Sun
shines. They postulated that starting as a very large cloud of matter of mass,
, and negligible density, the
Sun has been shrinking continuously. The shining of the Sun would then be due to the release of
gravitational potential energy through this slow contraction.
A9

Let us assume that the density of matter is uniform inside the Sun. Find the total gravitational potential
energy, , of the Sun at present, in terms of G,
and
.

0.3

A10

Estimate the maximum possible time,


(in years), for which the Sun could have been shining, according
to the KH hypothesis. Assume that the luminosity of the Sun has been constant throughout this period.

0.5

The
calculated above does not match the age of the solar system estimated from studies of meteorites.
This shows that the energy source of the Sun cannot be purely gravitational.
B Neutrinos from the Sun :
In 1938, Hans Bethe proposed that nuclear fusion of hydrogen into helium in the core of the Sun is the source
of its energy. The net nuclear reaction is:
Th l r
ur
, , produced in this reaction may be taken to be massless. They escape the Sun
and their detection on the Earth confirms the occurrence of nuclear reactions inside the Sun. Energy carried
away by the neutrinos can be neglected in this problem.
B1

Calculate the flux density,


, of the number of neutrinos arriving at the Earth, in units of
The
energy released in the above reaction is
. Assume that the energy radiated by the Sun is
entirely due to this reaction.

0.6

Travelling from the core of the Sun to the Earth, some of the electron neutrinos, , are converted to other
types of neutrinos, . The efficiency of the detector for detecting is 1/6 of its efficiency for detecting .
If there is no neutrino conversion, we expect to detect an average of
neutrinos in a year. However, due to
the conversion, an average of
neutrinos ( and combined) are actually detected per year.
B2

B3

In terms of

and

, calculate what fraction,

, of

is converted to

0.4

In order to detect neutrinos, large detectors filled with water are constructed. Although the interactions of
neutrinos with matter are very rare, occasionally they knock out electrons from water molecules in the
detector. These energetic electrons move through water at high speeds, emitting electromagnetic radiation
in the process. As long as the speed of such an electron is greater than the speed of light in water (refractive
index, ), this radiation, called Cherenkov radiation, is emitted in the shape of a cone.
Assume that an electron knocked out by a neutrino loses energy at a constant rate of per unit time, while it
travels through water. If this electron emits Cherenkov radiation for a time, , determine the energy
imparted to this electron (
, n,
and . (Assume the electron to
r d by the neutrino, in terms of ,
be at rest before its interaction with the neutrino.)

2.0

The fusion of H into He inside the Sun takes place in several steps. Nucleus of
(rest mass,
) is
produced in one of these intermediate steps. Subsequently, it can absorb an electron, producing
a
nucleus (rest mass,
<
) and emitting a . The corresponding nuclear reaction is:

When a Be nucleus
is at rest and absorbs an electron also at rest, the emitted
neutrino has energy
. However, the
nuclei are in random thermal motion due to the
temperature at the core of the Sun, and act as moving neutrino sources. As a result, the energy of emitted
neutrinos fluctuates with a root mean square (rms) value
.
B4

If
=
, calculate the rms speed of the Be nuclei,
, and hence estimate
depends on the rms value of the component of velocity along the line of sight).

. (Hint:

2.0

Theoretical Task 1 (T-1) : Solutions

1 of 7

Particles from the Sun1


Photons from the surface of the Sun and neutrinos from its core can tell us about solar temperatures
and also confirm that the Sun shines because of nuclear reactions.
Throughout this problem, take the mass of the Sun to be M = 2.00 1030 kg, its radius, R =
7.00 108 m, its luminosity (radiation energy emitted per unit time), L = 3.85 1026 W, and the
Earth-Sun distance, d = 1.50 1011 m.
Note:

1
x

eax + constant
(i)
xe dx =
a a2
 2

Z
x
2x
2
2 ax
(ii)
x e dx =
2 + 3 eax + constant
a
a
a
 3

Z
3x2 6x
6
x
3 ax
2 + 3 4 eax + constant
(iii)
x e dx =
a
a
a
a
Z

ax

A. Radiation from the Sun :


(A1) Assume that the Sun radiates like a perfect blackbody. Use this fact to calculate the temperature,
Ts , of the solar surface.

[0.3]

Solution:
2
)(Ts4 )
Stefans law: L = (4R


Ts =

L
2
4R

1/4

= 5.76 103 K

The spectrum of solar radiation can be approximated well by the Wien distribution law. Accordingly,
the solar energy incident on any surface on the Earth per unit time per unit frequency interval, u(),
is given by
R2 2h
u() = A 2 2 3 exp(h/kB Ts ),
d c
where A is the area of the surface normal to the direction of the incident radiation.
Now, consider a solar cell which consists of a thin disc of semiconducting material of area, A, placed
perpendicular to the direction of the Suns rays.
(A2) Using the Wien approximation, express the total power, Pin , incident on the surface of the solar
cell, in terms of A, R , d , Ts and the fundamental constants c, h, kB .
1

Amol Dighe (TIFR), Anwesh Mazumdar (HBCSE-TIFR) and Vijay A. Singh (ex-National Coordinator, Science
Olympiads) were the principal authors of this problem. The contributions of the Academic Committee, Academic Development Group and the International Board are gratefully acknowledged.

[0.3]

Theoretical Task 1 (T-1) : Solutions


Solution:

Z
u()d =

Pin =
0

A
0

2 of 7

2
R
2h 3
exp(h/kB Ts )d
2
d c2

kB Ts
kB Ts
h
. Then, =
x
d =
dx.
kB Ts
h
h
Z
2
2
2
2hAR
(kB Ts )4 3 x
12kB4 4 R
2kB4 4 R

6
=
Pin =
x
e
dx
=
T
A
T
A
s
s
c2 d2
h4
c2 h3
d2
c2 h3
d2
0

Let x =

(A3) Express the number of photons, n (), per unit time per unit frequency interval incident on the
surface of the solar cell in terms of A, R , d , Ts and the fundamental constants c, h, kB .

[0.2]

Solution:
u()
h
R2 2
= A 2 2 2 exp(h/kB Ts )
d c

n () =

The semiconducting material of the solar cell has a band gap of energy, Eg . We assume the following model. Every photon of energy E Eg excites an electron across the band gap. This electron
contributes an energy, Eg , as the useful output energy, and any extra energy is dissipated as heat (not
converted to useful energy).
(A4) Define xg = hg /kB Ts where Eg = hg . Express the useful output power of the cell, Pout , in
terms of xg , A, R , d , Ts and the fundamental constants c, h, kB .

[1.0]

Solution:
The useful power output is the useful energy quantum per photon, Eg hg , multiplied by
the number of photons with energy, E Eg .
Z
Pout = hg
n ()d
g

R2 2
= hg A 2 2
d c
= kB Ts xg A
=

2kB4
c2 h3

2
R
d2

2 exp(h/kB Ts )d

2
c2

kB Ts
h

3 Z

x2 ex dx

xg

2
R
4
Ts A 2 xg (x2g + 2xg + 2)exg
d

(A5) Express the efficiency, , of this solar cell in terms of xg .

[0.2]

Theoretical Task 1 (T-1) : Solutions

3 of 7

Solution:
Efficiency =

Pout
xg 2
=
(x + 2xg + 2)exg
Pin
6 g

(A6) Make a qualitative sketch of versus xg . The values at xg = 0 and xg should be clearly
shown. What is the slope of (xg ) at xg = 0 and xg ?

[1.0]

Solution:

1 3
(x + 2x2g + 2xg )exg
6 g
Put limiting values, (0) = 0
() = 0.
Since the polynomial has all positive coefficients, it increases monotonically; the exponential
function decreases monotonically. Therefore, has only one maximum.
=

d
1
= (x3g + x2g + 2xg + 2)exg
dxg
6


d
1
d
=
=0
dxg xg =0 3
dxg xg

xg
(A7) Let x0 be the value of xg for which is maximum. Obtain the cubic equation that gives x0 .
Estimate the value of x0 within an accuracy of 0.25. Hence calculate (x0 ).
Solution:
The maximum will be for

d
1
= (x3g + x2g + 2xg + 2)exg = 0
dxg
6
p(xg ) x3g x2g 2xg 2 = 0

A Numerical Solution by the Bisection Method:


Now,
p(0) = 2
p(1) = 4
p(2) = 2
p(3) = 10

2 < x0 < 3
p(2.5) = 2.375
2 < x0 < 2.5
p(2.25) = 0.171 2.25 < x0 < 2.5
The approximate value of xg where is maximum is x0 = 2.27.

[1.0]

Theoretical Task 1 (T-1) : Solutions

4 of 7

Alternative methods leading to the same result are acceptable.


(2.27) = 0.457
(A8) The band gap of pure silicon is Eg = 1.11 eV. Calculate the efficiency, Si , of a silicon solar cell
using this value.
Solution:

[0.2]

1.11 1.60 1019


= 2.23
1.38 1023 5763
xg
Si = (x2g + 2xg + 2)exg = 0.457
6
xg =

In the late nineteenth century, Kelvin and Helmholtz (KH) proposed a hypothesis to explain how the
Sun shines. They postulated that starting as a very large cloud of matter of mass, M , and negligible
density, the Sun has been shrinking continuously. The shining of the Sun would then be due to the
release of gravitational energy through this slow contraction.
(A9) Let us assume that the density of matter is uniform inside the Sun. Find the total gravitational
potential energy, , of the Sun at present, in terms of G, M and R .

[0.3]

Solution:
Z
The total gravitational potential energy of the Sun: =

3M
For constant density, =
3
4R
Z
=


G

4
m = r3
3

4 3
r
3

4r2

Gm dm
r

dm = 4r2 dr

5
 dr
16 2 G2 R
3 GM 2
=
=
r
3
5
5 R

(A10) Estimate the maximum possible time KH (in years), for which the Sun could have been shining, according to the KH hypothesis. Assume that the luminosity of the Sun has been constant
throughout this period.
Solution:
KH =
KH

3GM 2
=
= 1.88 107 years
5R L

The KH calculated above does not match the age of the solar system estimated from studies of meteorites. This shows that the energy source of the Sun cannot be purely gravitational.

[0.5]

Theoretical Task 1 (T-1) : Solutions

5 of 7

B. Neutrinos from the Sun:


In 1938, Hans Bethe proposed that nuclear fusion of hydrogen into helium in the core of the Sun is
the source of its energy. The net nuclear reaction is:
41 H 4 He + 2e+ + 2e
The electron neutrinos, e , produced in this reaction may be taken to be massless. They escape the
Sun and their detection on Earth confirms the occurrence of nuclear reactions inside the Sun. Energy
carried away by the neutrinos can be neglected in this problem.
(B1) Calculate the flux density, , of the number of neutrinos arriving at the Earth, in units of
m2 s1 . The energy released in the above reaction is E = 4.0 1012 J. Assume that the
energy radiated by the Sun is almost entirely due to this reaction.

[0.6]

Solution:
4.0 1012 J 2

3.85 1026
L

2
=
2 = 6.8 1014 m2 s1 .
2
11
2
12
4d E
4 (1.50 10 ) 4.0 10

Travelling from the core of the Sun to the Earth, some of the electron neutrinos, e , are converted to
other types of neutrinos, x . The efficiency of the detector for detecting x is 1/6th of its efficiency
for detecting e . If there is no neutrino conversion, we expect to detect an average of N1 neutrinos in
a year. However, due to the conversion, an average of N2 neutrinos (e and x combined) are actually
detected per year.
(B2) In terms of N1 and N2 , calculate what fraction, f , of e is converted to x .
Solution:

N1
Ne
Nx
N2

=
=
=
=

N0
N0 (1 f )
N0 f /6
Ne + Nx

OR
f
(1 f )N1 + N1 = N2
6


6
N2
f =
1
5
N1

[0.4]

Theoretical Task 1 (T-1) : Solutions

6 of 7

In order to detect neutrinos, large detectors filled with water are constructed. Although the interactions
of neutrinos with matter are very rare, occasionally they knock out electrons from water molecules in
the detector. These energetic electrons move through water at high speeds, emitting electromagnetic
radiation in the process. As long as the speed of such an electron is greater than the speed of light
in water (refractive index, n), this radiation, called Cherenkov radiation, is emitted in the shape of a
cone.
(B3) Assume that an electron knocked out by a neutrino loses energy at a constant rate of per
unit time, while it travels through water. If this electron emits Cherenkov radiation for a time
t, determine the energy imparted to this electron (Eimparted ) by the neutrino, in terms of
, t, n, me , c. (Assume the electron to be at rest before its interaction with the neutrino.)

[2.0]

Solution:
When the electron stops emitting Cherenkov radiation, its speed has reduced to vstop = c/n.
Its total energy at this time is
nme c2
m e c2
=
Estop = q
2
n2 1
1 vstop
/c2
The energy of the electron when it was knocked out is
nme c2
Estart = t +
n2 1
Before interacting, the energy of the electron was equal to me c2 .
Thus, the energy imparted by the neutrino is
2

Eimparted = Estart me c = t +


n

1 me c2
2
n 1

The fusion of H into He inside the Sun takes place in several steps. Nucleus of 7 Be (rest mass, mBe )
is produced in one of these intermediate steps. Subsequently, it can absorb an electron, producing a
7
Li nucleus (rest mass mLi < mBe ) and emitting a e . The corresponding nuclear reaction is:
7

Be + e 7 Li + e .

When a Be nucleus (mBe = 11.651027 kg) is at rest and absorbs an electron also at rest, the emitted
neutrino has energy E = 1.44 1013 J. However, the Be nuclei are in random thermal motion due
to the temperature Tc at the core of the Sun, and act as moving neutrino sources. As a result, the
energy of emitted neutrinos fluctuates with a root mean square value Erms .
(B4) If Erms = 5.54 1017 J, calculate the rms speed of the Be nuclei, VBe and hence estimate Tc .
(Hint: Erms depends on the rms value of the component of velocity along the line of sight.)
[2.0]

Theoretical Task 1 (T-1) : Solutions

7 of 7

Solution:
Moving 7 Be nuclei give rise to Doppler effect for neutrinos. Since the fractional change
in energy (Erms /E 104 ) is small, the Doppler shift may be considered in the nonrelativistic limit (a relativistic treatment gives almost same answer). Taking the line of sight
along the z-direction,
Erms
vz,rms
=
E
c
= 3.85 104
1 VBe
=
3 c
VBe =

3 3.85 104 3.00 108 m s1 = 2.01 105 m s1 .

The average temperature is obtained by equating the average kinetic energy to the thermal
energy.
3
1
2
mBe VBe
= kB Tc
2
2

Tc = 1.13 107 K

Q T-2
Page 1 of 2
The Extremum Principle1
A

The Extremum Principle in Mechanics


Consider a horizontal frictionless
plane shown in Fig. 1. It is
divided into two regions, I and II, by a line AB satisfying the
equation
. The potential energy of a point particle of mass
in
region I is
while it is
in region II. The particle is sent
from the origin O with speed
along a line making an angle
with
the x-axis. It reaches point P in region II traveling with speed
along a
line that makes an angle
with the x-axis. Ignore gravity and
relativistic effects in this entire task T-2 (all parts).

A1
A2

Obtain an expression for in terms of


Express in terms of , and .

(Total Marks: 10)

and

II

Figure 1

0.2
0.3

We define a quantity called action


, where
is the infinitesimal length along the trajectory

of a particle of mass moving with speed


. The integral is taken over the path. As an example, for a
particle moving with constant speed on a circular path of radius , the action for one revolution will be
. For a particle with constant energy , it can be shown that of all the possible trajectories between
two fixed points, the actual trajectory is the one on which defined above is an extremum (minimum or
maximum). Historically this is known as the Principle of Least Action (PLA).

A3

PLA implies that the trajectory of a particle moving between two fixed points in a region of constant
potential will be a straight line. Let the two fixed points and in Fig. 1 have coordinates
and
respectively and the boundary point where the particle transits from region I to region II have
coordinates
Note that
is fixed and the action depends on the coordinate
only. State the
expression for the action
. Use PLA to obtain the relationship between
and these coordinates.

The Extremum Principle in Optics


II

A light ray travels from medium I to medium II with refractive indices

and
respectively. The two media are separated by a line parallel to
the x-axis. The light ray makes an angle with the y-axis in medium I

I
and in medium II (see Fig. 2). To obtain the trajectory of the ray, we
make use of another extremum (minimum or maximum) principle known

as Fermats principle of least time.

1.0

Figure 2

B1

The principle states that between two fixed points, a light ray moves along a path such that time taken
between the two points is an extremum. Derive the relation between
and
on the basis of
Fermats principle.

Shown in Fig. 3 is a schematic sketch of the path of a laser beam incident

horizontally on a solution of sugar in which the concentration of sugar


decreases with height. As a consequence, the refractive index of the
solution also decreases with height.

0.5

Figure 3: Tank of Sugar Solution

B2
B3

Assume that the refractive index


depends only on . Use the equation obtained in B1 to obtain the
expression for the slope
of the beams path in terms of refractive index
at
and
.
The laser beam is directed horizontally from the origin
into the sugar solution at a height
from the
bottom of the tank as shown in figure 3. Take
where
and are positive constants.
Obtain an expression for in terms of and related quantities for the actual trajectory of the laser beam.

1.5
1.2

Manoj Harbola (IIT-Kanpur) and Vijay A. Singh (ex-National Coordinator, Science Olympiads) were the principal
authors of this problem. The contributions of the Academic Committee, Academic Development Group and the
International Board are gratefully acknowledged.

Q T-2
Page 2 of 2
You may use:

constant, where

B4
C

C1

C2

or

Obtain the value of , the point where the beam meets the bottom of the tank. Take
,
cm (1 cm = 10-2 m).

The Extremum Principle and the Wave Nature of Matter


We now explore the connection between the PLA and the wave nature of a moving particle. For this we
assume that a particle moving from to can take all possible trajectories and we will seek a trajectory
that depends on the constructive interference of de Broglie waves.
As the particle moves along its trajectory by an infinitesimal distance , relate the change
in the phase
of its de Broglie wave to the change
in the action and the Planck constant.
Recall the problem from part A where the particle traverses from
to
(see Fig. 4). Let an opaque partition be placed at the
boundary AB between the two regions. There is a small opening
CD of width in AB such that
and
.

Matter Wave Interference


Consider an electron gun at which directs a collimated beam of
electrons to a narrow slit at in the opaque partition A B at
such that
is a straight line. is a point on the screen
at
(see Fig. 5). The speed in I is

ms
and
. The potential in II is such that speed

m s . The distance
is
(
). Ignore electron-electron interaction.

0.6

0.8

II

A
D

Consider two extreme paths C and D such that C lies on


the classical trajectory discussed in part A. Obtain the phase
difference
between the two paths to first order.
D

cm,

1.2

CD d

Figure 4

II

215.00 nm

250 mm

Figure 5

D1
D2
D3
D4

If the electrons at have been accelerated from rest, calculate the accelerating potential .
Another identical slit is made in the partition A B at a distance of
nm ( nm
m) below slit
(Fig. 5). If the phase difference between de Broglie waves arriving at P through the slits F and G is
,
calculate .
What is the smallest distance
from P at which null (zero) electron detection maybe expected on the
screen? [Note: you may find the approximation
useful]
The beam has a square cross section of

and the setup is 2 m long. What should be the


minimum flux density Imin (number of electrons per unit normal area per unit time) if, on an average, there
is at least one electron in the setup at a given time?

0.3
0.8
1.2
0.4

Theoretical Task 2 (T-2) : Solutions

1 of 9

The Extremum Principle1


A. The Extremum Principle in Mechanics
Consider a horizontal frictionless x-y plane
shown in Fig. 1. It is divided into two regions,
I and II, by a line AB satisfying the equation
x = x1 . The potential energy of a point particle of mass m in region I is V = 0 while it is
V = V0 in region II. The particle is sent from
the origin O with speed v1 along a line making
an angle 1 with the x-axis. It reaches point P
in region II traveling with speed v2 along a line
that makes an angle 2 with the xaxis. Ignore
gravity and relativistic effects in this entire task
T-2 (all parts).

Figure 1

(A1) Obtain an expression for v2 in terms of m, v1 and V0 .

[0.2]

Solution:
From the principle of Conservation of Mechanical Energy
1 2 1 2
mv = mv + V0
2 1 2 2
v2 = (v12

2V0 1/2
)
m

(A2) Express v2 in terms of v1 , 1 and 2 .


Solution:
At the boundary there is an impulsive force ( dV /dx) in the x direction. Hence
only the velocity component in the xdirection v1x suffers change . The component in
the ydirection remains unchanged. Therefore
v1y = v2y
v1 sin 1 = v2 sin 2

R
We define a quantity called action A = m v(s) ds, where ds is the infinitesimal length along
the trajectory of a particle of mass m moving with speed v(s). The integral is taken over the
path. As an example. for a particle moving with constant speed v on a circular path of radius
R, the action A for one revolution will be 2mRv. For a particle with constant energy E, it can
be shown that of all the possible trajectories between two fixed points, the actual trajectory is
the one on which A defined above is an extremum (minimum or maximum). Historically this
is known as the Principle of Least Action (PLA).
1

Manoj Harbola (IIT-Kanpur) and Vijay A. Singh (ex-National Coordinator, Science Olympiads) were the
principal authors of this problem. The contributions of the Academic Committee, Academic Development Group
and the International Board are gratefully acknowledged.

[0.3]

Theoretical Task 2 (T-2): Solutions

2 of 9

(A3) PLA implies that the trajectory of a particle moving between two fixed points in a region
of constant potential will be a straight line. Let the two fixed points O and P in Fig. 1
have coordinates (0,0) and (x0 ,y0 ) respectively and the boundary point where the particle
transits from region I to region II have coordinates (x1 ,). Note x1 is fixed and the action
depends on the coordinate only. State the expression for the action A(). Use PLA to
obtain the the relationship between v1 /v2 and these coordinates.

[1.0]

Solution:
By definition A() from O to P is
q
p
A() = mv1 x21 + 2 + mv2 (x0 x1 )2 + (y0 )2
Differentiating w.r.t. and setting the derivative of A() to zero
(x21

v1
v2 (y0 )

=0
2
1/2
+ )
[(x0 x1 )2 + (y0 )2 ]1/2

(y0 ) (x21 + 2 )1/2


v1
=
v2
[(x0 x1 )2 + (y0 )2 ]1/2

Note this is the same as A2, namely v1 sin 1 = v2 sin 2 .

B. The Extremum Principle in Optics


A light ray travels from medium I to medium
II with refractive indices n1 and n2 respectively.
The two media are separated by a line parallel
to the x-axis. The light ray makes an angle i1
with the y-axis in medium I and i2 in medium II
(see Fig. 2). To obtain the trajectory of the ray,
we make use of another extremum (minimum or
maximum) principle known as Fermats principle of least time.
Figure 2
(B1) The principle states that between two fixed points, a light ray moves along a path such
that the time taken between the two points is an extremum. Derive the relation between
sin i1 and sin i2 on the basis of Fermats principle.
Solution:
The speed of light in medium I is c/n1 and in medium II is c/n2 ,
where c is the speed of light in vacuum. Let the two media be separated
by the fixed line y = y1 . Then time T () for light to travel from origin
(0,0) and (x0 ,y0 ) is
q
p
T () = n1 ( y12 + 2 )/c + n2 ( (x0 )2 + (y0 y1 )2 )/c

[0.5]

Theoretical Task 2 (T-2): Solutions

3 of 9

Differentiating w.r.t. and setting the derivative of T () to zero


(y12

n1
n2 (y0 )

=0
2
1/2
+ )
[(x0 )2 + (y0 y1 )2 ]1/2
n1 sin i1 = n2 sin i2

[Note: Derivation is similar to A3. This is Snells law.]


Shown in Fig. 3 is a schematic sketch of the
path of a laser beam incident horizontally on
a solution of sugar in which the concentration
of sugar decreases with height. As a consequence, the refractive index of the solution also
decreases with height.

Figure 3

(B2) Assume that the refractive index n(y) depends only on y. Use the equation obtained in B1
to obtain the expresssion for the slope dy/dx of the beams path in terms of n0 at y = 0
and n(y).

[1.5]

Solution:
From Snells law n0 sin i0 = n(y) sin i
dy
Then,
= cot i
dx

n0 sin i0 = r

n(y)

dy 2
)
dx
s
2
dy
n(y)
=
1
dx
n0 sin i0
1+(

(B3) The laser beam is directed horizontally from the origin (0,0) into the sugar solution at a
height y0 from the bottom of the tank as shown. Take n(y) = n0 ky where n0 and k
are positive constants.
Obtain an expression for x in terms of y and related Rquantities.
R
You may use:
sec d = ln(sec + tan ) + constant sec = 1/ cos or xdx
2 1 =

2
ln(x + x 1) + constant.
Solution:
Z

Note i0 = 90o so sin i0 = 1.

dy
r
=
n0 ky 2
(
) 1
n0 sin i0

Z
dx

[1.2]

Theoretical Task 2 (T-2): Solutions

4 of 9

Method I We employ the substitution


n0 ky
n0
Z d( n0 )
Z
k
p
= dx
2 1
=

Let = sec . Then


n0
ln(sec + tan ) = x + c
k
Or METHOD II
We employ the substition
n0 ky
n0
Z d( n0 )
Z
k
p
= dx
2 1
=

n0
ln
k

n0 ky
+
n0

!
n0 ky 2
(
) 1 = x + c
n0

Now continuing
Considering the substitutions and boundary condition, x = 0 for y = 0 we obtain that
the constant c = 0.
Hence we obtain the following trajectory:
!
r
n0 ky
n0 ky 2
n0
ln
+ (
) 1
x=
k
n0
n0

(B4) Obtain the value of x0 , the point where the beam meets the bottom of the tank. Take y0
= 10.0 cm, n0 = 1.50, k = 0.050 cm1 (1 cm = 102 m).
Solution:
Given y0 = 10.0 cm.
From (B3)

n0 = 1.50

k = 0.050 cm1



n0 n0 ky
x0 =
ln
+
k
n0
Here y = y0

n0 ky
n0

2

!1/2
1

[0.8]

Theoretical Task 2 (T-2): Solutions


"

5 of 9

1/2 #
n0
(n0 + ky0 )
(n0 + ky0 )
x0 =
ln
+
1
k
n0
n20

!1/2
 2
2
2

= 30 ln
+
1
1.5
1.5


"

 1/2 #
4
7
= 30 ln
+
3
9


4
+ 0.88
= 30 ln
3
= 24.0 cm

C. The Extremum Principle and the Wave Nature of Matter


We now explore between the PLA and the wave nature of a moving particle. For this we
assume that a particle moving from O to P can take all possible trajectories and we will seek
a trajectory that depends on the constructive interference of de Broglie waves.
(C1) As the particle moves along its trajectory by an infinitesimal distance s, relate the change
in the phase of its de Broglie wave to the change A in the action and the Planck
constant.
Solution:
From the de Broglie hypothesis
dB = h/mv
where is the de Broglie wavelength and the other symbols have their usual meaning
=

2
s

2
mvs
h
2A
=
h

[0.6]

Theoretical Task 2 (T-2): Solutions


(C2)

[1.2]

Recall the problem from part A where the particle


traverses from O to P (see Fig. 4). Let an opaque
partition be placed at the boundary AB between the
two regions. There is a small opening CD of width
d in AB such that d  (x0 x1 ) and d  x1 .
Consider two extreme paths OCP and ODP such
that OCP lies on the classical trajectory discussed in
part A. Obtain the phase difference CD between
the two paths to first order.

Figure 4

Solution:

A II

D
F
C

x1

Consider the extreme trajectories OCP and ODP of (C1)


The geometrical path difference is ED in region I and CF in region II.
This implies (note: d  (x0 x1 ) and d  x1 )
CD =

6 of 9

2d sin 1 2d sin 2

1
2

2mv1 d sin 1 2mv2 d sin 2

h
h
md
= 2
(v1 sin 1 v2 sin 2 )
h
=0
(from A2 or B1)

CD =

Thus near the clasical path there is invariably constructive interference.

Theoretical Task 2 (T-2): Solutions

7 of 9

D. Matter Wave Interference


Consider an electron gun at O which directs a collimated beam of electrons to a
narrow slit at F in the opaque partition
A1 B1 at x = x1 such that OFP is a straight
line. P is a point on the screen at x = x0
(see Fig. 5). The speed in I is v1 = 2.0000
107 m s1 and = 10.0000 . The potential in region II is such that the speed v2 =
1.9900 107 m s1 . The distance x0 x1
is 250.00 mm (1 mm = 103 m). Ignore
electron-electron interaction.

Figure 5
(D1) If the electrons at O have been accelerated from rest, calculate the accelerating potential
U1 .
Solution:
qU1 =

[0.3]

1
mv 2
2

9.11 1031 4 1014


J
2
= 2 9.11 1017 J
=

2 9.11 1017
eV
1.6 1019

= 1.139 103 eV (w 1100 eV )


U1 = 1.139 103 V
(D2) Another identical slit G is made in the partition A1 B1 at a distance of 215.00 nm (1 nm
= 109 m) below slit F (Fig. 5). If the phase difference between de Broglie waves ariving
at P from F and G is 2 , calculate .

Solution: Phase difference at P is


P =

2d sin 2d sin

1
2

= 2(v1 v2 )

md
sin 10 = 2
h

= 5.13

[0.8]

Theoretical Task 2 (T-2): Solutions

8 of 9

(D3) What is is the smallest distance y from P at which null (zero) electron detection maybe
expected on the screen? [Note: you may find the approximation sin( + ) sin +
cos useful]
Solution:
y

A1 II

F
215 nm

B1
x1

From previous part for null (zero) electron detection = 5.5 2


mv1

sin( + ) =

=
=
=

d sin mv2 d sin( + )

= 5.5
h
h

mv1 d sin
5.5
h
mv2 d
h
h 5.5
v1
sin
v2
m v2 d
5.5
2
sin 10
1.99
1374.78 1.99 107 2.15 107
0.174521 0.000935

This yields = 0.0036


The closest distance to P is
y =
=
=
=

(x0 x1 )(tan( + ) tan )


250(tan 9.9964 tan 10)
0.0162mm
16.2 m

The negative sign means that the closest minimum is below P.


Approximate Calculation for and y
Using the approximation sin( + ) sin + cos
The phase difference of 5.5 2 gives
mv1

d sin 10
d(sin 10 + cos 10 )
mv2
= 5.5
h
h

From solution of the previous part


mv1

d sin 10
dsin10
mv2
= 5.13
h
h

[1.2]

Theoretical Task 2 (T-2): Solutions


Therefore
mv2

9 of 9

d cos 10
= 0.3700
h

This yields 0.0036


y = 0.0162 mm = 16.2m as before

(D4) The electron beam has a square cross section of 500 nm 500 nm and the setup is 2 m
long. What should be the minimum beam flux density Imin (number of electrons per unit
normal area per unit time) if, on an average, there is at least one electron in the setup at
a given time?
Solution: The product of the speed of the electrons and number of electron per unit
volume on an average yields the intensity.
Thus N = 1 = Intensity Area Length/ Electron Speed
= Imin 0.25 1012 2/2 107
This gives Imin = 4 1019 m2 s1
R. Bach, D. Pope, Sy-H Liou and H. Batelaan, New J. of Physics Vol. 15, 033018 (2013).

[0.4]

Q T-3
Page 1 of 2
The Design of a Nuclear Reactor1

(Total Marks: 10)


235

Uranium occurs in nature as UO2 with only 0.720% of the uranium atoms being U. Neutron induced
fission occurs readily in 235U with the emission of 2-3 fission neutrons having high kinetic energy. This
fission probability will increase if the neutrons inducing fission have low kinetic energy. So by reducing the
kinetic energy of the fission neutrons, one can induce a chain of fissions in other 235U nuclei. This forms the
basis of the power generating nuclear reactor (NR).
A typical NR consists of a cylindrical tank of height H and radius R filled with a material called moderator.
Cylindrical tubes, called fuel channels, each containing a cluster of cylindrical fuel pins of natural UO2 in
solid form of height H, are kept axially in a square array. Fission neutrons, coming outward from a fuel
channel, collide with the moderator, losing energy and reach the surrounding fuel channels with low enough
energy to cause fission (Figs I-III). Heat generated from fission in the pin is transmitted to a coolant fluid
flowing along its length. In the current problem we shall study some of the physics behind the (A) Fuel Pin,
(B) Moderator and (C) NR of cylindrical geometry.
Fig-I

A1

A2
A3

A4

Fuel Pin
Data
for UO2

Fig-II

Fig-III

1. Molecular weight Mw = 0.270 kg mol-1


3. Melting point Tm = 3.138103 K

Schematic sketch of the


Nuclear Reactor (NR)
Fig-I: Enlarged view of a fuel
channel (1-Fuel Pins)
Fig-II: A view of the NR
(2-Fuel Channels)
Fig-III: Top view of NR
(3-Square Arrangement of
Fuel Channels and 4-Typical
Neutron Paths).
Only components relevant to
the problem are shown (e.g.
control rods and coolant are
not shown).

2. Density = 1.060104 kg m-3


4. Thermal conductivity = 3.280 W m-1 K-1

Consider the following fission reaction of a stationary 235U after it absorbs a neutron of negligible kinetic
energy.
235
U + 1n 94Zr + 140Ce + 2 1n +
Estimate
(in MeV) the total fission energy released. The nuclear masses are: m(235U) = 235.044 u;
94
m( Zr) = 93.9063 u; m(140Ce) = 139.905 u; m(1n) = 1.00867 u and 1 u = 931.502 MeV c-2. Ignore charge
imbalance.
Estimate N the number of 235U atoms per unit volume in natural UO2.
Assume that the neutron flux density, = 2.0001018 m-2 s-1 on the fuel is uniform. The fission crosssection (effective area of the target nucleus) of a 235U nucleus is f = 5.40010-26 m2. If 80.00% of the
fission energy is available as heat, estimate Q (in W m-3), the rate of heat production in the pin per unit
volume. 1MeV = 1.60210-13 J
The steady-state temperature difference between the center (Tc) and the surface (Ts) of the pin can be
expressed as TcTs = k F(Q,a,), where k = 1 4 is a dimensionless constant and a is the radius of the pin.
Obtain F(Q,a,) by dimensional analysis. Note that is the thermal conductivity of UO2.

0.8

0.5
1.2

0.5

Joseph Amal Nathan (BARC) and Vijay A. Singh (ex-National Coordinator, Science Olympiads) were the
principal authors of this problem. The contributions of the Academic Committee, Academic Development Group and
the International Board are gratefully acknowledged.

Q T-3
Page 2 of 2
A5
B

The desired temperature of the coolant is 5.770102 K. Estimate the upper limit au on the radius a of the
pin.

1.0

The Moderator
Consider the two dimensional elastic collision between a neutron of mass 1 u and a moderator atom of mass
A u. Before collision all the moderator atoms are considered at rest in the laboratory frame (LF). Let and
be the velocities of the neutron before and after collision respectively in the LF. Let be the velocity
of the center of mass (CM) frame relative to LF and be the neutron scattering angle in the CM frame. All
the particles involved in collisions are moving at nonrelativistic speeds.
The collision in LF is shown schematically, where L is the scattering angle (Fig-IV). Sketch the collision
schematically in CM frame. Label the particle velocities for 1, 2 and 3 in terms of , and . Indicate
the scattering angle .

Fig-IV

B1

Collision in the Laboratory Frame


1-Neutron before collision
2-Neutron after collision
3-Moderator Atom before collision
4-Moderator Atom after collision

1.0

B2
B3
B4

C1
C2

Obtain v and V, the speeds of the neutron and moderator atom in the CM frame after collision, in terms of A
and .
Derive an expression for G(, ) = Ea Eb , where Eb and Ea are the kinetic energies of the neutron, in the
LF, before and after the collision respectively and
.
Assume that the above expression holds for D2O molecule. Calculate the maximum possible fractional
energy loss
of the neutron for the D2O (20 u) moderator.
The Nuclear Reactor
To operate the NR at any constant neutron flux (steady state), the leakage of neutrons has to be
compensated by an excess production of neutrons in the reactor. For a reactor in cylindrical geometry the
leakage rate is k1 [(2.405 R)2 + ( H)2] and the excess production rate is k2 . The constants k1 and k2
depend on the material properties of the NR.
Consider a NR with k1 = 1.02110-2 m and k2 = 8.78710-3 m-1. Noting that for a fixed volume the leakage
rate is to be minimized for efficient fuel utilization, obtain the dimensions of the NR in the steady state.
The fuel channels are in a square arrangement (Fig-III) with the nearest neighbour distance 0.286 m. The
effective radius of a fuel channel (if it were solid) is 3.61710-2 m. Estimate the number of fuel channels Fn
in the reactor and the mass M of UO2 required to operate the NR in steady state.

1.0
1.0
0.5

1.5
1.0

Theoretical Task 3 (T-3) : Solutions

1 of 9

The Design of a Nuclear Reactor1


Uranium occurs in nature as UO2 with only 0.720% of the uranium atoms being 235 U. Neutron
induced fission occurs readily in 235 U with the emission of 2-3 fission neutrons having high
kinetic energy. This fission probability will increase if the neutrons inducing fission have low
kinetic energy. So by reducing the kinetic energy of the fission neutrons, one can induce a chain
of fissions in other 235 U nuclei. This forms the basis of the power generating nuclear reactor
(NR).
A typical NR consists of a cylindrical tank of height H and radius R filled with a material called
moderator. Cylindrical tubes, called fuel channels, each containing a cluster of cylindrical fuel
pins of natural UO2 in solid form of height H, are kept axially in a square array. Fission
neutrons, coming outward from a fuel channel, collide with the moderator, losing energy, and
reach the surrounding fuel channels with low enough energy to cause fission (Figs I-III). Heat
generated from fission in the pin is transmitted to a coolant fluid flowing along its length. In
the current problem we shall study some of the physics behind the (A) Fuel Pin, (B) Moderator
and (C) NR of cylindrical geometry.

Fig-I

Fig-III

Fig-II

Schematic sketch of the


Nuclear Reactor (NR)
Fig-I: Enlarged view of a
fuel channel (1-Fuel Pins)
Fig-II: A view of the NR
(2-Fuel Channels)
Fig-III: Top view of NR
(3-Square Arrangement of
Fuel
Channels
and
4-Typical Neutron Paths).
Only components relevant
to the problem are shown
(e.g. control rods and
coolant are not shown).

A. Fuel Pin
Data for UO2
1. Molecular weight Mw =0.270 kg mol
3. Melting point Tm =3.138103 K

2. Density =1.060104 kg m3
4. Thermal conductivity =3.280 W m1 K1

A1 Consider the following fission reaction of a stationary


negligible kinetic energy.
235
1

235

U after it absorbs a neutron of

U +1 n 94 Zr +140 Ce + 2 1 n + E

Joseph Amal Nathan (BARC) and Vijay A. Singh (ex-National Coordinator, Science Olympiads) were the
principal authors of this problem. The contributions of the Academic Committee, Academic Development Group
and the International Board are gratefully acknowledged.

Theoretical Task 3 (T-3) : Solutions

2 of 9

Estimate E (in MeV) the total fission energy released. The nuclear masses are: m(235 U)
= 235.044 u; m(94 Zr) = 93.9063 u; m(140 Ce) = 139.905 u; m(1 n) = 1.00867 u and 1 u =
931.502 MeV c2 . Ignore charge imbalance.

[0.8]

Solution: E = 208.684 MeV


Detailed solution: The energy released during the transformation is
E = [m(235 U) + m(1 n) m(94 Zr) m(140 Ce) 2m(1 n)]c2
Since the data is supplied in terms of unified atomic masses (u), we have
E = [m(235 U) m(94 Zr) m(140 Ce) m(1 n)]c2

= 208.684 MeV [Acceptable Range (208.000 to 209.000)]


from the given data.
A2 Estimate N the number of

235

U atoms per unit volume in natural UO2 .

[0.5]

Solution: N = 1.702 1026 m3


Detailed solution: The number of UO2 molecules per m3 of the fuel N1 is given
in the terms of its density , the Avogadro number NA and the average molecular
weight Mw as
NA
N1 =
Mw
10600 6.022 1023
= 2.364 1028 m3
=
0.270
Each molecule of UO2 contains one uranium atom. Since only 0.72% of these are
U,
N = 0.0072 N1
= 1.702 1026 m3 [Acceptable Range (1.650 to 1.750)]
235

A3 Assume that the neutron flux = 2.000 1018 m2 s1 on the fuel is uniform. The fission
cross-section (effective area of the target nucleus) of a 235 U nucleus is f = 5.400 1026
m2 . If 80.00% of the fission energy is available as heat, estimate Q (in W m3 ) the rate of
heat production in the pin per unit volume. 1MeV = 1.602 1013 J.
Solution: Q = 4.917 108 W/m3
Detailed solution: It is given that 80% of the fission energy is available as heat
thus the heat energy available per fission Ef is from a-(i)
Ef = 0.8 208.7 MeV
= 166.96 MeV
= 2.675 1011 J
The total cross-section per unit volume is N f . Thus the heat produced per unit

[1.2]

Theoretical Task 3 (T-3) : Solutions

3 of 9

volume per unit time Q is


Q = N f Ef
= (1.702 1026 ) (5.4 1026 ) (2 1018 ) (2.675 1011 ) W/m3
= 4.917 108 W/m3 [Acceptable Range (4.800 to 5.000)]
A4 The steady-state temperature difference between the center (Tc ) and the surface (Ts ) of the
pin can be expressed as Tc Ts = kF (Q, a, ) where k = 1/4 is a dimensionless constant
and a is the radius of the pin. Obtain F (Q, a, ) by dimensional analysis.
Solution: Tc Ts =

[0.5]

Qa2
.
4

Detailed solution: The dimensions of Tc Ts is temperature. We write this as


Tc Ts = [K]. Once can similarly write down the dimensions of Q, a and . Equating
the temperature to powers of Q, a and , one could state the following dimensional
equation:
K = Q a
= [M L1 T 3 ] [L] [M L1 T 3 K 1 ]
This yields the following algebraic equations
= -1 equating powers of temperature
+ = 0 equating powers of mass or time. From the previous equation we get = 1
Next + + = 0 equating powers of length. This yields = 2.
Qa2
where we insert the dimensionless factor 1/4 as sugThus we obtain Tc Ts =
4
gested in the problem. No penalty if the factor 1/4 is not written.
Note: Same credit for alternate ways of obtaining , , .
A5 The desired temperature of the coolant is 5.770 102 K. Estimate the upper limit au on
the radius a of the pin.
Solution: au = 8.267 103 m.
Detailed solution: The melting point of UO2 is 3138 K and the maximum temperature of the coolant is 577 K. This sets a limit on the maximum permissible temperature
(Tc Ts ) to be less than (3138 - 577 = 2561 K) to avoid meltdown. Thus one may
take a maximum of (Tc Ts ) = 2561 K.
Noting that = 3.28 W/m - K, we have
a2u =

2561 4 3.28
4.917 108

Where we have used the value of Q from A2. This yields au w 8.267 103 m. So
au = 8.267 103 m constitutes an upper limit on the radius of the fuel pin.
Note: The Tarapur 3 & 4 NR in Western India has a fuel pin radius of 6.090 103
m.

[1.0]

Theoretical Task 3 (T-3) : Solutions

4 of 9

B. The Moderator
Consider the two dimensional elastic collision between a neutron of mass 1 u and a moderator
atom of mass A u. Before collision all the moderator atoms are considered at rest in the

laboratory frame (LF). Let


vb and
va be the velocities of the neutron before and after collision

respectively in the LF. Let vm be the velocity of the center of mass (CM) frame relative to LF
and be the neutron scattering angle in the CM frame. All the particles involved in collisions
are moving at non-relativistic speeds
B1 The collision in LF is shown schematically with L as the scattering angle (Fig-IV). Sketch
the collision schematically in CM frame. Label the particle velocities for 1, 2 and 3 in

terms of
vb ,
va and
v
m . Indicate the scattering angle .

va

Fig-IV

vb

[1.0]

Collision in the Laboratory Frame


1-Neutron before collision
2-Neutron after collision
3-Moderator Atom before collision
4-Moderator Atom after collision

Solution:

Center of Mass Frame

Laboratory Frame

va


va vm

vb


vb vm

3
4

vm

B2 Obtain v and V , the speeds of the neutron and the moderator atom in the CM frame after
the collision, in terms of A and vb .
Solution: Detailed solution: Before the collision in the CM frame (vb vm ) and
vm will be magnitude of the velocities of the neutron and moderator atom respectively.
vb
From momentum conservation in the CM frame, vb vm = Avm gives vm = A+1
.
After the collision, let v and V be magnitude of the velocities of neutron and moderator
atom respectively in the CM frame. From conservation laws,
v = AV

and

1
1 2
1
1
(vb vm )2 + Avm
= v 2 + AV 2 .( [0.2 + 0.2])
2
2
2
2

[1.0]

Theoretical Task 3 (T-3) : Solutions

5 of 9

vb
Avb
and V = A+1
. (OR) From definition of center of mass frame
Solving gives v = A+1
vb
Avb
vm = A+1 . Before the collision in the CM frame vb vm = A+1
and vm will be magnitude of the velocities of the neutron and moderator atom respectively. In elastic
collision the particles are scattered in the opposite direction in the CM frame and so
vb
Avb
and V = A+1
( [0.2 + 0.1]).
the speeds remain same v = A+1

Note: Alternative solutions are worked out in the end and will get appropriate weightage.
B3 Derive an expression for G(, ) = Ea /Eb , where Eb and Ea are the kinetic energies of the
neutron, in the LF, before and after the collision respectively, and [(A 1)/(A + 1)]2 ,

[1.0]

Solution:
G(, ) =

1
A2 + 2A cos + 1
Ea
= [(1 + ) + (1 ) cos ] .
=
2
Eb
(A + 1)
2

2
2
2
Detailed solution: Since
va =
v +
v
m , va = v + vm + 2vvm cos ( [0.3]). Substi2
2
2
A v
vb
2Avb2
tuting the values of v and vm , va2 = (A+1)b 2 + (A+1)
2 + (A+1)2 cos ( [0.2]), so
A2 + 2A cos + 1
va2
Ea
=
.
=
vb2
Eb
(A + 1)2

G(, ) =

A2 + 1
2A
1
+
cos = [(1 + ) + (1 ) cos ] .
2
2
(A + 1)
(A + 1)
2

Alternate form
= 1

(1 )(1 cos )
.
2

Note: Alternative solutions are worked out in the end and will get appropriate weightage.
B4 Assume that the above expression holds for D2 O molecule. Calculate the maximum posa
sible fractional energy loss fl EbEE
of the neutron for the D2 O (20 u) moderator.
b
Solution: fl = 0.181
Detailed solution: The maximum energy loss will be when the collision is head
on ie., Ea will be minimum for the scattering angle = .
So Ea = Emin = Eb .
For D2 O, = 0.819 and maximum fractional loss
ceptable Range (0.170 to 0.190)]

Eb Emin
Eb

= 1 = 0.181. [Ac-

[0.5]

Theoretical Task 3 (T-3) : Solutions

6 of 9

C. The Nuclear Reactor


To operate the NR at any constant neutron flux (steady state), the leakage of neutrons has to
be compensated by an excess production
of neutrons
in the reactor. For a reactor in cylindrical
h
i


2
2
geometry the leakage rate is k1 2.405
+ H
and the excess production rate is k2 . The
R
constants k1 and k2 depend on the material properties of the NR.
C1 Consider a NR with k1 = 1.021102 m and k2 = 8.787103 m1 . Noting that for a
fixed volume the leakage rate is to be minimized for efficient fuel utilisation obtain the
dimensions of the NR in the steady state.
Solution: R = 3.175 m, H = 5.866 m.
Detailed solution: For constant volume V = R2 H,
"
2   #
d
2.405
2
+
= 0,
dH
R
H


2
2
d 2.4052 H
2.4052
+ 2 =
2 3 = 0,
dH
V
H
V
H

gives


2.405 2
R

=2


2
.
H

For steady state,


1.021 102

"

2.405
R

2
+

 2
H

#
= 8.787 103 .

Hence H = 5.866 m [Acceptable Range (5.870 to 5.890)]


R = 3.175 m [Acceptable Range (3.170 to 3.180)].

Alternative Non-Calculus Method


 to Optimize
2  
2.405
2
Minimisation of the expression
+
, for a fixed volume V =
R
H
R2 H:
2.4052 H
2
Substituting for R2 in terms of V, H we get
+ 2,
V
H
2
2
2
2.405 H 2.405 H

which can be written as,


+
+ 2.
2V
2V
H
Since all the terms are positive applying AMGM inequality for three positive terms we
get
r
r
2.4052 H
2.4052 H
2
2 H
2 H
2
4 4
+
+
2.405
2.405

2
3
3 2.405
2V
2V
H

2 =
.
3
2V
2V
H
4V 2

[1.5]

Theoretical Task 3 (T-3) : Solutions

7 of 9

The RHS is a constant. The LHS is always greater or equal to this constant implies that this is the minimum value the LHS can achieve. The minimum is achieved
2.4052 H
=
when all the three positive terms are equal, which gives the condition
2V


2
 2
2
2.405

=
2
.
H2
R
H
For steady state,
1.021 102

"

2.405
R

2
+

 2
H

#
= 8.787 103 .

Hence H = 5.866 m [Acceptable Range (5.870 to 5.890)]


R = 3.175 m [Acceptable Range (3.170 to 3.180)].
2
Note: Putting the condition in the RHS gives the minimum as 2 . From the condiH
r
4 4
3
2.4052 2
2
2.405

3
tion we get 3 =
2 =
.
H
2V
H
4V 2
Note: The radius and height of the Tarapur 3 & 4 NR in Western India is 3.192 m and
5.940 m respectively.
C2 The fuel channels are in a square arrangement (Fig-III) with nearest neighbour distance
0.286 m. The effective radius of a fuel channel (if it were solid) is 3.617102 m. Estimate
the number of fuel channels Fn in the reactor and the mass M of UO2 required to operate
the NR in steady state.
Solution: Fn = 387 and M = 9.892 104 kg.
Detailed solution: Since the fuel channels are in square pitch of 0.286 m, the effective area per channel is 0.2862 m2 = 8.180 102 m2 .
The cross-sectional area of the core is R2 = 3.142 (3.175)2 = 31.67 m2 , so the
maximum number of fuel channels that can be accommodated in the cylinder is the
31.67
integer part of 0.0818
= 387.
Mass of the fuel=387Volume of the roddensity
= 387 ( 0.036172 5.866) 10600 = 9.892 104 kg.
Fn = 387 [Acceptable Range (380 to 394)]
M = 9.892 104 kg [Acceptable Range (9.000 to 10.00)]

Note 1: (Not part of grading) The total volume of the fuel is 387 ( 0.036172
5.866) = 9.332 m3 . If the reactor works at 12.5 % efficieny then using the result of
a-(iii) we have that the power output of the reactor is 9.332 4.917 108 0.125 =

[1.0]

Theoretical Task 3 (T-3) : Solutions

8 of 9

573 MW.
Note 2: The Tarapur 3 & 4 NR in Western India has 392 channels and the mass of the
fuel in it is 10.15 104 kg. It produces 540 MW of power.
Alternative Solutions to sub-parts B2 and B3: Let be the scattering angle of the
Moderator atom in the LF, taken clockwise with respect to the initial direction of the neutron
before collision. Let U be the speed of the Moderator atom, in the LF, after collision. From
momentum and kinetic conservation in LF we have
vb = va cos L + AU cos ,
0 = va sin L AU sin ,
1
1
1 2
vb =
AU 2 + va2 .
2
2
2

(1)
(2)
(3)

Squaring and adding eq(1) and (2) to eliminate and from eq(3) we get
A2 U 2 = va2 + vb2 2va vb cos L ,
A2 U 2 = Avb2 Ava2 ,

(4)

2va vb cos L = (A + 1)va2 (A 1)vb2 .

(5)

which gives
(ii) Let v be the speed of the neutron after collision in the COMF. From definition of center
vb
of mass frame vm =
.
A+1
va sin L and va cos L are the perpendicular and parallel components of va , in the LF, resolved
along the initial direction of the neutron before collision. Transforming these to the COMF
gives va sin L and va cos L vm as the perpendicular
p and parallel components of v. Substitut2 2v v cos
ing for vm and for 2va vb cos L from eq(5) in v = va2 sin2 L + va2 cos2 L + vm
a m
L
Avb
2
. Squaring the components of v to eliminate L gives va =
and simplifying gives v =
A+1
2
v 2 + vm
+ 2vvm cos . Substituting for v and vm and simplifying gives,
va2
Ea
A2 + 2A cos + 1
.
=
=
vb2
Eb
(A + 1)2
Ea
A2 + 1
2A
1
G(, ) =
=
+
cos = [(1 + ) + (1 ) cos ] .
2
2
Eb
(A + 1)
(A + 1)
2
(OR)
vb
. After the collision, let v and V
A+1
be magnitude of the velocities of neutron and moderator atom respectively in the COMF.
From conservation laws in the COMF,
(iii) From definition of center of mass frame vm =

v = AV

and

1
1
1 2
1
(vb vm )2 + Avm
= v 2 + AV 2 .
2
2
2
2

vb
Avb
Solving gives v = A+1
and V = A+1
. We also have v cos = va cos L vm , substituting for vm
and for va cos L from eq(5) and simplifying gives

va2
Ea
A2 + 2A cos + 1
=
=
.
vb2
Eb
(A + 1)2

Theoretical Task 3 (T-3) : Solutions


G(, ) =

9 of 9

A2 + 1
Ea
2A
1
=
+
cos = [(1 + ) + (1 ) cos ] .
2
2
Eb
(A + 1)
(A + 1)
2

(OR)
vb
. After the collision, let v and V
A+1
be magnitude of the velocities of neutron and moderator atom respectively in the CM frame.
From conservation laws in the CM frame,

(iv) From definition of center of mass frame vm =

v = AV

and

1
1 2
1
1
(vb vm )2 + Avm
= v 2 + AV 2 .
2
2
2
2

Avb
vb
Solving gives v = A+1
and V = A+1
. U sin and U cos are the perpendicular and parallel
components of U , in the LF, resolved along the initial direction of the neutron before collision.
Transforming these to the COMF gives U sin and U cos + vm as the perpendicular and
2
2V vm cos . Since V = vm
parallel components of V . So we get U 2 = V 2 sin2 +V 2 cos2 +vm
2
2
we get U = 2vm (1 cos ). Substituting for U from eq(4) and simplifying gives

va2
Ea
A2 + 2A cos + 1
=
=
.
vb2
Eb
(A + 1)2
G(, ) =

A2 + 1
Ea
2A
1
=
+
cos = [(1 + ) + (1 ) cos ] .
2
2
Eb
(A + 1)
(A + 1)
2

A2 + 2A cos + 1
Note: We have va =
vb . Substituting for va , v, vm in v cos = va cos L vm
A+1
gives the relation between L and ,
cos L =

A cos + 1
.
+ 2A cos + 1

A2

Treating the above equation as quadratic in cos gives,


p
sin2 L cos L A2 sin2 L
cos =
.
A
For L = 0 the root with the negative sign gives = 180 which is not correct so,
p
cos L A2 sin2 L sin2 L
cos =
.
A
Substituting the above expression for cos in the expression for
of cos L

va2
gives an expression in terms
vb2

p
va2
Ea
A2 + 2 cos L A2 sin2 L + cos 2L
=
.
=
vb2
Eb
(A + 1)2

Q E-I
Page 1 of 6

Diffraction due to Helical Structure1

(Total marks: 10)

Introduction
The X-ray diffraction image of DNA (Fig. 1) taken in Rosalind Franklins laboratory,
famously known as Photo 51, became the basis of the discovery of the double
helical structure of DNA by Watson and Crick in 1952. This experiment will help you
understand diffraction patterns due to helical structures using visible light.
Objective
To determine geometrical parameters of helical structures using diffraction.

Figure 1: Photo 51

Figure 2: Apparatus for E-I


List of apparatus
[1]
[2]
[3]
[4]
[5]
[6]
[7]
[8]
[9]
[10]

Wooden platform
[11] Plastic clips
Laser source with its mount and base
[12] Circular black stickers
DC regulated power supply for the Laser source
[13] Mechanical pencil
Sample holder with its base
[14] Digital caliper with a mount
Left reflector (front coated mirror)
[15] Plastic scale (30 cm)
Right reflector (front coated mirror)
[16] Measuring tape (1.5 m)
Screen (10 cm x 30 cm) with its mount and base
[17] Pattern marking sheets
Plane mirror (10 cm x 10 cm)
[18] Laser safety goggles
Sample I (helical spring)
[19] Battery operated flashlight
Sample II (double-helix-like pattern printed on glass plate)
Note: Items [1], [3], [14], [15], [16] and [18] are also used in experiment E-II.

Praveen Pathak (HBCSE-TIFR, Mumbai), Charudatt Kadolkar (IIT, Guwahati), and Manish Kapoor (Christ Church College,
Kanpur) were the principal authors of this problem. The contributions of the Academic Committee, Academic Development
Group and the International Board are gratefully acknowledged.

Q E-I
Page 2 of 6
Description of apparatus
Wooden platform [1]: A pair of guiding rails, laser, reflectors, screen and sample mounts are rigidly
fixed on it.
Laser source with its mount and base [2]: Laser source of wavelength
) is fixed in a metallic mount clamped to the base using a ball joint ([20] in Fig. 3) allowing
the adjustment in X-Y-Z directions. The laser body can be rotated and clamped using the top lock-in
screw. The beam focus can be adjusted by rotating the front lens cap (red arrow in Fig. 3) to obtain a
clear and sharp diffraction pattern.
DC regulated power supply [3]: The front panel has an intensity switch (high/low), socket for laser
source connector and three USB sockets. The back panel has power switch and mains power socket
(inset of Fig. 4).

Figure 3: Laser source and sample holder.


[20] Ball joint.

Figure 5: Left reflector and screen


Figure 4: DC regulated power supply
Sample holder with its base [4]: Use the top locking screw to affix the samples in it (Fig. 3). The
sample holder can be adjusted horizontally, vertically and rotated.
Left reflector [5]: This reflector is fixed to the platform (Fig. 5). Do not use the side marked X.
Right reflector [6]: This reflector is fixed to the platform and is removable (It will be removed in
experiment E-II). Do not use the side marked X.
Screen with its mount [7]: The screen is mounted on ball joint and a base allowing rotational
adjustments in all directions (Fig 5). The screen can be fixed as shown in Fig. 2 or Fig. 6 as required.

Q E-I
Page 3 of 6
Sample I [9]: A helical spring fixed on a circular mount using white acrylic plates.
Sample II [10]: A double-helix-like pattern
printed on a glass plate which is fixed on a
circular mount.
Digital caliper with a mount [14]: Digital
caliper is fixed to a mount (the mount is
required in E-II). It has an On/Off switch, a
switch to reset the reading to zero, a mm/inch
selector (keep on mm), a locking screw and a
knob for moving the right jaw. The digital
caliper can be used to make measurements on
pattern marking sheets.
Pattern marking sheets [17]: The given Figure 6: Alternate position of screen compared to
that shown in Fig. 2
pattern marking sheets can be folded in half and
clipped onto the screen using the plastic clips. Ensure that you mark the diffraction pattern within the
rectangular box.
Theory
A laser beam of wavelength , falling normally on a cylindrical wire of diameter , is diffracted in the
direction perpendicular to the wire. The resulting intensity pattern as observed on a screen is shown in
Fig. 7.

Figure 7: Schematic of the diffraction pattern due to single cylindrical


wire of diameter .
The intensity distribution as a function of angle
)

with the incident direction is given by

)*

The central spot is bright and for other angles, when


Thus the intensity distribution has
minimum at the angle

Here

Figure 8: Schematic of
diffraction pattern due to two
cylindrical wires

refers to both sides of the central spot (

) is zero, the intensity vanishes.


, given by

).

The diffraction pattern due to two parallel identical wires kept at a distance d from each other (Fig. 8)
is a combination of two patterns (diffraction due to a single wire and interference due to two wires).
The resultant intensity distribution is given by,
)
where

Q E-I
Page 4 of 6
For a screen placed at a large distance D from the wire, the
positions of the minima on the screen are observed at
due to diffraction and at
(
)
due to interference (where
). Similarly for
a set of four identical wires (Fig. 9), the net intensity
distribution is a combination of diffraction from each wire and
interference due to pairs of wires and hence depends on ,
and . In other words, the combination of three different
intensity patterns is observed.

Figure 9: Set of four wires

Initial adjustments
1. Switch on the laser source and adjust both reflectors so that the laser spot falls on the screen.
2. Use the plastic scale and adjust the laser mount and reflectors such that the laser beam is parallel
to the wooden platform.
3. Make sure that the laser spot falls near the centre of the screen.
4. Switch off the laser source. Clamp the pattern marking sheet on the screen.
5. Clamp the given plane mirror on the screen using plastic clips and switch on the laser again.
6. Adjust the screen so that the laser beam retraces its path back to the laser source. Remove the
mirror once your alignment is completed.
7. Lights in the cubicle may be switched on/off as required.

Experiment
Part A: Determination of geometrical parameters of a helical spring2
Sample I is a helical spring of radius and pitch made of a wire of uniform thickness as shown
in Fig. 10(a). When viewed at normal incidence its projection is equivalent to two sets of parallel
wires of the same thickness separated by distance
and angle
between them (Fig. 10(b)).

Figure 10: (a) Typical view of helical spring (b)


Schematic diagram when viewed at normal incidence

Reference: G. Braun, D. Tierney and H. Schmitzer, Phys. Teach. 49, 140 (2011).

Q E-I
Page 5 of 6

Mount sample I in the sample holder ensuring that the spring is vertical.
Obtain a clear and sharp X-shaped diffraction pattern on the pattern marking sheet.
For this you may adjust
- laser beam focus (rotate lens cap)
- beam orientation (rotate the laser body so that only two turns of the spring are illuminated)
- laser intensity (high/low switch on power supply)
- ambient light (by switching on or off cubicle light)
If the central maximum is very bright, you may stick circular black stickers on the pattern marking
sheet to reduce scattering.
Tasks
A1
A2
A3
A4
A5
A6
A7
A8

Description
Mark the appropriate positions (using given pencil [13]) of the intensity minima to determine
and
on the both sides of the central spot on the pattern-marking sheet. Please label your patternmarking sheets as P-1, P-2 etc.
Measure the appropriate distances using digital calipers and record them in Table A1 for
determining .
Plot a suitable graph, label it Graph A1 and from the slope, determine .
Measure the appropriate distances and record them in Table A2 for determining .
Plot a suitable graph, label it Graph A2 and from the slope, determine .
From the X-shaped pattern, determine the angle .
Express in terms of
and and calculate .
Express in terms of and
and calculate (neglect ).
Part B: Determination of geometrical parameters of double-helix-like pattern
Figure 11(a) shows two turns of a double helix. Fig. 11(b) is a two-dimensional projection of this
double helix when viewed at normal incidence. Each helix of thickness
has an angle
and
perpendicular distance
between turns. The separation between two helices is . Sample II is a
double-helix-like pattern printed on glass plate (Fig. 12), whose diffraction pattern is similar to that of
a double helix. In this part, you will determine the geometrical parameters of sample II.

Figure 11: (a) Typical view of double-helical spring (b) Its schematic diagram when viewed at normal
incidence.

Marks
0.7
0.5
0.7
0.8
0.6
0.2
0.2
0.2

Q E-I
Page 6 of 6

Figure 12: Double-helix-like pattern of sample II

Tasks
B1
B2
B3
B4
B5
B6
B7
B8

Mount the sample II in sample holder.


Attach a new pattern-marking sheet on the screen.
Obtain clear and sharp X-shaped diffraction pattern on the screen.
Description
Mark the appropriate positions of the minima on either side of the central spot to determine
and . You can use more than one pattern marking sheets.
Measure the appropriate distances and record them in Table B1 for determining
Plot suitable graph, label it Graph B1 and from the slope, determine .
Measure the appropriate distances and record them in Table B2 for determining .
Plot suitable graph, label it Graph B2 and from the slope determine .
Measure the appropriate distances and record them in Table B3 for determining
Plot suitable graph, label it Graph B3 and from the slope, determine .
From the X-shaped pattern, determine the angle .

Marks
1.1
0.5
0.5
1.2
0.5
1.6
0.5
0.2

S E-I
Page 1 of 6

Diffraction due to Helical Structure1


Tasks
A1

Part A: Determination of geometrical parameters of a helical spring


Description
Number of attached pattern marking sheet(s) for Part A: 2 with label(s): P1, P2 (patterns on page 7)

Marks
0.7

Table A1: Observations from pattern P1


Sr. No.

Order ( )

24.40

47.24

70.69

94.08

117.53

140.28

A2

in mm

0.5

Graph A1
160.00
140.00

y = 23.247x + 1.004

(xn-x-n) in mm

120.00
100.00
80.00
60.00
40.00

A3

0.7

20.00
0.00
0

Graph A1 for determination of : versus (


Slope of the graph A1 = 23.25 mm
Calculation of :

Praveen Pathak (HBCSE-TIFR, Mumbai), Charudatt Kadolkar (IIT, Guwahati), and Manish Kapoor (Christ Church College,
Kanpur) were the principal authors of this problem. The contributions of the Academic Committee, Academic Development
Group and the International Board are gratefully acknowledged.

S E-I
Page 2 of 6

Table A2: Observations from pattern P1


Sr. No.

in mm

9.39

13.43

17.53

28.98

33.53

37.66

41.61

12

52.93

13

56.76

10

14

61.03

11

15

64.74

A4

0.8

Graph A2
70.00
y = 3.9527x + 5.6225

(xm-x-m) in mm

60.00

A5

50.00
40.00
30.00
20.00

0.6

10.00
0.00
0

10

15

20

m
Graph A2 for determination of :
Slope of the graph A2 = 3.95 mm
Calculation of :

versus (

A6
A7

0.2
Expression of

in terms of

and

0.2

S E-I
Page 3 of 6

Expression of

in terms of

and

A8

0.2
mm
Total

Part B: Determination of geometrical parameters of double-helix-like pattern


Tasks
Description
B1
Attached pattern marking sheet number(s): 2 with label(s): P3, P4 (patterns on page 7)

3.9
Marks
1.1

Table B1: Observations from pattern P3

Order (

Sr. No.

B2

in mm

21.24

41.12

62.41

84.40

104.41

124.25

0.5

Graph B1
140.00
y = 20.769x + 0.2807

B3

(xn-x-n) in mm

120.00
100.00
80.00

0.5

60.00
40.00
20.00
0.00
0

n
Graph B1 for determination of

versus (

S E-I
Page 4 of 6
Slope of the graph B1 = 20.8 mm
Calculation of :

Table B2: Observations from pattern P3

in mm

Sr. No.
1

5.84

10.29

14.83

18.84

26.44

30.65

35.26

38.34

B4

1.2

Graph B2
45.00
40.00

y = 4.0692x + 2.2154

(xm-x-m) in mm

35.00

B5

30.00
25.00
20.00
15.00

0.5

10.00
5.00
0.00
0

m
Graph B2 for determination of :
Slope of the graph B2 = 4.07 mm
Calculation of :

versus (

10

S E-I
Page 5 of 6

Table B3 Observations from pattern P4


Order (

Sr. No.

B6

in mm

11.64

15.77

19.71

26.33

30.14

33.69

39.62

10

43.70

11

47.75

1.6

Graph B3
60.00

(xl-x-l) in mm

50.00

y = 3.5231x + 8.6778
R = 0.9988

40.00
30.00
20.00

B7

0.5
10.00
0.00
0

10

12

l
Graph B3 for determination of : versus (
Slope of the graph B3 = 3.52 mm
Calculation of :

B8

0.2
Total

Reference for Part A : G. Braun, D. Tierney and H. Schmitzer, Phys. Teach. 49, 140 (2011).

6.1

S E-I
Page 6 of 6

Pattern P1 (

Pattern P3 (

Pattern P2

Pattern P4 (

Q E-II
Page 1 of 6

Diffraction due to surface tension waves on water1


Introduction
Formation and propagation of waves on a liquid surface are important and well-studied phenomena.
For such waves, the restoring force on the oscillating liquid is partly due to gravity and partly due to
surface tension. For wavelengths much smaller than a critical wavelength, c, the effect of gravity is
negligible and only surface tension effects need be considered (

, where is the surface

tension, is the density of the liquid and g is the acceleration due to gravity).
In this part, you will study surface tension waves on the surface of a liquid, which have wavelengths
smaller than c. Surface tension is a property of liquids due to which the liquid surface behaves like a
stretched membrane. When the liquid surface is disturbed, the disturbance propagates as a wave just
as on a membrane. An electrically-driven vibrator is used to produce waves on the water surface.
When a laser beam is incident at a glancing angle on these surface waves, they act as a reflection
grating, producing a well-defined diffraction pattern.
Surface tension waves are damped (their amplitude gradually decreases) as they propagate. This
damping is due to the viscosity of the liquid, a property where adjacent layers of a liquid oppose
relative motion between them.
Objective
To use diffraction from surface tension waves on water to determine surface tension and viscosity of
the given water sample.
List of apparatus
[1]
[2]
[3]
[4]
[5]
[6]
[7]
[8]
[9]

Light meter (connected to light sensor


assembly)
Light sensor assembly mounted on vernier
caliper placed on a screen base
Tablet computer (used as sine wave generator)
Digital multimeter
Vibrator control box
Wooden platform
Track for moving light sensor assembly
DC regulated power supply
Hex key, measuring tape and plastic scale

Figure 1: Wooden platform unit


[10] Scale and rider with vibrator position marker
[11] Vibrator assembly
[12] Water tray
[13] Plastic cover
[14] Assembly for adjusting vibrator height
[15] Laser source 2
(Wavelength, L = 635 nm, 1nm = 10-9 m)
1

Shirish Pathare (HBCSE, Mumbai) and K G M Nair (CMI, Chennai) were the principal authors of this
problem. The contributions of the Academic Committee, Academic Development Group and the International
Board are gratefully acknowledged.

Q E-II
Page 2 of 6
[16]

Water sample for the experiment

[17] 500 ml measuring cylinder


Figure 2: Vibrator/laser source unit

Description of apparatus
a) Tablet computer as sine wave generator

Note

[18]: Power Switch


[19]: Volume up
[20]: Volume down
[21]: Charging port
[22]: Socket for Audio connector pin of cable coming from vibrator control
box[5]
Figure 3: Switches and sockets of the tablet
Keep the tablet always charging.
Gently press the power switch once to display initial screen.
Keep the output volume at maximum using the Volume up button[19].

Touch and slide


Tap the icon [24] to start the
the icon[23] to
sine wave generator
unlock
Figure 4: Initial screens of the tablet
[25]: Waveform selector (always keep at SIN)
[26]: Amplitude slider
[27]: Frequency slider
[28]: Frequency value field [Hz]
[29]: Application status indicator/switch
OFF - the sine wave generator is OFF
ON - the sine wave generator is ON
Figure 5: The sine wave generator application
To vary frequency
Tap frequency value field [28] (Fig. 5) to reveal number pad
Use backspace button [30] to erase frequency value
Enter the required frequency, and press Finished button[31]

Figure 6: Screen showing number pad to enter frequency value


To vary amplitude
Use amplitude slider [26] on tablet screen or the variable knob [33] on vibrator control box [5] to vary
output amplitude.

Q E-II
Page 3 of 6
b) Vibrator control box, digital multimeter, DC regulated power supply and their connections

[32]: Sockets to connect cables


from multimeter
[33]: Knob for varying amplitude
of the sine wave
[34]: Socket for pin of the cable
from the vibrator assembly
[35]: USB pin to be connected to
DC regulated power supply
[36]: Audio pin to be connected
to the tablet

[37]: Vibrator strip

Figure 7: Vibrator control box[5]

Figure 9: Digital multimeter[4]

Figure 10: Laser source 2 [15] (mounted


on a metal block) with connector [42]

[38]: Pin of the cable from


vibrator assembly
Figure 8: Vibrator assembly[11]
[39]: AC/DC
selector switch
[40]: Range
selector knob
[41]: Input
sockets

[43]: Intensity switch (keep on High


position)
[44]: USB socket for USB pin from
vibrator control box
[45]: Socket for connector from laser
source 2
Figure 11: DC regulated power supply[8]

[36][22]
[38][34]
[41][32]
[35][44] and [42][45]
Figure 12: Connections between tablet, vibrator control box and DC regulated power supply
c) Light sensor assembly and light meter

[46]: Circular aperture on the light sensor


[47]: Power switch of the light meter
[48]: A, B, C Sensitivity ranges of the light meter
Figure 13: Light sensor assembly and light meter

One jaw of the vernier caliper


Tighten the screw using
fits into a slot at the back of the the hex key
light sensor
Figure 14: Attaching light sensor assembly

Q E-II
Page 4 of 6
Initial Adjustments

Figure 15:
Figure 16: Base
Figure 17: Correct position of the vibrator strip and black
Removing the right screws touching
knob for height adjustment
reflector
the wooden strip
1. Disconnect the laser 1 connector and insert the laser 2 connector into the socket of the DC regulated
power supply. Note: Laser 2 has been already adjusted for a specific angle of incidence. Do not touch
the laser source!
2. Remove the right reflector used in E-I by turning the bolt under the wooden platform (Fig. 15).
3. Remove the screen used in E-I and insert the light sensor assembly into the screen base. Place the
screen base between the guiding rails of the track.
4. Position the wooden platform [6] with its base screws touching the wooden strip attached to the
working table (Fig. 16).
5. Raise the side flap of the plastic cover on the vibrator/laser source unit. Pour exactly 500 ml of the
water sample into the tray [12] using the measuring cylinder [17].
6. Switch on the laser. Locate the reflected laser spot on the light sensor. As you move the light sensor
assembly back and forth along the track, the laser spot should move vertically and not at an angle to
the vertical. Minor lateral adjustment of the wooden platform and vertical movement of light sensor
assembly will allow you to get the laser spot exactly on the aperture. The intensity shown by the light
meter will be maximum, if the centre of the laser spot coincides with the centre of the aperture,.
7. The vibrator strip has already been arranged in the correct vertical position. Do NOT change the
black knob of the height adjustment assembly [14] (Fig. 17).
8. The vibrator assembly can be moved back and forth horizontally. Vibrator position marker
indicates the position of the assembly on the scale [10].
9. While recording data, keep the flap of the plastic cover lowered in order to protect the water surface
from air currents.

Experiment
Part C: Measurement of angle between the laser beam and the water surface

Q E-II
Page 5 of 6
Figure 18: Measurement of angle
Tasks
C1
C2

Description
Move the light sensor assembly in suitable steps along the track. Note down the Xdisplacement of
the assembly and the corresponding Y-displacement of the laser spot. Record your readings in
Table C1. (Select appropriate range in the light meter.)
Plot a suitable graph (label it Graph C1) and determine the angle in degrees from its slope.

Marks
1.0
0.6

Part D: Determination of surface tension of the given water sample


From diffraction theory it can be shown that
(1)
where,

is the wave number of the surface tension waves,

w and L being the wavelengths of the surface tension waves and the laser respectively.
The angle is the angular distance between the central maximum and the first-order
maximum (Fig. 19).
The vibration frequency (f) of the waves is related to the wave number k by

where,

(2)

, is the density of the water and q is an integer.

Figure 19: Schematic diagram of the apparatus


1. Fix the light sensor assembly (using the tightening bolt at the screen base) at the position shown
in Fig. 1. Select the appropriate range on the light meter.
Task
D1

Description
Measure the length l1 between the light sensor aperture and outer edge of the water tray. You will see
a line where the laser strikes the water surface. The centre of this line is the point of incidence of the
laser. Measure l2, the distance of this point from the edge. Obtain L. Record it on your answersheet.
2. Set the vibrator position marker at 7.0 cm mark on the horizontal scale [10].
3. Set the sine wave frequency to 60 Hz and adjust its amplitude such that the first- and secondorder maxima of the diffraction pattern are clearly visible (Fig. 19 inset).

Marks
0.3

Q E-II
Page 6 of 6
Tasks
D2

D3

D4

Description
Marks
Measure the distance between the second-order maximum above and below the central maximum.
Hence calculate x1. Record your observations in Table D1. Repeat this by increasing the frequencies
2.8
in appropriate steps.
Identify the appropriate variables for a suitable graph whose slope would give the value of q. Enter
the variable values in Table D2. Plot the graph to find q (label it Graph D1). Write down equation
0.9
2 with the appropriate integer value of q.
From the equation 2, identify the appropriate variables for a suitable graph whose slope would give
the value of . Enter the variable values in Table D3. Plot the graph to determine (label it Graph
1.2
-3
D2). ( =1000 kg.m ).
Part E: Determination of the attenuation constant, and the viscosity of the liquid,
The surface tension waves are damped due to the viscosity of water. The wave amplitude, h, decreases
exponentially with the distance, s, measured from the vibrator,
(3)
where, h0 is the amplitude at the vibrator position and is the attenuation constant.
Experimentally, amplitude h0 can be related to the voltage (Vrms) applied to the vibrator assembly as,
(

(4)

The attenuation constant is related to the viscosity of the liquid as


(5)
where, is the viscosity of the liquid.
1. Set the vibrator position marker at 8.0 cm.
2. Adjust the frequency to 100 Hz.
3. Adjust the light sensor using the vernier caliper such that the first-order maximum of the
diffraction pattern falls on the aperture.
4. Adjust the amplitude of sine wave (Vrms) such that the reading in the light meter is 100 on
range A. Note down Vrms corresponding to the light meter reading.
5. Move the vibrator away from the point of incidence of the laser in steps of 0.5 cm and adjust
Vrms to get the light meter reading 100. Note down corresponding Vrms.
Tasks
E1
E2
E3

Description
Record your data for every step in Table E1.
Plot a suitable graph (label it Graph E1) and determine the attenuation constant from its slope.
Calculate the viscosity of the given water sample.

Marks
1.9
1.0
0.3

S E-II
Page 1 of 6
Diffraction due to surface tension waves on water1
Part C: Measurement of angle,
[C1]

Table C1

Obs.
no.

X /cm

Y /cm

1
2
3
4
5
6
7
8
9
10

2.0
4.0
6.0
8.0
10.0
12.0
14.0
16.0
18.0
20.0

0.136
0.285
0.425
0.549
0.703
0.846
0.965
1.124
1.251
1.390

[C2]
Graph C1 for determination of : X versus Y

Shirish Pathare (HBCSE, Mumbai)and K G M Nair (CMI, Chennai) were the principal authors of this problem. The
contributions of the Academic Committee, Academic Development Group and the International Board are gratefully
acknowledged.

S E-II
Page 2 of 6
Slope = 0.0699

= 4.0
Part D: Determination of the surface tension of the liquid
[D1]:
l1 = 98.5 cm

l2 = 5.5 cm

L = 1.04 m

[D2]:
Table D1
Obs.
no.
1
2
3
4
5
6
7
8
9
10
11

f /Hz

2x2 /cm

x1 /cm

x1 /m

60
70
80
90
100
110
120
130
140
150
160

0.782
0.880
0.966
1.030
1.096
1.184
1.253
1.336
1.415
1.489
1.545

0.196
0.220
0.242
0.258
0.274
0.296
0.313
0.334
0.354
0.372
0.386

0.00196
0.00220
0.00242
0.00258
0.00274
0.00296
0.00313
0.00334
0.00354
0.00372
0.00386

[D3]:

1 s 2p sin q
f = 2
(x1 )q

4p r l L
2

ln

1
ln
2
4

2 sin

q
ln x 1

2

S E-II
Page 3 of 6

Graph for determination of q: ln(f) versus ln(x1)

Table D2
Obs.
No.
1
2
3
4
5
6
q7= 2.90
8
9
10
11

S E-II
Slope = 1.45

ln x1
-6.235
-6.119
-6.024
-5.960
-5.900
-5.823
-5.767
-5.702
-5.644
-5.594
-5.557

Determination of surface tension:


Equation 2:

[D4]:
Graph for determination of : f 2 versus x13
Table D3
Obs.
No.
1
2
3
4
5
6
7
8
9
10
11

f 2( 103)
/Hz2
3.6
4.9
6.4
8.1
10.0
12.1
14.4
16.9
19.6
22.5
25.6

x13(10-8)
/m3
0.75
1.07
1.42
1.72
2.06
2.59
3.07
3.73
4.44
5.15
5.75

Surface Tension:

Calculations:

Slope = 4.39 1011 Hz2/m3

ln f
4.094
4.248
Page 4 of 6
4.382
4.500
4.605
4.700
4.787
4.868
4.942
5.011
5.075

S E-II
Page 5 of 6
Slope

2 sin
3

1000

2 3 . 14
( 635 10

( 0 . 0698
)

s
7.415 1015 = 4.39 1011
1000

\s = 59.2mN/m

Part E: Determination of the viscosity of the water sample


[E1]: Frequency of the signal generator = 100 Hz
Table E1
Obs.
No.
1
2
3
4
5
6
7
8
9
10
11

s
Vrms /V
/cm
8.0
8.5
9.0
9.5
10.0
10.5
11.0
11.5
12.0
12.5
13.0

0.0276
0.0330
0.0385
0.0441
0.0534
0.0622
0.0745
0.0870
0.1050
0.1215
0.1412

[E2]:
Graph for determination of : ln (Vrms) versus s

ln(Vrms)
-3.590
-3.411
-3.257
-3.121
-2.930
-2.777
-2.597
-2.442
-2.254
-2.108
-1.958

( 1 . 04 )

)
3

S E-II
Page 6 of 6

Slope = 0.331 cm-1

\d = 0.4 0.3310 = 0.1324cm-1


= 13.2 m-1
[E3]:
Determination of viscosity,:

8 f
3

3
8 f

3
8

13 . 2 59 . 2 10
3 . 14 100

= 0.93 mPa.s

0 . 933 mPa.s

Potrebbero piacerti anche